You are on page 1of 207

KENDRIYA VIDYALAYA GACHIBOWLI, GPRA CAMPUS, HYD–32

SAMPLE PAPER TEST 01 FOR BOARD EXAM (2022-23)


(ANSWERS)
SUBJECT: MATHEMATICS (041) MAX. MARKS : 80
CLASS : XII DURATION: 3 HRS
General Instructions:
1. This Question paper contains - five sections A, B, C, D and E. Each section is compulsory.
However, there are internal choices in some questions.
2. Section A has 18 MCQ’s and 02 Assertion-Reason based questions of 1 mark each.
3. Section B has 5 Very Short Answer (VSA)-type questions of 2 marks each.
4. Section C has 6 Short Answer (SA)-type questions of 3 marks each.
5. Section D has 4 Long Answer (LA)-type questions of 5 marks each.
6. Section E has 3 source based/case based/passage based/integrated units of assessment (4
marks each) with sub parts.

SECTION – A
Questions 1 to 20 carry 1 mark each.
1. For any matrix A = [aij], if cij denotes its cofactors then find the value of a11c12 + a12c22 + a13c32.
(a) 1 (b) -1 (c) 0 (d) none of these
Ans: Zero

1 2   3 1   7 11
2. If     , then write the value of k.
3 4   2 5  k 23
(a) 17 (b) -17 (c) 13 (d) -13
Ans:

 
3. The magnitude of each of the two vectors a and b , having the same magnitude such that the
angle between them is 60° and their scalar product is 9/2, is
(a) 2 (b) 3 (c) 4 (d) 5
Ans: (b) 3

1
4. If f ( x)  x 2 sin , where x ≠ 0, then the value of the function f at x = 0, so that the function is
x
continuous at x = 0, is
(a) 0 (b) -1 (c) 1 (d) None of these
1
Ans (a) f ( x)  x 2 sin , where x ≠ 0
x
Hence, value of the function f at x = 0, so that it is continuous at x = 0 is 0.
2
dx
5. The value of e
0
1
sin x
is

(a)  (b) 0 (c) 3  (d) /2

Prepared by: M. S. KumarSwamy, TGT(Maths) Page - 1-


Ans: (a) 

6. If m and n are the order and degree, respectively of the differential equation
2
d y
3
 dy 
2
y    x 3  2   xy  sin x , then write the value of m + n.
 dx   dx 
(a) 1 (b) 2 (c) 3 (d) 4
Ans:

7. Corner points of the feasible region for an LPP are (0, 2), (3, 0), (6, 0), (6, 8) and (0, 5).
Let F =4x + 6y be the objective function. The minimum value of F occurs at
(a) Only (0, 2)
(b) Only (3, 0)
(c) the mid-point of the line segment joining the points (0, 2) and (3, 0)
(d) any point on the line segment joining the points (0, 2) and (3, 0)
Ans: (d) any point on the line segment joining the points (0, 2) and (3, 0)

Hence, minimum value of F occurs at any points on the line segment joining the points (0, 2) and
(3, 0).

8. The projection of the vector 2i  3 j  2k ℎ i  2 j  k is


(a) 10/√6 (b) 10/√3 (c) 5/√6 (d) 5/√3
Ans:

Prepared by: M. S. KumarSwamy, TGT(Maths) Page - 2-


sin 2 x  cos2 x
9. Evaluate:  sin 2 x cos 2 x dx
(a) tanx – cotx + C (b) –tanx + cotx + C
(c) tanx + cotx + C (d) –tanx – cotx +C
Ans: (c) tanx + cotx + C

2  1 10


10. If x    y      , find the value of x.
3 1 5
(a) 1 (b) 2 (c) 3 (d) 4
Ans:

11. Feasible region (shaded) for a LPP is shown in the given figure.
The maximum value of the Z = 0.4x + y is

(a) 45 (b) 40 (c) 50 (d) 41

Ans: (d) 41

2x  5 3
12. If = 0, find x.
5x  2 9
(a) 13 (b) 3 (c) -13 (d) √3
Ans:

Prepared by: M. S. KumarSwamy, TGT(Maths) Page - 3-


13. If A is a non-singular matrix of order 3 and |A| = – 4, find |adj A|.
1
(a) 4 (b) 16 (c) 64 (d)
4
Ans:

14. If A and B are two independent events with P(A) = 3/5 and P(B) = 4/9, then find P ( A  B) .
(a) 1/9 (b) 2/9 (c) 1/3 (d) 4/9
Ans:

dy
15. If  y sin 2 x , y(0) = 1, then solution is
dx
2 2
(a) y = esin x
(b) y = sin2x (c) y = cos2x (d) y = ecos x

Ans:

d2y
16. If y = 5 cos x – 3 sin x, then is equal to:
dx 2
(a) –y (b) y (c) 25y (d) 9y
Ans: (a) –y

x 3 y  2 z 5
17. If the equation of a line AB is   , find the direction ratios of a line parallel to
1 2 4
AB.
(a) 1, 2, 4 (b) 1, 2, –4 (c) 1, –2, –4 (d) 1, –2, 4
Ans: (d) The direction ratios of line parallel to AB is 1, –2 and 4.

18. If a line makes angles α, β, γ with the positive direction of co-ordinates axes, then find the value
of sin2α + sin2β + sin2γ.
(a) 1 (b) 2 (c) 3 (d) 4
Ans:

Prepared by: M. S. KumarSwamy, TGT(Maths) Page - 4-


ASSERTION-REASON BASED QUESTIONS
In the following questions, a statement of assertion (A) is followed by a statement of Reason (R).
Choose the correct answer out of the following choices.
(a) Both A and R are true and R is the correct explanation of A.
(b) Both A and R are true but R is not the correct explanation of A.
(c) A is true but R is false.
(d) A is false but R is true.

x 5 y  4 z 6
19. Assertion: If the cartesian equation of a line is   , then its vector form is
3 7 2

r  5i  4 j  6k   (3i  7 j  2k )
Reason: The cartesian equation of the line which passes through the point (–2, 4, –5) and
x3 y4 z8 x3 y4 z 8
parallel to the line given by   is   .
3 5 6 2 4 5
Ans: (c) A is true but R is false.
x 5 y  4 z 6
In assertion the given cartesian equation is  
3 7 2
 
    
 a  5i  4 j  6k , b  3i  7 j  2k 
  
The vector equation of the line is given by r  a   b

 r  5i  4 j  6k   (3i  7 j  2k )
Thus Assertion is correct.
In reason it is given that the line passes through the point (–2, 4, –5) and is parallel to
x3 y4 z8
 
3 5 6
Clearly, the direction ratios of line are (3, 5, 6).
Now the equation of the line (in cartesian form) is
x  (2) y  4 z  (5) x2 y4 z 5
    
3 5 6 3 5 6
Hence, Reason is wrong.
−1  1 1 
20. Assertion (A): The domain of the function 2 is  ,     ,  
 2 2 
−1 
Reason (R): (−2) = −
4
Ans: (c) A is true but R is false.
1 1
–1 is defined if ≤ −1 ≥ 1. Hence, –12 will be defined if ≤  or ≥ .
2 2
Hence, A is true.
 
The range of the function is [0, ] −  
2
R is false.
SECTION – B
Questions 21 to 25 carry 2 marks each.

21. Prove that the Greatest Integer Function f : R R, given by f(x) = [x] is neither one-one nor
onto. Where [x] denotes the greatest integer less than or equal to x.

Prepared by: M. S. KumarSwamy, TGT(Maths) Page - 5-


Ans: Given f : R R defined by f(x) = [x]
For one-one: We know by definition that for a ≤ x < a +1, f(x) = a, a is an integer,
i.e. for x1, x2 ∈ [a, a+1), x1 ≠ x2, f(x1) = f(x2) = a.
Hence, not one-one.
For onto: For y (non-integer) ∈ R in co-domain there does not exist x ∈ R in domain such that
f(x) = y. Hence, not onto.
OR
 1 
If sin  sin 1  cos 1 x  =1, then find the value of x.
 5 
Ans:

1  cos kx
 x sin x , if x  0
22. Find the value (s) of k so that the following function f ( x )   is continuous at
 1
, if x  0
 2
x = 0.
Ans:

    
23. If | a  b |2  ( a.b) 2  144 and | a | 4 , then find the value of | b | .

 
24. Find the angle between the vectors a  i  j  k and b  i  j  k .

Prepared by: M. S. KumarSwamy, TGT(Maths) Page - 6-


OR
x  3 y 1 z  5
Find the coordinates of the point where the line   cuts the XY plane.
3 1 5
Ans:

 1  x  1 x  dy 1
25. If y = sin 1   , then show that 
 2  dx 2 1  x 2
Ans:

SECTION – C
Questions 13 to 22 carry 3 marks each.
26. In a group of 50 scouts in a camp, 30 are well trained in first aid techniques while the remaining
are well trained in hospitality but not in first aid. Two scouts are selected at random from the
group. Find the probability distribution of number of selected scouts who are well trained in first
aid.
Ans:
Let X be no. of selected scouts who are well trained in first aid. Here random variable X may
have value 0, 1, 2.
20
C 20 19 38
Now, P(X = 0) = 50 2  
C2 50  49 245

Prepared by: M. S. KumarSwamy, TGT(Maths) Page - 7-


20
C1  30C1 20  30  2 120
P(X = 1) =  
50
C2 50  49 245
C2 30  29 87
30
P(X = 2) =  
C2 50  49 245
50

Now probability distribution table is

OR
An urn contains 5 white and 8 white black balls. Two successive drawing of three balls at a time
are made such that the balls are not replaced before the second draw. Find the probability that the
first draw gives 3 white balls and second draw gives 3 black balls.


x sin x
27. Evaluate:  1  cos
0
2
x
dx

OR
3
Evaluate:  | x 2  2 x | dx.
1
Ans:

Prepared by: M. S. KumarSwamy, TGT(Maths) Page - 8-


dx
28. Evaluate:  9x 2
 6 x  10
.

29. Find the general solution of the following differential equation; x dy – (y + 2x2)dx = 0
Ans:

OR
dy
Solve:  1  x  y  xy
dx

Prepared by: M. S. KumarSwamy, TGT(Maths) Page - 9-


30. Solve the following Linear Programming Problem graphically:
Maximise Z = x + 2y subject to the constraints: x + 2y ≥ 100; 2x – y < 0; 2x + y ≤ 200; x, y ≥ 0
Ans: Maximise Z = x + 2y
Subject to constraints : x + 2y ≥ 100, 2x – y < 0, 2x + y ≤ 200 and x, y ≥ 0.
Converting the inequations into equations, we obtain the lines
l1 : x + 2y = 100 ...(i)
l2 : 2x – y = 0 ...(ii)
l3 : 2x + y = 200 ...(iii)
l4 : x = 0 ...(iv)
and l5 : y = 0 ...(v)
x y
By intercept form, we get  1
100 50
⇒ The line l1 meets the coordinate axes at (100, 0) and (0, 50).
l2 : 2x = y
⇒ The line l2 passes through origin and (50, 100), (100, 200)
x y
l3 :  1
100 200
⇒ The line l3 meets the coordinates axes at (100, 0) and (0, 200).
l4 : x = 0 is the y-axis, l5 : y = 0 is the x-axis

Now, plotting the above points on the graph, we get the feasible region of the LPP as shaded
region ABCD. The coordinates of the corner points of the feasible region ABCD are A(20, 40),
B(50, 100), C(0, 200), D(0, 50).
Now, ZA = 20 + 2 × 40 = 100
ZB = 50 + 2 × 100 = 250,
ZC = 0 + 2 × 200 = 400
ZD = 0 + 2 × 50 = 100
Hence, Z is maximum at C(0, 200) and having value 400.

(1  sin x )
31. Evaluate:  e x dx
(1  cos x)
Ans:

Prepared by: M. S. KumarSwamy, TGT(Maths) Page - 10


-
SECTION – D
Questions 32 to 35 carry 5 marks each.

32. Find the area of the region bounded by the parabola y2 = 8x and the line x = 2.
Ans:


33. Find the shortest distance between the lines r  3i  2 j  4k   (i  2 j  2k ) and

r  5i  2 j   (3i  2 j  6k ) . If the lines intersect find their point of intersection.
Ans:

Prepared by: M. S. KumarSwamy, TGT(Maths) Page - 11


-
OR
Find the vector equation of the line passing through (1, 2, – 4) and perpendicular to the two lines:
x  8 y  19 z  10 x  15 y  29 z  5
  and  
3 16 7 3 8 5
Ans:

34. Let N be the set of all natural numbers and let R be a relation on N × N defined by (a,b)R(c,d)
such that ad=bc for all (a,b),(c,d)∈N×N. Show that R is an equivalence relation on N×N.
Ans: For reflexive: (a, b) R (a, b) such that ab = ba, which is true in N.
Hence, reflexive.
For symmetric:(a, b) R (c, d) such that ad = bc and cb =da for (c, d) R (a, b).
Hence, symmetric.
For transitive: Consider (a, b) R (c, d) and (c, d) R (e, f) such that ad = bc and cf = de then
we have ad⋅cf = bc⋅de and therefore af = be (a,b) R (e, f).
Hence, transitive. Since relation R is reflexive, symmetric and transitive. Hence, relation R is an
equivalence relation.
OR
Show that the relation S in the set R of real numbers, defined as
S = {(a, b) : a, b ∈ R and a ≤ b3} is neither reflexive, nor symmetric, nor transitive.
Ans:

Prepared by: M. S. KumarSwamy, TGT(Maths) Page - 12


-
1 1 0   2 2 4 
35. Given A =  2 3 4  and B =  4 2 4  , verify that BA = 6I, use the result to solve the
0 1 2   2 1 5 
system x – y = 3, 2x + 3y + 4z = 17, y + 2z = 7.
Ans:

SECTION – E(Case Study Based Questions)


Questions 35 to 37 carry 4 marks each.

36. Case-Study 1: Read the following passage and answer the questions given below.

The temperature of a person during an intestinal illness is given by


( ) = −0.1 ² + + 98.6,0 ≤ ≤ 12, m being a constant, where f(x) is the temperature in °F at
x days.
(i) Is the function differentiable in the interval (0, 12)? Justify your answer.
(ii) If 6 is the critical point of the function, then find the value of the constant

Prepared by: M. S. KumarSwamy, TGT(Maths) Page - 13


-
(iii) Find the intervals in which the function is strictly increasing/strictly decreasing.
OR
(iii) Find the points of local maximum/local minimum, if any, in the interval (0, 12) as well as
the points of absolute maximum/absolute minimum in the interval [0, 12]. Also, find the
corresponding local maximum/local minimum and the absolute maximum/absolute minimum
values of the function.
Ans: (i) f( ) = −0.1 ² + + 98.6, being a polynomial function, is differentiable
everywhere, hence, differentiable in (0, 12)
(ii) '( ) = − 0.2 +
Since, 6 is the critical point,
'(6) = 0 ⇒ = 1.2
(iii) ( ) = −0.1 ² + 1.2 + 98.6
'( ) = − 0.2 + 1.2 = −0.2( − 6)
In the Interval f’(x) Conclusion
(0, 6) +ve f is strictly increasing in [0, 6]
(6, 12) -ve f is strictly decreasing in [6, 12]

OR
(iii) ( ) = −0.1 ² + 1.2 + 98.6,
'( ) = − 0.2 + 1.2, '(6) = 0,
''( ) = − 0.2
''(6) = − 0.2 < 0
Hence, by second derivative test 6 is a point of local maximum. The local maximum value = (6)
= − 0.1 × 6² + 1.2 × 6 + 98.6 = 102.2
We have (0) = 98.6, (6) = 102.2, (12) = 98.6
6 is the point of absolute maximum and the absolute maximum value of the function = 102.2.
0 and 12 both are the points of absolute minimum and the absolute minimum value of the
function = 98.6.

37. Case-Study 2: Read the following passage and answer the questions given below.
An architect designs a building for a multinational company. The floor consists of a rectangular
region with semicircular ends having a perimeter of 200 m as shown here:

(i) If x and y represents the length and breadth of the rectangular region, then find the relation
between the variable.
(ii) Find the area of the rectangular region A expressed as a function of x.
(iii) Find the maximum value of area A.
OR
The CEO of the multi-national company is interested in maximizing the area of the whole floor
including the semi-circular ends. Find the value of x for this to happen.
Ans:

Prepared by: M. S. KumarSwamy, TGT(Maths) Page - 14


-
OR

38. Case-Study 3: Read the following passage and answer the questions given below.

In an office three employees Vinay, Sonia and Iqbal process incoming copies of a certain form.
Vinay process 50% of the forms. Sonia processes 20% and Iqbal the remaining 30% of the
forms. Vinay has an error rate of 0.06, Sonia has an error rate of 0.04 and Iqbal has an error rate
of 0.03.
(i) Find the conditional probability that an error is committed in processing given that Sonia
processed the form.
(ii) Find the probability that Sonia processed the form and committed an error.

Prepared by: M. S. KumarSwamy, TGT(Maths) Page - 15


-
(iii) The manager of the company wants to do a quality check. During inspection he selects a
form at random from the days output of processed forms. If the form selected at random has an
error, find the probability that the form is not processed by Vinay.
OR
If the form selected at random has an error, find the probability that the form is processed by
Sonia
Ans: Let V : Vinay processes form; S : Sonia processes form; I : Iqbal processes form; E : Error
rate
P(V) = 50% = 50/100 = 5/10 ; P(S) = 20% = 20/100 = 2/10 ; P(I) = 30% = 30/100 = 3/10;
P(E/V) = 0.06; P(E/S) = 0.04; P(E/I) = 0.03
(i) Required conditional probability = P(E/S) = 0.04
(ii) P(Sonia processed the form and committed an error) = P(S) + P(E/S) = 2/10 × 0.04 = 0.008
(iii) P(Form is processed by Vinay)

OR

Prepared by: M. S. KumarSwamy, TGT(Maths) Page - 16


-
KENDRIYA VIDYALAYA GACHIBOWLI, GPRA CAMPUS, HYD–32
SAMPLE PAPER TEST 02 FOR BOARD EXAM (2022-23)
(ANSWERS)
SUBJECT: MATHEMATICS (041) MAX. MARKS : 80
CLASS : XII DURATION: 3 HRS
General Instructions:
1. This Question paper contains - five sections A, B, C, D and E. Each section is compulsory.
However, there are internal choices in some questions.
2. Section A has 18 MCQ’s and 02 Assertion-Reason based questions of 1 mark each.
3. Section B has 5 Very Short Answer (VSA)-type questions of 2 marks each.
4. Section C has 6 Short Answer (SA)-type questions of 3 marks each.
5. Section D has 4 Long Answer (LA)-type questions of 5 marks each.
6. Section E has 3 source based/case based/passage based/integrated units of assessment (4
marks each) with sub parts.

SECTION – A
Questions 1 to 20 carry 1 mark each.
2 3 5
1. Find the cofactor of a12 in the following: 6 0 4
1 5 7
(a) -46 (b) 46 (c) 0 (d) 1
Ans: (b) 46

1 3   y 0  5 6 
2. If 2     , then write the value of x and y.
0 x   1 2  1 8 
(a) x = 3, y = 3 (b) x = 3, y = 2 (c) x = 2, y = 2 (d) x = 2, y = 3
Ans: (a) x = 3, y = 3

dy
3. If y = a 2  x 2 , then y is:
dx
(a) 0 (b) x (c) -x (d) 1
Ans: (c) -x

Prepared by: M. S. KumarSwamy, TGT(Maths) Page - 1-


 
4. Find the angle between the vectors a  i  j  k and b  i  j  k
 1  1  2
(a) cos1    (b) 60° (c) cos1    (d) cos1   
 2  3  3
 1
Ans: (c) cos1   
 3

3
log x 2
5. The value of 2 x dx is
3 3 1
(a) log 6 log   (b) log   (c) 2 log 3 (d)   log 6
2 2 3
3
Ans: (a) log 6 log  
2

6. If m and n are the order and degree, respectively of the differential equation
2
 dy  d y
2
5 x    2  6 y  log x , then write the value of m + n.
 dx  dx
(a) 1 (b) 2 (c) 3 (d) 4
Ans: (c) 3
Here, m = 2 and n = 1 then m + n = 2 + 1 = 3

7. In an LPP, the objective function is always:


(a) linear (b) quadratic (c) cubic (d) biquadratic
Ans: (a) linear
2
dx
8. The value of is x
1 x2  1
:

(a) π/3 (b) π/2 (c) π/4 (d) π/6


Ans: (a) π/3

Prepared by: M. S. KumarSwamy, TGT(Maths) Page - 2-


9. If (i  3 j  8k )  (3i   j   k ) = 0, then λ and μ are respectively:
(a) 27, -9 (b) 9, 9 (c) -9, 18 (d) -1, 1
Ans: (a) 27, -9

10. A is a skew-symmetric matrix and a matrix B such that B’AB is defined, then B’AB is a:
(a) symmetric matrix (b) skew-symmetric matrix
(c) Diagonal matrix (d) upper triangular symmetric
Ans: (b) skew-symmetric matrix
A is a skew-symmetric matrix
⇒ A’ = -A
Consider (B’AB)’ = (AB)'(B’)’ = B’A'(B’)’
= B’A’B = B'(-A)B = -B’AB
As (B’AB) = -B’AB
Hence, B’AB is a skew-symmetric matrix.

11. For an L.P.P. the objective function is Z = 4x + 3y, and the feasible region determined by a set of
constraints (linear inequations) is shown in the graph.

Which one of the following statements is true?


(a) Maximum value of Z is at R.
(b) Maximum value of Z is at Q.
(c) Value of Z at R is less than the value at P.
(d) Value of Z at Q is less than the value at R.
Ans: (b) Maximum value of Z is at Q.

Prepared by: M. S. KumarSwamy, TGT(Maths) Page - 3-


Z = 4x + 3y
at P (0, 40), Z = 4(0) + 3(40) = 120
at Q (30, 20), Z = 4(30) + 3(20) = 180
at R (40, 0), Z = 4(40) + 3(0) = 160
∴ Zmax = 180 at Q (30, 20)

2
4 1 3 2 x 3
12. If   , then the value of x is:
2 1 1 x 2 1
(a) 6 (b) 3 (c) 7 (d) 1
Ans: (a) 6
2
4 1 3 2 x 3
  ⇒ (4 – 2)² = (3x – 2) – (x + 6)
2 1 1 x 2 1
⇒ 4 = 3x – 2 – x – 6 ⇒ 2x = 12 ⇒ x = 6

13. If A and B are square matrices of order 3 such that |A| = 1 and |B| = 3, then the value of |3AB| is:
(a) 3 (b) 9 (c) 27 (d) 81
Ans: (d) 81
As AB is of order 3 and
|3AB| = 3³|AB|
= 27|A||B| = 27 × 1 × 3 = 81

14. If P(A) = 0.4, P(B) = 0.8 and P(B/A) = 0.6 then P(A U B) is equal to
(a) 0.24 (b) 0.3 (c) 0.48 (d) 0.96
Ans: (d) 0.96

15. Which of the following is a homogeneous differential equation?


(a) (4x + 6y + 5)dy − (3y + 2x + 4)dx = 0
(b) (xy)dx − (x3 + y3)dy = 0
(c) (x3 + 2y2)dx + 2xy dy = 0
(d) y2dx + (x2 − xy − y2)dy = 0
Ans: (d) y2dx + (x2 − xy − y2)dy = 0

1  sin x
16. If y = tan 1 , then value of dy/dx at x = π/6 is:
1  sin x
(a) 1/2 (b) −1/2 (c) 1 (d) -1
Ans: (b) −1/2

x  3 y  2 z 1
17. The straight line   is:
3 1 0
(a) parallel to x-axis (b) parallel to y-axis
(c) parallel to z-axis (d) perpendicular to z-axis
Ans: (c) parallel to z-axis

Prepared by: M. S. KumarSwamy, TGT(Maths) Page - 4-


18. The points (1, 2, 3), (4, 0, 4), (– 2, 4, 2), (7, – 2, 5) are:
(a) collinear (b) are the vertices of a square
(c) are the vertices of a rectangle (d) None of these
Ans: (a) collinear

ASSERTION-REASON BASED QUESTIONS


In the following questions, a statement of assertion (A) is followed by a statement of Reason (R).
Choose the correct answer out of the following choices.
(a) Both A and R are true and R is the correct explanation of A.
(b) Both A and R are true but R is not the correct explanation of A.
(c) A is true but R is false.
(d) A is false but R is true.

x 1 y  2 z  3
19. Assertion (A) : The angle between the straight lines   and
2 5 4
x 1 y  2 z  3
  is 90°.
1 2 3
Reason (R) : Skew lines are lines in different planes which are parallel and intersecting.
Ans: (c) A is true but R is false.
Assertion is correct.
x 1 y  2 z  3 x 1 y  2 z  3
Give that   and  
2 5 4 1 2 3

20. Assertion (A): Domain of f(x) = sin–1x + cosx is [-1, 1]


Reason (R): Domain of a function is the set of all possible values for which function will be
defined.
Ans: (a) Both A and R are true and R is the correct explanation of A.

SECTION – B
Questions 21 to 25 carry 2 marks each.

21. Show that the modulus function f : R → R given by f (x) = |x|, is neither one-one nor onto, where
|x| is x, if x is positive or 0 and |x| is –x, if x is negative.
 x, if x  0
Ans: f ( x) | x | 
 x, if x  0
One-one: Let x1 = 1, x2 = –1 be two elements belongs to R
f(x1) = f(1) = |1| and f(x2) = f(–1) = – (–1) = 1
⇒ f(x1) = f(x2) for x1 ≠ x2
⇒ f(x) is not one-one.
Onto: Let f(x) = –1 ⇒ |x| = –1 ∈ R, which is not possible.

Prepared by: M. S. KumarSwamy, TGT(Maths) Page - 5-


⇒ f(x) is not onto.
Hence, f is neither one-one nor onto function.
OR
1 2x 1 1  y 
2
Find the value of the tan  sin 1  cos  , |x| < 1, y > 0 and xy < 1.
2 1  x2 1 y2 
Ans:

 sin 5 x
  cos x, if x  0
22. For what value of ‘k’ is the function f ( x)   3 x continuous at x = 0?
 k , if x  0
Ans:

23. Show that the line through the points (1, –1, 2), (3, 4, –2) is perpendicular to the line through the
points (0, 3, 2) and (3, 5, 6).
Ans: Let A (1, –1, 2) and B (3, 4, – 2) be given points.
Direction ratios of AB are
(3 – 1), {(4 – (–1)}, (–2 –2) i.e., 2, 5, – 4.
Let C (0, 3, 2) and D (3, 5, 6) be given points.
Direction ratios of CD are
(3 – 0), (5 – 3), (6 – 2) i.e., 3, 2, 4.
We know that two lines with direction ratios a1, b1, c1 and a2, b2, c2 are perpendicular if
a1a2 + b1b2 + c1c2 = 0.
∴ 2 × 3 + 5 × 2 + (– 4) × 4 = 6 +10 –16 = 0, which is true.
It will shows that lines AB and CD are perpendicular.
       
24. Given, p  3i  2 j  4k , a  i  j, b  j  k , c  i  k and p  xa  yb  zc , then find the value of x,
y, z.
Ans:

Prepared by: M. S. KumarSwamy, TGT(Maths) Page - 6-


OR
Using vectors, find the area of the triangle with vertices A(1, 1, 2), B(2, 3, 5) and C(1, 5, 5).
Ans:

dy
25. If y = log(cos ex), then find .
dx
Ans:

SECTION – C
Questions 13 to 22 carry 3 marks each.
2
dx
26. Find the value of  x(1  log x)
1
2
.

Ans:

OR

x tan x
Evaluate:  sec x.cos ecx dx
0

Ans:

Prepared by: M. S. KumarSwamy, TGT(Maths) Page - 7-


( x 2  3x )
27. Evaluate:  dx
( x  1)( x  2)
Ans:

28. Solve : (x2 – yx2)dy + (y2 + x2y2)dx = 0


Ans:

OR

Solve : (x2 + y2) dx – 2xydy = 0


Ans:

Prepared by: M. S. KumarSwamy, TGT(Maths) Page - 8-


29. Solve the following Linear Programming Problem graphically:
Minimise Z = 13x – 15y subject to the constraints x + y ≤ 7, 2x – 3y + 6 ≥ 0, x ≥ 0 and y ≥ 0.
Ans: Minimise Z = 13x – 15y ...(i)
Subject to the constraints
x + y ≤ 7 ...(ii)
2x – 3y + 6≥ 0 ...(iii)
x ≥ 0, y ≥ 0 ...(iv)
Shaded region shown as OABC is bounded and coordinates of its corner points are (0, 0), (7, 0),
(3, 4) and (0, 2) respectively.

Prepared by: M. S. KumarSwamy, TGT(Maths) Page - 9-


Hence, the minimum value of Z is –30 at (0, 2).

30. In a hostel, 60% of the students read Hindi newspaper, 40% read English newspaper and 20%
read both Hindi and English newspaper. A student is selected at random.
(a) Find the probability that the student reads neither Hindi nor English newspaper.
(b) If she reads Hindi newspaper, find the probability that she reads English newspaper.
(c) If she reads English newspaper, find the probability that she reads Hindi newspaper.
Ans: Let A be the event that a student reads Hindi newspaper and B be the event that a student
reads English newspaper.
P(A) = 60/100 = 0.6, P(B)= 40/100 = 0.4 and P(A ∩ B) = 20/100 = 0.2
(a) Now P(A ∪ B) = P(A) + P(B) – P(A ∩ B)
= 0.6 + 0.4 – 0.2 = 0.8
Probability that she reads neither Hindi nor English newspaper
= 1 – P(A ∪ B) = 1– 0.8 = 0.2 = 1/5
P( A  B) 0.2 1
(b) P( B / A)   
P( A) 0.6 3
P( A  B) 0.2 1
(c) P( A / B)   
P( B) 0.4 2
OR
The random variable X has a probability distribution P(X) of the following form, where k is
some number:
 k , if x  0
 2k , if x  1

P( X )  
 3k , if x  2
0, if otherwise
(a) Determine the value of k.
(b) Find P(X < 2), P(X ≤ 2), P(X ≥ 2).
Ans: (a) k + 2k + 3k =1 [∵ p1 + p2 + p3 + ... + pn =1]
⇒ 6k = 1 ⇒ k = 1/6
1 1
(b) P (X < 2) = k + 2k = 3k = 3  
6 2
1
P (X ≤ 2) = 3k + 2k + k = 6k = 6   1
6
1 1
P (X ≥ 2) = 3k = 3  
6 2

x2  1
31. Evaluate:  2 dx
( x  2)( x 2  3)
Ans:

Prepared by: M. S. KumarSwamy, TGT(Maths) Page - 10


-
SECTION – D
Questions 32 to 35 carry 5 marks each.

32. Using integration, find the area of triangle ABC, whose vertices are A(2, 5), B(4, 7) and C(6, 2).
Ans:

33. Find the vector equation of the line through the point (1, 2, –4) and perpendicular to the two lines
 
r  (8i  19 j  10k )   (3i  16 j  7 k ) and r  (15i  29 j  5k )   (3i  8 j  5k )
Ans:

Prepared by: M. S. KumarSwamy, TGT(Maths) Page - 11


-
OR
Find the shortest distance between the following lines :

r  (i  j  k )  s (2i  j  k )

r  (i  j  2k )  t (4i  2 j  2k )
Ans:

34. Show that each of the relation R in the set A = {x ∈ Z : 0 ≤ x ≤ 12}, given by R = {(a, b) :|a – b|
is a multiple of 4} is an equivalence relation.
Find the set of all elements related to 1 in each case.
Ans: A = {x ∈ Z : 0 ≤ x ≤ 12}
R = {(a, b) : | a – b | is a multiple of 4}
Reflexive: Let x ∈A ⇒ |x – x| = 0, which is a multiple of 4.
⇒ (x, x) ∈ R ∀ x ∈A
∴ R is reflexive.
Symmetric: Let x, y ∈A and (x, y) ∈ R
⇒ |x – y| is a multiple of 4
or x – y = ± 4p {p is any integer}
⇒ y – x =  4p
⇒ | y – x | is a multiple of 4. ⇒ (y, x) ∈ R
⇒ R is symmetric.
Transitive: Let x, y, z ∈ A, (x, y) ∈ R and (y, z) ∈ R
⇒ |x - y| is multiple of 4 and | y – z |is multiple of 4
⇒ x – y is multiple of 4 and y – z is multiple of 4
⇒ (x – y) + (y – z) is multiple of 4 ⇒ (x – z) is multiple of 4.

Prepared by: M. S. KumarSwamy, TGT(Maths) Page - 12


-
⇒ |x – z| is multiple of 4.
⇒ (x, z) ∈ R ⇒ R is transitive.
So, R is an equivalence relation.
Let B be the set of elements related to 1.
∴ B = {a ∈ A : |a – 1| is multiple of 4}
⇒ B = {1, 5, 9} {as |1 – 1| = 0, |1 – 5 |= 4, |1 – 9| = 8}
OR
x 1
Let A = R – {2} and B = R – {1}. If f : A → B is a function defined by f ( x )  , then show
x2
that f is one-one and onto.
Ans:

 2 3 5 
35. If A =  3 2 4  , find A–1 and hence solve the system of linear equations: 2x – 3y + 5z = 11,
 1 1 2 
3x + 2y – 4z = –5; x + y – 2z = –3.
Ans:

Prepared by: M. S. KumarSwamy, TGT(Maths) Page - 13


-
SECTION – E(Case Study Based Questions)
Questions 35 to 37 carry 4 marks each.

36. Case Study 1 : On one day, Maths teacher is conducted Mental Ability test. Anand, Sanjay and
Aditya are trying to solve a given Mental ability problem in Mathematics whose respective
1 1 1
probabilities of solving it are , and . They were asked to solve it independently.
2 3 4

Based on the above data, answer any four of the following questions.
(i) Find the probability that Anand alone solves it.
(ii) Find the probability that exactly one of them solves it.
Ans: (i) Let A → event that Anand solves
B → event that Sanjay solves
C → event that Aditya solves

Prepared by: M. S. KumarSwamy, TGT(Maths) Page - 14


-
37. Case-Study 2: Read the following passage and answer the questions given below.
In an elliptical sport field the authority wants to design a rectangular soccer field with the
x2 y2
maximum possible area. The sport field is given by the graph of 2  2  1
a b

(i) If the length and the breadth of the rectangular field be 2x and 2y respectively, then find the
area function in terms of x.
(ii) Find the critical point of the function.
(iii) Use First derivative Test to find the length 2x and width 2y of the soccer field (in terms of a
and b) that maximize its area.
OR
(iii) Use Second Derivative Test to find the length 2x and width 2y of the soccer field (in terms
of a and b) that maximize its area.
Ans:

Prepared by: M. S. KumarSwamy, TGT(Maths) Page - 15


-
38. Case-Study 3: An owner of a car rental company have determined that if they charge customers
Rs x per day to rent a car, where 50 ≤ x ≤ 200, then number of cars (n), they rent per day can be
shown by linear function n(x) = 1000 – 5x. If they charge Rs. 50 per day or less they will rent all
their cars. If they charge Rs. 200 or more per day they will not rent any car.

Prepared by: M. S. KumarSwamy, TGT(Maths) Page - 16


-
Based on the above information, answer the following question.
(i) If R(x) denote the revenue, then find the value of x at which R(x) has maximum value.
(ii) Find the Maximum revenue collected by company
OR
Find the number of cars rented per day, if x = 75.
Ans: (i) Let x be the price charge per car per day and n be the number of cars rented per day.
R(x) = n × x = (1000 – 5x) x = –5x2 + 1000x
⇒ R′(x) = 1000 – 10x
For R(x) to be maximum or minimum, R′(x) = 0
⇒ –10x + 1000 = 0 ⇒ x = 100
Also, R″(x) = –10 < 0
Thus, R(x) is maximum at x = 100
(ii) At x = 100, R(x) is maximum.
Maximum revenue = R(100) = –5(100)2 + 1000(100) = Rs. 50,000
OR
If x = 75, number of cars rented per day is given by
n = 1000 – 5 × 75 = 625

Prepared by: M. S. KumarSwamy, TGT(Maths) Page - 17


-
KENDRIYA VIDYALAYA GACHIBOWLI, GPRA CAMPUS, HYD–32
SAMPLE PAPER TEST 03 FOR BOARD EXAM (2022-23)
(ANSWERS)
SUBJECT: MATHEMATICS (041) MAX. MARKS : 80
CLASS : XII DURATION: 3 HRS
General Instructions:
1. This Question paper contains - five sections A, B, C, D and E. Each section is compulsory.
However, there are internal choices in some questions.
2. Section A has 18 MCQ’s and 02 Assertion-Reason based questions of 1 mark each.
3. Section B has 5 Very Short Answer (VSA)-type questions of 2 marks each.
4. Section C has 6 Short Answer (SA)-type questions of 3 marks each.
5. Section D has 4 Long Answer (LA)-type questions of 5 marks each.
6. Section E has 3 source based/case based/passage based/integrated units of assessment (4
marks each) with sub parts.

SECTION – A
Questions 1 to 20 carry 1 mark each.

 0 2b 2 
1. The matrix A =  3 1 3  is a symmetric matrix. Then the value of a and b respectively are:
3a 3 1
2 3 1 1 3 1
(a) , (b) , (c) -2, 2 (d) ,
3 2 2 2 2 2
2 3
Ans: (a) ,
3 2

7 6 x
2. If one root of the equation 2 x 2 = 7 is x = -9, then the other two roots are:
x 3 7
(a) 6, 3 (b) 6, -3 (c) -2, -7 (d) 2, 6
Ans: (c) -2, -7
7 6 x
2 x 2 = 7(7x – 6) – 6(14 – 2x) + x(6 – x2)
x 3 7
= -x3 + 67x – 126
= (x + 9)(-x2 + 9x – 14)

Prepared by: M. S. KumarSwamy, TGT(Maths) Page - 1-


= (x + 9)(-x – 2) (x + 7)
Hence the other two roots are -2 and -7.

3. If f (x) = x tan–1 x, then f ′ (1) =


 1  1 
(a) 1  (b)  (c)  (d) 2
4 2 4 2 4
1 
Ans: (b) 
2 4

 
4. The value of λ such that the vector a  2iˆ   ˆj  k and b  iˆ  2 ˆj  3k are orthogonal is:
(a) 3/2 (b) −5/2 (c) −1/2 (d) 1/2
Ans: (b) −5/2
 
Since, two non-zero vectors a and b are orthogonal

5. The area (in sq. m) of the shaded region as shown in the figure is:

(a) 32/3 sq. units (b) 16/3 sq. units (c) 4 sq. units (d) 16 sq. units
Ans: (a) 32/3 sq. units
Given curves are x = y2 and x = 4.
So, their points of intersection are (4, 2) and (4, -2).

d2y dy
6. The order and the degree of the differential equation 2 x 2 2
 3  y  0 are:
dx dx
(a) 1, 1 (b) 2, 1 (c) 1, 2 (d) 3, 1
Ans: (b) 2, 1
The highest order is 2 and the degree of the highest order is 1.
Hence, the order is 2 and the degree is 1.

Prepared by: M. S. KumarSwamy, TGT(Maths) Page - 2-


7. A set of values of decision variables that satisfies the linear constraints and non-negativity
conditions of an L.P.P. is called its:
(a) Unbounded solution (b) Optimum solution
(c) Feasible solution (d) None of these
Ans: (c) Feasible solution

a
a
8. The value of 
0 x  ax
dx is:

(a) a/2 (b) a (c) a2 (d) 0


Ans: (a) a/2

   
9. For any vector a , the value of | a  iˆ |2  | a  ˆj |2  | a  k |2 is:
(a) a (b) a2 (c) 1 (d) 0
2
Ans: (b) a

10. For an L.P.P. the objective function is Z = 400x + 300y, and the feasible region determined by a
set of constraints (linear inequations) is shown in the graph.

Find the coordinates at which the objective function is maximum.


(a) (20, 0) (b) (40, 0) (c) (40, 160) (d) (20, 180)
Ans: (c) (40, 160)
Value of z at each corner point
z at (20, 0), z = 400  20 + 300  0 = 8000
z at (40, 0) = 400  40 + 300  0 = 16000
z at (40, 160) = 400  40 + 300  160 = 16000 + 48000 = 64000
z at (20, 180) = 400  20 + 300  180 = 8000 + 54000 = 62000
max z = 64000 for x = 40, y = 160

11. A function f : Z → Z given by f (x) = 5x + 3 is

Prepared by: M. S. KumarSwamy, TGT(Maths) Page - 3-


(a) one-one but not onto. (b) bijective
(c) onto but not one-one (d) None of these
Ans: (a) one-one but not onto.
The given function: f : Z → Z given by f (x) = 5x + 3.
(i) One-one: Take two integers x1 and x2, such that f (x1) = f (x2) ⇒ 5x1 + 3 = 5x2 + 3
⇒ 5x1 = 5x2 ⇒ x1 = x2
⇒ given function is one-one.
(ii) Onto: Take y = 5x + 3 ⇒ x = (y − 3)/5
For every integer y, there does not exist an integer x. [∵ (y − 3)/5 is not an integer for all y]

1 0 4 
12. The cofactor of (-1) in the matrix  3 5 1 is:
 0 1 2 
(a) 1 (b) 2 (c) -1 (d) 0
Ans: (c) -1
1 0
Cofactor of (-1) = (1) 23 = (-1) × 1 = -1
0 1

13. If A and B are two events such that P(A) = 1/2, P(B) = 1/3 and P(A/B) = 1/4, then P(A' ∩ B')
equals
(a) 1/12 (b) 3/4 (c) 1/4 (d) 3/16
Ans: (c) 1/4

14. Let A be a non-singular matrix of order (3 × 3). Then |adj.A| is equal to


(a) |A| (b) |A|2 (c) |A|3 (d) 3|A|
2
Ans: (b) |A|
If A is a matrix of order n × n then |adj A| = |A|n – 1

dy
15. The general solution of the differential equation  2  y is:
dx
(a) 2y = x log 2 + C log 2 (b) 2y = x log 3 – C log 3
(c) y = x log 2 – C log 2 (d) None of these
Ans: (a) 2y = x log 2 + C log 2

16. The domain, for which tan-1x > cot-1x holds true, is:
(a) x = 1 (b) x > 1 (c) x < 1 (d) Not defined

Prepared by: M. S. KumarSwamy, TGT(Maths) Page - 4-


Ans: (b) x > 1
The graphs of tan-1x and cot-1x indicate that tan-1x > cot-1x for x > 1.

x 1 y  3 1 z
17. A point that lies on the line   is:
2 4 7
(a) (1, -3, 1) (b) (-2, 4, 7) (c) (-1, 3, 1) (d) (2, -4, -7)
Ans: (a) (1, -3, 1)
x  1 y  3 z 1
The equation of the Line can be written as  
2 4 7
So, it passes through (1, -3, 1).

18. The direction ratios of the line 6x – 2 = 3y + 1 = 2z – 2 are:


(a) 6, 3, 2 (b) 1, 1, 2 (c) 1, 2, 3 (d) 1, 3, 2
Ans: (c) 1, 2, 3
Given the equation of a line is
6x – 2 = 3y + 1 = 2z – 2
1 1
x y
 1  1
 6  x    6  y    2( z  1)  3 3  z 1
 3  3 1 2 3
This shows that the given line passes through (1/3, -1/3, 1), and has direction ratios 1, 2, and 3.

ASSERTION-REASON BASED QUESTIONS


In the following questions, a statement of assertion (A) is followed by a statement of Reason (R).
Choose the correct answer out of the following choices.
(a) Both A and R are true and R is the correct explanation of A.
(b) Both A and R are true but R is not the correct explanation of A.
(c) A is true but R is false.
(d) A is false but R is true.

x  2 y 1 z  z x  3 y z 1
19. Assertion (A): Lines   and   are coplanar.
2 3 1 3 2 2
Reason (R): Let line l1 passes through the point (x1, y1, z1) and parallel to the vector whose
direction ratios are a1, b1 and c1; and let line l2 passes through the point (x2, y2, z2) and parallel
to the vector whose direction ratios are a2, b2 and c2.
x 2  x1 y2  y1 z2  z1
Then both lines l1 and l2 are coplanar if and only if a1 b1 c1 =0
a2 b2 c2
Ans: (a) Both A and R are true and R is the correct explanation of A.

20. Assertion (A): sin-1(sin(2π/3)) = 2π/3


Reason (R): sin-1(sin θ) = θ, if θ ∈ [(-π)/2, π/2]
Ans: (d) A is false but R is true.

The principal value branch of sin–1x is [(-π)/2, π/2]


Let x = sin θ ⇒ θ = sin–1x
sin–1(sin θ) = sin–1 x = θ
sin–1(sin θ) = θ, if θ ∈ [(-π)/2, π/2]
Hence R is true.

Hence A is false.

Prepared by: M. S. KumarSwamy, TGT(Maths) Page - 5-


SECTION – B
Questions 21 to 25 carry 2 marks each.

21. Check whether the relation R in the set Z of integers defined as R = {(a, b) : a + b is “divisible by
2”} is reflexive, symmetric or transitive. Write the equivalence class containing 0 i.e., [0]. 3
Ans: Reflexive: (a, a) : a + a = 2a which is even
∴ divisible by 2
∴ (a, a) ∈ R ∀ a ∈ Z. Hence R is reflexive.
Symmetric: If (a, b) ∈ R, then a + b is “divisible by 2”
Let a + b = 2m ⇒ b + a = 2m ...[∵ b + a = a + b
⇒ (b, a) ∈ R ∀ a, b ∈ z
Hence R is symmetric.
Transitive: If (a, b) ∈ R and (b, c) ∈ R
Let a + b = 2m ...(i)
b + c = 2n ...(ii)
Adding (i) and (ii), we have a + b + b + c = 2m + 2n
a + 2b + c = 2m + 2n ⇒ a + c = 2m + 2n – 2b ⇒ a + c = 2(m + n – b)
⇒ a + c = 2k ...where[k = m + n – b ⇒ (a, c) ∈ R
Hence R is transitive. Equivalence class containing 0 i.e. [0] = {......–4, –2, 0, 2, 4, ......}
OR
 cos x   3 
Express tan 1  ,  x  in simplest form.
 1  sin x  2 2
Ans:

22. Show that the function f(x) = x3 – 3x2 + 6x – 100 is increasing on R.


Ans: We have, f(x) = x3 – 3x2 + 6x – 100 ...(i)
Differentiating (i) w.r.t. x, we get
f ′(x) = 3x2 – 6x + 6 = 3(x2 – 2x + 1) + 3
= 3(x – 1)2 + 3 > 0
For all values of x, (x – 1)2 is always positive
∴ f ′ (x) > 0
So, f(x) is an increasing function on R.

23. Discuss the continuity of the following function at x = 0 :


 x 4  2 x 3  x 2
 , if x  0
f ( x)   tan 1 x
 0, if x  0

Ans:

Prepared by: M. S. KumarSwamy, TGT(Maths) Page - 6-


and f(0) = 0 (given)
so, L.H.L = R.H.L = f(0)
Hence given function is continuous at x = 0

24. Find the vector equation of the line joining (1, 2, 3) and (–3, 4, 3) and show that it is
perpendicular to the z-axis.
Ans: Vector equation of the line passing through (1, 2, 3) and (–3, 4, 3) is

  
25. Prove that the points A, B and C with position vectors a , b and c respectively are collinear if
      
and only if a  b  b  c  c  b  0
Ans:

OR
   
     
If a  7i  j  4k and b  2i  6 j  3k , then find the projection of b on a .
 
Ans: Given a  7i  j  4k and b  2i  6 j  3k

Prepared by: M. S. KumarSwamy, TGT(Maths) Page - 7-


SECTION – C
Questions 13 to 22 carry 3 marks each.
x2  x
4
26. Find the value of 
2 2x 1
dx .

Ans:

OR
2
dx
Find the value of  x(1  log x)
1
2
.

Ans:

3x  1
27. Evaluate:  ( x  1) ( x  3) dx
2

Ans:

Prepared by: M. S. KumarSwamy, TGT(Maths) Page - 8-


dy
28. Solve the following differential equation:  x 3 cos ecy , given that y(0) = 0.
dx
Ans:

OR

  y 
Solve the following differential equation:  x sin 2    y  dx  xdy  0
 x 
  y 
Ans: We have  x sin 2    y  dx  xdy  0
 x 

Prepared by: M. S. KumarSwamy, TGT(Maths) Page - 9-


29. Solve the following Linear Programming Problem graphically:
Maximise z = 8x + 9y subject to the constraints: 2x + 3y ≤ 6, 3x – 2y ≤ 6, y ≤ 1; x, y ≥ 0
Ans:
Let l1 : 2x + 3y = 6, l2 : 3x – 2y = 6, l3 : y = 1; x = 0, y = 0
Solving l1 and l3, we get D (1.5, 1)
 30 6 
Solving l1 and l2, we get C  , 
 13 13 

Shaded portion OADCB is the feasible region,


where coordinates of the corner points are O(0, 0),
 30 6 
A(0, 1), D(1.5, 1), C  ,  , B(2, 0).
 13 13 
The value of the objective function at these points are:
Corner points Value of the objective function z = 8x + 9y
O (0, 0) 8×0+9×0=0
A (0, 1) 8×0+9×1=9
D (1.5, 1) 8 × 1.5 + 9 × 1 = 21
 30 6  30 6
8   9  = 22.6 (Maximum)
C , 
 13 13  13 13
B (2, 0) 8 × 2 + 9 × 0 = 16
 30 6 
The maximum value of z is 22.6, which is at C  , 
 13 13 

30. Two numbers are selected at random (without replacement) from first 7 natural numbers. If X
denotes the smaller of the two numbers obtained, find the probability distribution of X.
Ans: First 7 natural numbers are 1, 2, 3, 4, 5, 6, 7.
(1, 2) (1, 3) (1, 4) (1, 5) (1, 6) (1, 7) 
(2, 1) (2, 3) (2, 4) (2, 5) (2, 6) (2, 7) 
 
(3, 1) (3, 2) (3, 4) (3, 5) (3, 6) (3, 7) 
 
S = (4, 1) (4, 2) (4, 3) (4, 5) (4, 6) (4, 7)  i.e. 42 ways
(5, 1) (5, 2) (5, 3) (5, 4) (5, 6) (5, 7) 
 
(6, 1) (6, 2) (6, 3) (6, 4) (6, 5) (6, 7) 
 
(7, 1) (7, 2) (7, 3) (7, 4) (7, 5) (7, 6) 
12 2 10 5 8 4
P( X  1)   , P ( X  2)   , P ( X  3)  
42 7 42 21 42 21

Prepared by: M. S. KumarSwamy, TGT(Maths) Page - 10


-
6 1 4 2 2 1
P( X  4)   , P( X  5)   , P( X  6)  
42 7 42 21 42 21
∴ Probability distribution is
X 1 2 3 4 5 6
2 5 4 1 2 1
P(X)
7 21 21 7 21 21

OR
There are three coins, one is a two headed coin (having head on both the faces), another is a
biased coin that comes up heads 75% of the time and the third is an unbiased coin. One of the
three coins is chosen at random and tossed. If it shows head. What is probability that it was the
two headed coin?
Ans: Let E1 : Two headed coin is chosen
E2 : Coin chosen is biased
E3 : Coin chosen is unbiased
A : Coin shows head
1 1 1
P( E1 )  , P( E2 )  , P( E3 ) 
3 3 3
75 3 1
P( A / E1 )  1, P( A / E2 )   , P ( A / E3 ) 
100 4 2
Using Baye’s theorem,
P( E1 )  P( A∣E1 )
P( E1 ∣ A) 
P( E1 )  P( A∣ E1 )  P( E2 )  ( A∣ E2 )  P( E3 )  ( A∣ E3 )
1 1 1
1
3 3 3 1 12 4
     
1  1 3 1 1 1 3 1 4  3  2 3 9 9
 1         3  12  6
3  3 4 3 2 12

 /2
dx
31. Evaluate:  1
0 tan x
Ans:

Prepared by: M. S. KumarSwamy, TGT(Maths) Page - 11


-
SECTION – D
Questions 32 to 35 carry 5 marks each.

32. Using integration, find the smaller area enclosed by the circle x2 + y2 = 4 and the line x + y = 2.
Ans:

33. Find the vector equation of the line through the point (1, 2, –4) and perpendicular to the two lines
 
r  (8i  19 j  10k )   (3i  16 j  7 k ) and r  (15i  29 j  5k )   (3i  8 j  5k )
Ans:

OR
Find the shortest distance between the following lines :

l1 : r  (i  2 j  4k )   (2i  3 j  6k )

l : r  (3i  3 j  5k )   (4i  6 j  12k )
2
Ans:

Prepared by: M. S. KumarSwamy, TGT(Maths) Page - 12


-
34. Define the relation R in the set × as follows:
For (a, b), (c, d) ∈ × , (a, b) R (c, d) iff ad = bc. Prove that R is an equivalence relation in
× .
Ans: Let ( , ) ∈ × . Then we have
ab = ba (by commutative property of multiplication of natural numbers)
⟹( , ) ( , )
Hence, R is reflexive.
Let ( , ), ( , ) ∈ × such that (a, b) R (c, d). Then ad = bc
⟹ = (by commutative property of multiplication of natural numbers
⟹( , ) ( , )
Hence, R is symmetric.
Let ( , ), ( , ), ( , ) ∈ × such that (a, b) R (c, d) and (c, d) R (e, f).
Then ad = bc, cf = de
⟹ =
⟹ =
⟹( , ) ( , )
Hence, R is transitive.
Since, R is reflexive, symmetric and transitive, R is an equivalence relation on × .

OR

Given a non-empty set X, define the relation R in P(X) as follows:


For A, B ∈ ( ), ( , ) ∈ iff ⊂ . Prove that R is reflexive, transitive and not symmetric.
Ans: Let ∈ ( ). Then ⊂
⟹( , )∈
Hence, R is reflexive.
Let , , ∈ ( ) such that ( , ), ( , ) ∈
⟹ ⊂ , ⊂

⟹ ⊂
⟹( , )∈
Hence, R is transitive.
∅, ∈ ( ) such that ∅ ⊂ . Hence, (∅, ) ∈ . But, ⊄ ∅, which implies that ( , ∅) ∉ .
Thus, R is not symmetric.

 1 2 3
35. If A =  3 2 2  , then find A–1 and use it to solve the following system of the equations :
 2 1 1 
x + 2y – 3z = 6, 3x + 2y – 2z = 3, 2x – y + z = 2
Ans:

Prepared by: M. S. KumarSwamy, TGT(Maths) Page - 13


-
SECTION – E(Case Study Based Questions)
Questions 35 to 37 carry 4 marks each.

36. Case-Study 1: Read the following passage and answer the questions given below.

Prepared by: M. S. KumarSwamy, TGT(Maths) Page - 14


-
There are two antiaircraft guns, named as A and B. The probabilities that the shell fired from
them hits an airplane are 0.3 and 0.2 respectively. Both of them fired one shell at an airplane at
the same time.

(i) What is the probability that the shell fired from exactly one of them hit the plane?
(ii) If it is known that the shell fired from exactly one of them hit the plane, then what is the
probability that it was fired from B?
Ans:
(i)P(Shell fired from exactly one of them hits the plane)
= P[(Shell from A hits the plane and Shell from B does not hit the plane) or (Shell from A does
not hit the plane and Shell from B hits the plane)]
= 0.3 × 0.8 + 0.7 × 0.2 = 0.38
(ii)P(Shell fired from B hit the plane/Exactly one of them hit the plane)
P(Shell fired from B hit the plane  Exactly one of them hit the plane)
=
P(Exactly one of them hit the plane)
P(Shell from only B hit the plane) 0.14 7
= = 
P(Exactly one of them hit the plane) 0.38 19

37. Case-Study 2: Read the following passage and answer the questions given below.

The temperature of a person during an intestinal illness is given by


1 493
( )=− ²+ + , 0 ≤ ≤ 12, m being a constant, where f(x) is the temperature in °F at
10 50
x days.
(i) Is the function differentiable in the interval (0, 12)? Justify your answer.
(ii) If 6 is the critical point of the function, then find the value of the constant

(iii) Find the intervals in which the function is strictly increasing/strictly decreasing.
OR
(iii) Find the points of local maximum/local minimum, if any, in the interval (0, 12) as well as
the points of absolute maximum/absolute minimum in the interval [0, 12]. Also, find the
corresponding local maximum/local minimum and the absolute maximum/absolute minimum
values of the function.
Ans: (i) f( ) = −0.1 ² + + 98.6, being a polynomial function, is differentiable
everywhere, hence, differentiable in (0, 12)
(ii) '( ) = − 0.2 +
Since, 6 is the critical point,
'(6) = 0 ⇒ = 1.2
(iii) ( ) = −0.1 ² + 1.2 + 98.6
'( ) = − 0.2 + 1.2 = −0.2( − 6)

Prepared by: M. S. KumarSwamy, TGT(Maths) Page - 15


-
In the Interval f’(x) Conclusion
(0, 6) +ve f is strictly increasing in [0, 6]
(6, 12) -ve f is strictly decreasing in [6, 12]

OR
(iii) ( ) = −0.1 ² + 1.2 + 98.6,
'( ) = − 0.2 + 1.2, '(6) = 0,
''( ) = − 0.2
''(6) = − 0.2 < 0
Hence, by second derivative test 6 is a point of local maximum. The local maximum value = (6)
= − 0.1 × 6² + 1.2 × 6 + 98.6 = 102.2
We have (0) = 98.6, (6) = 102.2, (12) = 98.6
6 is the point of absolute maximum and the absolute maximum value of the function = 102.2.
0 and 12 both are the points of absolute minimum and the absolute minimum value of the
function = 98.6.

38. Case-Study 3:
Sonam wants to prepare a sweet box for Diwali at home. For making lower part of box, she takes
a square piece of cardboard of side 18 cm.
Now, x cm be the length of each side of the square cardboard which is to be cut off from corner
of the square piece of side 18 cm.

Based on the above information, answer the following questions :


(i) Express Volume of the open box formed by folding up the cutting corner in terms of x and
dV
find the value of x for which = 0.
dx
(ii) Sonam is interested in maximising the volume of the box. So, what should be the side of the
square to be cut off so that the volume of the box is maximum?
Ans: (i) height of open box = x cm
Length of open box = 18 – 2x
and width of open box = 18 – 2x
∴ Volume (V) of the open box = x × (18 – 2x) × (18 – 2x)
⇒ V = x(18 – 2x)2
dV
⇒ = x ⋅ 2(18 – 2x)(–2) + (18 – 2x)2
dx
= (18 – 2x)(–4x + 18 – 2x)
= (18 – 2x)(18 – 6x)
dV
Now, = 0 ⇒ 18 – 2x = 0 or 18 – 6x = 0
dx
⇒ x = 9 or 3
Prepared by: M. S. KumarSwamy, TGT(Maths) Page - 16
-
dV
(ii) We have, V = x(18 – 2x)2 and = (18 – 2x)(18 – 6x)
dx
d 2V
⇒ = (18 – 2x)(–6) + (18 – 6x)(–2)
dx 2
= (–2)[54 – 6x + 18 – 6x]
= (–2)[72 – 12x] = 24x – 144
d 2V
For x = 3, <0
dx 2
d 2V
and for x = 9, >0
dx 2
So, volume will be maximum when x = 3.

Prepared by: M. S. KumarSwamy, TGT(Maths) Page - 17


-
KENDRIYA VIDYALAYA GACHIBOWLI, GPRA CAMPUS, HYD–32
SAMPLE PAPER TEST 04 FOR BOARD EXAM (2022-23)
(ANSWERS)
SUBJECT: MATHEMATICS (041) MAX. MARKS : 80
CLASS : XII DURATION: 3 HRS
General Instructions:
1. This Question paper contains - five sections A, B, C, D and E. Each section is compulsory.
However, there are internal choices in some questions.
2. Section A has 18 MCQ’s and 02 Assertion-Reason based questions of 1 mark each.
3. Section B has 5 Very Short Answer (VSA)-type questions of 2 marks each.
4. Section C has 6 Short Answer (SA)-type questions of 3 marks each.
5. Section D has 4 Long Answer (LA)-type questions of 5 marks each.
6. Section E has 3 source based/case based/passage based/integrated units of assessment (4
marks each) with sub parts.

SECTION – A
Questions 1 to 20 carry 1 mark each.

 1
1. The value of sin  cos 1  is:
 2
(a) 1 (b) 1/√2 (c) 1/2 (d) √3/2
Ans: (d) √3/2
 1  3
sin  cos 1   sin 
 2 3 2

2. A dice is tossed thrice. The probability of getting an odd number at least once is: [1]
(a) 7/8 (b) 1/3 (c) 3/8 (d) 1/8
Ans: (a) 7/8
Required probability = 1 – Probability of getting no odd number
1 1 1 1 7
= 1 –    = 1 – 
2 2 2 8 8

    
3. For any two vectors a and b if a  b then the value of a.b is:
(a) 0 (b) 1 (c) 2 (d) -1
Ans: (a) 0
     
a  b | a || b | cos  | a || b | cos 90  0
2
 dy  d y
2
4. What is the sum of order and degree of the differential equation: 5 x    2  6 y  log x
 dx  dx
(a) 1 (b) 2 (c) 3 (d) not defined
Ans: (c) 3
Order = 2, Degree = 1. So, 2 + 1 = 3

5. If A and B are square matrices of order 3 such that |A| = 1 and |B| = 3, then the value of |3AB| is:
(a) 3 (b) 9 (c) 27 (d) 81
Ans: (d) 81
As AB is of order 3 and
|3AB| = 33|AB| = 27|A||B| = 27 × 1 × 3 = 81

Prepared by: M. S. KumarSwamy, TGT(Maths) Page - 1-


6. The area bounded by the shaded region as shown in the figure below is:

(a) 3/2 sq. units (b) 9/4 sq. units (c) 4 sq. units (d) 5/2 sq. units
Ans: (b) 9/4 sq. units

1
7. The value of e x
1
dx is:

(a) log ex + C (b) log|1 – e-x| + C


1
(c) log log x + C (d) log|ex – 1| + C
e
Ans: (b) log|1 – e-x| + C

8. The area bounded by the curve y = f(x), the y-axis, y = c and y = d is:
d b d b
(a)  f ( x)dx (b)  f ( x)dx (c)  y.dx (d)  y.dx
c a c a
d
Ans: (a)  f ( x)dx
c

9. If (iˆ  3 ˆj  8kˆ)  (3iˆ   ˆj   kˆ)  0 then λ and μ are respectively:


(a) 27, -9 (b) 9, 9 (c) -9, 18 (d) -1, 1
Ans: (a) 27, -9
Given, (iˆ  3 ˆj  8kˆ)  (3iˆ   ˆj   kˆ)  0
iˆ ˆj kˆ

1 3 9  0  iˆ(3  9 )  ˆj (   27)  kˆ(  9)  0iˆ  0 ˆj  0k
3  
On comparing the coefficients of i^,j^ and k^ we get,

Prepared by: M. S. KumarSwamy, TGT(Maths) Page - 2-


3µ + 9λ = 0, -µ + 27 = 0 and -λ – 9 = 0
⇒ µ = 27 and -λ = 9
⇒ µ = 27 and λ = -9

1 5 
10. For the matrix A =   , a symmetric matrix is:
6 7
1 7  1 5  5 1 5 1 
(a)   (b)   (c)   (d)  
5 6  5 2  6 7 5 2 
1 5 
Ans: (b)  
5 2 
Applying R2 → R2 – R1
1 5 
 A  
5 2 
T
1 5  1 5 
Now,    
5 2  5 2 

11. The interval in which y = x2e-x is increasing w.r.t x is:


(a) x ∈ (0, 2) (b) x ∈ [1, 0] (c) x ∈ (∞, 0] (d) x ∈ [0, ∞)
Ans: (a) x ∈ (0, 2)

1  sin x dy
12. If y = tan 1 , then value of at x = π/6 is:
1  sin x dx
(a) 1/2 (b) −1/2 (c) 1 (d) -1
Ans: (b) −1/2
 
1  cos   x 
y  tan 1
1  sin x
 tan 1  2   tan 1  tan    x  
1  sin x     2 2 
1  cos   x    
 2 
 x dy 1
   
2 2 dx 2

1
13. The value of  ( x  [ x])dx is:
1

(a) -1 (b) 0 (c) 1 (d) 2


Ans: (c) 1
1 1 1 0 1
I   ( x  [ x])dx   xdx  [ x ]dx  0  (1)  dx  0 dx [∵ x is an odd function]
1 1 1 1 0

Prepared by: M. S. KumarSwamy, TGT(Maths) Page - 3-


=0+1–0=1

14. In an LPP, the objective function is always:


(a) linear (b) quadratic (c) cubic (d) biquadratic
Ans: (a) linear

 x 1
15. If A =   and A2 is the identity matrix, then x is equal to:
1 0
(a) 0 (b) -1 (c) 1 (d) 2
Ans: (a) 0

1 2 4
16. The sum of cofactors of 7 and 12 in the determinant 5 7 8 is:
9 10 12
(a) -27 (b) -24 (c) -18 (d) 0
Ans: (a) -27

k 3 4 3
17. For what value of k ∈ N,  is .
4 k 0 1
(a) 4 (b) 1 (c) 3 (d) 0
Ans: (a) 4
k 3 4 3
Given, 
4 k 0 1
⇒ k2 – 12 = 4 – 0 ⇒ k2 = 16 ⇒ k = ±4 ⇒k=4∈N

18. If R is an equivalence relation defined in set A = {1, 2, 3 ….. 10} as R = {(a, b): |a – b| is a
multiple of 3}. The equivalence class of {1} is:
(a) {1} (b) {1, 2} (c) {1, 4, 10} (d) {1, 4, 7, 10}
Ans: (d) {1, 4, 7, 10}
For equivalence class {1}
(a, 1) ∈ R for a ∈ A
⇒ |a – 1| is a multiple of 3 ⇒ a – 1 = 3λ ⇒ a = 3λ + 1
a = 1, 4, 7, 10
∴ The equivalence class of {1} = {1, 4, 7, 10}

ASSERTION-REASON BASED QUESTIONS


In the following questions, a statement of assertion (A) is followed by a statement of Reason (R).
Choose the correct answer out of the following choices.
(a) Both A and R are true and R is the correct explanation of A.

Prepared by: M. S. KumarSwamy, TGT(Maths) Page - 4-


(b) Both A and R are true but R is not the correct explanation of A.
(c) A is true but R is false.
(d) A is false but R is true.

19. Assertion (A): A function f: N → N be defined by:


 n 1
 2 , if n is odd
f ( n)  
 n, if n is even for all n  N one  one
 2
Reason (R): A function f: A → B is said to be injective if f(a) = f(b) ⇒ a = b
Ans: (d) A is false but R is true.
11
For n = 1, f(1) = =1
2
For n = 2, f(2) = 2/2 = 1
⇒ f(1) = f(2) = 1
⇒ f(x) is not one-one
∴ Assertion is false

But the definition of injectivity is true so, the reason is true.

d xx
( x )  x x  x(1  2 log x )
x
20. Assertion (A):
dx
Reason (R): ( x )  x x  e x  e x log x
2 2 2
x x

Ans: (d) A is false but R is true.

SECTION – B
Questions 21 to 25 carry 2 marks each.

 x2  1 1 
21. Express tan 1   in the simplest form.
 x 
 
Ans:

Prepared by: M. S. KumarSwamy, TGT(Maths) Page - 5-


OR
 x 
Write in the simplest form of tan 1  
 a x 
2 2

Ans:

22. Find the area bounded by the curve y = cos x, x ∈ [0, π]



Ans:  | cos x | dx
0

OR

2
2  sin x
Find the value of  log 2  sin x dx

2

Ans:

 
23. Find the angle between the vectors a  iˆ  ˆj  k and b  iˆ  ˆj  k .
Ans: Let θ be the angle between the given two vectors. then,

Prepared by: M. S. KumarSwamy, TGT(Maths) Page - 6-


   
24. Find the value of λ and µ, if a  b  0 , where a  2iˆ  6 ˆj  27k and b  iˆ   ˆj   k .
Ans:

dy
25. Solve the differential equation: x  y  x2
dx
Ans:

SECTION – C
Questions 26 to 31 carry 3 marks each.
26. Let R be a relation on the set A of ordered pairs of positive integers defined as (x, y) R(u, v) if
and only if xv = yu. Show that, R is an equivalence relation. [3]
Ans: Clearly, (x, y) S(u, v) for all (x, y) ∈ A,
since xy = yx for all positive integers x and y.
This show that S is reflexive.
Further, (x, y) S(u, v)
⇒ xv = yu ⇒ uy = vx ⇒ (u, v) S(x, y)
and hence (u, v) S(x, y)
This shows that S is symmetric.
Suppose, (x, y) S(u, v) and (u, v) S(a, b)
So, xv = yu and ub = va

Prepared by: M. S. KumarSwamy, TGT(Maths) Page - 7-


a u b a a b
⇒ xv  yu ⇒ xv   yu [∵ ub = va ⇒  ]
u u v u u v
⇒ xb = ya
⇒ (x, y) S(a, b)
This shows that S is transitive.
Hence, S is an equivalence relation.

 dy  b
27. If x = a sin 2t (1 + cos 2t) and y = b cos 2t (1 – cos 2t) show that   
 dx t   a
4
Ans: Given that x = a sin2t (1 + cos2t) and y = b cos2t(1 – cos2t)
dx
= a[cos2t × 2(1 + cos2t) + sin2t(-sin2t) × 2]
dt
= 2a[cos2t (1 + cos2t) – sin 2t]
= 2a[cos2t + cos22t – sin22t) = 2a(cos2t + cos4t)
dy
= b[-sin2t × 2(1 – cos2t) + cos2t(2 × sin2t)
dt
= 2b[-sin2t + 2sin2t cos2t) = 2b(sin4t – sin2t)

OR
Find the derivative of the function given by f(x) = (1 + x)(1 + x2)(1 + x4)(1 + x8) and hence find
f'(1).
Ans:

28. Evaluate:  1  4x  x 2 dx
Ans:

Prepared by: M. S. KumarSwamy, TGT(Maths) Page - 8-


OR

x sin x
Evaluate:  1  cos
0
2
x
dx

Ans:

dy
29. Solve the differential equation: ( x  1)  2e  y  1 ; given y = 0 when x = 0.
dx
dy
Ans: Given differential equation is ( x  1)  2e  y  1
dx

30. The random variable X can take only the values 0, 1, 2, 3. Given that:
P(X = 0) = P(X = 1) = P and P(X = 2) = P(X = 3) such that Pi xi2  2Pi xi , find the value of P.

Prepared by: M. S. KumarSwamy, TGT(Maths) Page - 9-


Ans: Let P(X = 2) = P(X = 3) = a
1
 pi  1  a  2  p
 pi xi2  2 pi xi
 0( p )  1( p)  4(a)  9(a)  2(0( p)  1( p)  2(a )  3(a ))
 p  13a  2 p  10a
1  3
 p  3  p   p 
2  8
OR
1 1 1
A problem in Mathematics is given to three students whose chances of solving it are , , is
2 3 4
the probability that the problem is solved.
Ans: Let A, B, and C be the three students and P(A), P(B), P(C) be the probabilities of solving a
problem respectively.
P(A) = 1/2, P(B) = 1/3, P(C) = 1/4
P[problem will be solved at least by 1] = 1  P( A) P( B) P(C )
= 1 – [1 – P(A)] [1 – P(B)] [1 – P(C)]
1 2 3 1 3
=1–   =1– =
2 3 4 4 4

x  2 y 1 z  3
31. Find the points on the line   at a distance of 5 units from the point P(1, 3, 3).
3 2 2
x  2 y 1 z  3
Ans: Given, the equation of a line is:   = λ (say)
3 2 2
⇒ x = 3λ – 2, y = 2λ – 1, z = 2λ + 3
So, we have a point on the line is:
Q(3λ – 2, 2λ – 1, 2λ + 3) …….(i)
Now, given that distance between two points P(1, 3, 3) and Q(3λ – 2, 2λ – 1, 2λ + 3) is 5 units
i.e. PQ = 5
⇒ [(3  2  1) 2  (2  1  3)2  (2  3  3) 2 ]  5
On Squaring both sides, we get
(3λ – 3)2 + (2λ – 4)2 + (2λ)2 = 25
⇒ 9λ2 + 9 – 18λ + 4λ2 + 16 – 16λ + 4λ2 = 25
⇒ 17λ2 – 34λ = 0
⇒ 17λ (λ – 2) = 0
Either 17λ = 0 or λ – 2 = 0
∴ λ = 0 or 2
On putting λ = 0 and λ = 2 in equation (i),
we get the required point as (-2, -1, 3) or (4, 3, 7)

SECTION – D
Questions 32 to 35 carry 5 marks each.

32. Using integration, find the area of a ΔPQR, the coordinates of vertices being P(1, 6), Q(2, 8), and
R(3, 4).
Ans: On plotting the points P(1, 6), Q(2, 8), and R(3, 4), we notice, we have to find the shaded
area.

Prepared by: M. S. KumarSwamy, TGT(Maths) Page - 10


-
∴ ar(PQR) = ar(∠PQM) + ar(MQRN) – ar(∠PRN) ……..(i)
For ar(∠PQM): Curve PQ, x-axis between x = 1 and x = 2
86
Equation of PQ: y – 6 = (x – 1)
2 1
⇒ y – 6 = 2(x – 1)
⇒ y = 2x + 4
2
∴ ar(∠PQM) =  (2 x  4)dx = [ x 2  4 x ]12 = (4 + 8) – (1 + 4) = 7 ……(ii)
1
For ar(MQRN): Curve QR, x-axis between x = 2 and x = 3
48
Equation of QR: y – 8 = (x – 2)
3 2
⇒ y – 8 = -4(x – 2)
⇒ y = -4x + 16
3
∴ ar(MQRN) =  (4 x  16)dx = [2 x 2  16 x]02 = (-18 + 48) – (-8 + 32)
2
= 30 – 24 = 6 …….(iii)
For ar(LPRN): curve PR: x-axis between x = 1 and x = 3
46
Equation of PR: y – 6 = (x – 1) ⇒ y – 6 = -(x – 1)
3 1
⇒ y = -x + 7
2
3
  x2   9   1  33 13
∴ ar(∠PRN) =  ( x  7)dx =   7 x  =   21    7  = 
1  2 1  2   2  2 2
= 10 Sq. units …..(iv)
Substituting from (ii), (iii), (iv) in (i), we get
ar(ΔPQR) = 7 + 6 – 10 = 3 Sq. units

33. A tank with a rectangular base and rectangular sides, open at the top is to be constructed so that
its depth is 2 m and volume is 8 cu m. If the building of the tank costs ₹ 70 per sq meter for the
base and ₹ 45 per sq meter for the sides, what is the cost of the least expensive tank?
Ans: Let x and y be the length and breadth of the rectangular base of the open tank.
Then, the volume (V) and the surface area (S) of the tank are given by,
V = 2xy; S = xy + 2(2x + 2y)
⇒ 2xy = 8 ⇒ xy = 4 (∵ V = 8 cu. m)
 4
and S = 4  4  x   ……(i)
 x
dS  4 d 2 S 32
Now,  4 1  2   2  3
dx  x  dx x

Prepared by: M. S. KumarSwamy, TGT(Maths) Page - 11


-
dS  4
Equating to 0, we have, 4  1  2   0
dx  x 
⇒x =4⇒x=2
2

d 2 S 32
At x = 2,  =4>0
dx 2 8
So, S is the least when x = 2
Now, when x = 2, y = 2 (∵ xy = 4)
Thus, area of the base = xy = 4 m2
⇒ cost of the base = ₹ 280
and Area of four walls = 4(x + y) = 16
⇒ cost of walls = ₹ 720
⇒ Total cost of the tank = (₹ 280 + ₹ 720) = ₹ 1000
OR
x2 y2

Find the area of the greatest rectangle that can be inscribed in an ellipse  1.
a2 b2
x2 y2
Ans: Let ABCD be a rectangle having area A inscribed in an ellipse 2  2  1 …..(i)
a b

The area A of the rectangle is 4xy i.e. A = 4xy which gives A2 = 16x2y2 = s (say)


34. Find the shortest distance between the lines r  (4i  j )   (i  2 j  3k ) and

r  (i  j  2k )   (2i  4 j  5k )
Ans:

Prepared by: M. S. KumarSwamy, TGT(Maths) Page - 12


-
 1 1 0   2 2 4 
35. Given A = 2 3 4 and B =  4 2 4  , verify that BA = 6I, how can we use the result
 
   
 0 1 2   2 1 5 
to find the values of x, y, z from given equations x – y = 3, 2x + 3y + 4z = 17, y + 2z = 17
 1 1 0   2 2 4 
Ans: We have A =  2 3 4  and B =  4 2 4 
 
 0 1 2   2 1 5 
1 1 0   2 2 4   2  4  0 2  2  0 4  4  0 
Now, AB   2 3 4   4 2 4    4  12  8 4  6  4 8  12  20 
    
 0 1 2   2 1 5   0  4  4 0  2  2 0  4  10 
6 0 0 1 0 0
 0 6 0  6 0
 1 0
   
 0 0 6   0 0 1 
1
 AB  61  A 1  B
6
 2 2 4 
 A  4 2 4 
1 1
6 
 2 1 5 
The given system of linear equations can be written in matrix form as AX = B, where
 1 1 0  x  3
A  2 3 4 , X  y , B  17 
   
     
 0 1 2   z   7 
 X  A 1 B

Prepared by: M. S. KumarSwamy, TGT(Maths) Page - 13


-
 2 2 4   3   2  3  2  17  4  7 
 X  4 2 4 17  X  4  3  2 17  4  7 
1    1
6   6 
 2 1 5   7   2  3  117  5  7 
x   2 
  y    1  x  2, y  1, z  4
   
 z   4 
OR
 2 3 5 
If A =  3 2 4  ,find A-1. How we can use A-1 to find x, y, z for the following system of
 1 1 2 
equations: 2x − 3y + 5z = 11, 3x + 2y − 4z = 5, x + y − 2z = 3
Ans: Given system of equations
2x − 3y + 5z = 11
3x + 2y − 4z = 5
x + y − 2z = 3
 2 3 5  x 11
This can be written as AX = B where A   3 2 4  , X   y  , B   5 
     
 1 1 2   z   3 
Here, | A | 2(4  4)  3(6  4)  5(3  2) | A | 6  5  1
Since, ∣A∣ ≠ 0
Hence, the system of equations is consistent and has a unique solution given by X  A 1 B
adjA
A1  and adjA = CT
| A|
2 4
C11  (1)11  C11  4  4  0
1 2
3 4
C12  (1)1 2  C12  (6  4)  2
1 2
3 2
C13  (1)13  C13  3  2  1
1 1
3 5
C21  (1)2 1  C 21  (6  5)  1
1 2
2 5
C22  (1)2  2  C 22  4  5  9
1 2
2 3
C23  (1)2 3  C23  (2  3)  5
1 1
3 5
C31  (1)31  C31  12  10  2
2 4
2 5
C32  (1)3 2  C32  (8  15)  23
3 4
2 3
C33  (1)33  C33  4  9  13
3 2

Prepared by: M. S. KumarSwamy, TGT(Maths) Page - 14


-
0 2 1  0 1 2 
Hence, the co-factor matrix is C  1 9 5  adjA  C   2 9 23
  T
   
 2 23 13  1 5 13 
 0 1 2   0 1 2 
adjA 1 
A 1
 2 9 23  A   2 9 23
 1

| A | 1    
1 5 13   1 5 13
 x   0 1 2  11
Solution is given by  y    2 9 23  5 
 z   1 5 13  3 
x   56  x  8  x   1 
  
 y  22  45  69  y     1 
4   y    46
      4     
 z   11  25  39   z  12  z   25
Hence, x = −1,y = −46,z = −25

SECTION – E(Case Study Based Questions)


Questions 36 to 38 carry 4 marks each.

36. Case-Study 1: Read the following passage and answer the questions given below.
In a town, it’s rainy one-third of the day. Given that it is rainy, there will be heavy traffic with
probability 1/2. Given that it is not rainy, there will be heavy traffic with probability 1/4. If it’s
rainy and there is heavy traffic, I arrive late for work with probability 1/2. On the other hand, the
probability of being late is reduced to 1/8 if it, is not rainy and there is no heavy traffic. In other
situations (rainy and no heavy traffic, net rainy and heavy traffic), the probability of being late is
1/4. You pick a random day.

Based on the above information, answer the following questions:


(i) What is the probability that it’s not raining and there is heavy traffic and I am not late?
(ii) What is the probability that I am late?
(iii) Given that I arrived late at work, what is the probability that it rained that day?
OR
(iii) If P(not A) = 0.7, P(B) = 0.7 and P(B/A) = 0.5, then find the P(A/B)
Ans: From the given passage we can form a tree as below:

Prepared by: M. S. KumarSwamy, TGT(Maths) Page - 15


-
(i) P(RCTLC) = 1/8
(ii) P(I am late) = Sum of probabilities corresponds to “I am late”
= P(RTL) + (RTCL) + (RCTL) + (RCTCL)
1 1 1 1 11
    
12 24 24 16 48
 R  P( R  L)
(iii) P   
L P ( L)
Now, P(R ∩ L) = Sum of probabilities in which R and L are common
1 1 3 1 11
    and P( L) 
12 24 24 8 48
1
 R  P( R  L) 8 1 48 6
P       
L P( L) 11 8 11 11
48
OR
Given, P(A') = 0.7 and P(B') = 0.7 and P(B/A) = 0.5
Clearly, P(A) = 1 – P(A') = 1 – 0.7 = 0.3
 B  P( A  B) P( A  B)
Now, P     0.5   P( A  B)  0.15
 A P( A) 0.3
 A  P( A  B) 0.15 3
P     
B P( B) 0.7 14

37. Case-Study 2: Read the following passage and answer the questions given below.
Let R be the feasible region of a linear programming problem and let Z = ax + by be the
objective function. When Z has an optimal value (max. or min.), when the variable x and y are
subject to constraints described by linear inequalities, this optimal value occurs at the corner
point (vertex) of the feasible region.
Based on the above information, answer the following questions:
(i) What is an objective function of LPP? [1]
(ii) In solving an LPP “minimize f = 6x + 10y subject to constraints x ≥ 6, y ≥ 2, 2x + y ≥ 10, x ≥
0, y ≥ 0” which among is redundant constraint? [1]

Prepared by: M. S. KumarSwamy, TGT(Maths) Page - 16


-
(iii) The feasible region for an LPP is shown in the figure. Let Z = 3x – 4y, be the objective
function. Then, at which point minimum of Z occurs? [2]

OR
The feasible region for an LPP is shown shaded in the figure. Let F = 3x – 4y be the objective
function. Then, what is the maximum value of F. [2]

Ans: (i) Objective function is a linear function whose maximum or minimum values is to be
found.
(ii) When x ≥ 6 and y ≥ 2, then
2x + y ≥ 2 × 6 + 2
i.e., 2x + y ≥ 14
Hence, x ≥ 0, y ≥ 0, and 2x + y ≥ 10 are automatically satisfied by every point of the region.
Hence, answer is 2x + y ≥ 10, x ≥ 0, y ≥ 0.
(iii) Minimum of z = -32 at (0, 8)
Corner Point z = 3x – 4y
(0, 0) 0
(5, 0) 3 x 5 – 4 x 0 = 15
(6, 5) 3 x 6 – 4 x 5 = -2
(6, 8) 3 x 6 – 4 x 8 = -14
(4, 10) 3 x 4 – 4 x 10 = -28
(0, 8) 3 x 0 – 4 x 8 = -32

OR
Maximum of z = 0 at (0, 0)
Corner Point F = 3x – 4y
(0, 0) 0
(6,12) 3 x 6 – 4 x 12 = -30
(6, 16) 3 x 6 – 4 x 16 = -46
(0, 4) 3 x 0 – 4 x 4 = -16

38. Case-Study 3:
As we know good planning can save energy, time, and money. A farmer wants to construct a
circular well and a square garden in his field. He wants to keep their perimeters 600 m.

Prepared by: M. S. KumarSwamy, TGT(Maths) Page - 17


-
Based on the above information, answer the following questions:
(i) If the radius of the circular garden is ‘r’ m and the side of the square garden is ‘x’ m, then
what is the sum of their areas? And find the number which exceeds its square by the greatest
possible. [2]
(ii) At what radius, is the sum of their areas is least? [2]
Ans: (i) Sum of area = πr2 + x2
But 2πr + 4x = 600
600  2 r
x
4
 600  2 r 
2

∴ Sum of areas =  r 2   
 4 
Assume y is the difference between x and its square
i.e., y = x – x2
For max. difference between the numbers
dy
= 0 = 1 – 2x
dx
1
⇒x=
2
 600  2 r 
2

(ii) Sum of areas, S   r 2  x 2   r 2   


 4 
dS 4 
  2 r  (600  2 r )  (4r  300   r )
dr 16 2
dS 
Now,  0  (4r  300   r )  0   r  300  4r
dr 2
300
r
 4
d S 
2

 2   (4   )  0
 dr  r  300 2
 4
300
Hence, the sum of areas is least when r  .
 4

Prepared by: M. S. KumarSwamy, TGT(Maths) Page - 18


-
KENDRIYA VIDYALAYA GACHIBOWLI, GPRA CAMPUS, HYD–32
SAMPLE PAPER TEST 05 FOR BOARD EXAM (2022-23)
(ANSWERS)
SUBJECT: MATHEMATICS (041) MAX. MARKS : 80
CLASS : XII DURATION: 3 HRS
General Instructions:
1. This Question paper contains - five sections A, B, C, D and E. Each section is compulsory.
However, there are internal choices in some questions.
2. Section A has 18 MCQ’s and 02 Assertion-Reason based questions of 1 mark each.
3. Section B has 5 Very Short Answer (VSA)-type questions of 2 marks each.
4. Section C has 6 Short Answer (SA)-type questions of 3 marks each.
5. Section D has 4 Long Answer (LA)-type questions of 5 marks each.
6. Section E has 3 source based/case based/passage based/integrated units of assessment (4
marks each) with sub parts.

SECTION – A
Questions 1 to 20 carry 1 mark each.

 1 
1. What is the principal value of sin 1   ?
 2
       
(a) [-π, π] (b)  , (c) (-∞, ∞) (d)  ,
 2 2   4 4 
   
Ans: (b)  ,
 2 2 

2. Let A and B be the events associated with the sample space s, then the value of P(A/B) is:
(a) P(A/B) = 1 (b) P(A/B) = P(A) (c) 0 ≤ P(A/B) (d) 0 ≤ P(A/B) ≤ 1
Ans: (d) 0 ≤ P(A/B) ≤ 1

3. The vector equation of the line joining the points (3, -2, -5) and (3, -2, 6) is:
(a) (4iˆ  4 ˆj  5kˆ)   (12kˆ) (b) (4iˆ  4 ˆj  5kˆ)   (12kˆ)
(c) (6iˆ  2 ˆj  2kˆ)   (5kˆ) (d) (9iˆ  9 ˆj  2kˆ)   (2kˆ)
Ans: (a) (4iˆ  4 ˆj  5kˆ)   (12kˆ)
The vector equation of a line joining the points (3, -2, -5) and (3, -2, 6) is

r  (3iˆ  2 ˆj  5kˆ)  [(3  3)iˆ  (2  2) ˆj  (6  5)kˆ

 r  (3iˆ  2 ˆj  5kˆ)   (11kˆ)

Prepared by: M. S. KumarSwamy, TGT(Maths) Page - 1-


     
4. What is the angle between vectors a and b if | a | 1 , | b | 2 and a  b  i  j  k ?
(a) π/2 (b) π/3 (c) 2π/2 (d) π/6
Ans: (b) π/3

5. The area of a parallelogram whose one diagonal and one side are represented by 2i and 3 j is:
(a) 6 sq. units (b) 36 sq. units (c) 3 sq. units (d) 3/2 sq. units
Ans: (a) 6 sq. units
Area = | 2i  (3 j ) | 6 | k | 6 1 = 6 sq. units

6. If 3x + 2y = sin y, then dy/dx is:


3 sin y  1 2  sin y 2  cos y
(a) (b) (c) (d)
cos y  2 2 3 3
3
Ans: (a)
cos y  2
Given 3x + 2y = sin y
On differentiating w.r.t. x, we get
dy dy dy dy 3
3  2  cos y  (cos y  2)  3  
dx dx dx dx cos y  2

dy
7. If y = a 2  x 2 , then y is:
dx
(a) 0 (b) x (c) -x (d) 1
Ans: (c) -x

2
dx
8. The value of x
1 x2  1
is:

(a) π/3 (b) π/2 (c) π/4 (d) π/6


Ans: (a) π/3

Prepared by: M. S. KumarSwamy, TGT(Maths) Page - 2-


9. The area (in sq. units) enclosed by the curve shown in the given figure is: [1]

(a) 8/3 (b) 24/7 (c) 32/3 (d) 16/3


Ans: (c) 32/3

3
log x 2
10. The value of  dx is:
2 x
(a) log 6 log(3/2) (b) log(3/2)
1
(c) 2 log 3 (d) log 6
3
Ans: (a) log 6 log(3/2)

dy 1 y
11. The integrating factor of y is:
dx x
ex e x
(a) (b) (c) xex (d) x2ex
x x
ex
Ans: (a)
x

 2 x  y 4 x   7 7 y  13
12. If    then the value of x + y is:
 5x  7 4 x   y x  6 
(a) 7 (b) 4 (c) 5 (d) 2
Ans: (c) 5
On comparing the elements of the matrix, we get

Prepared by: M. S. KumarSwamy, TGT(Maths) Page - 3-


4x = x + 6 ⇒ 3x = 6 ⇒x=2
∴ 2x + y = 7
⇒ y = 7 – 2x ⇒y=7–2x2=3
∴x+y=2+3=5
1 1 1
13. The maximum value of ∆ = 1 1  sin  1 , where θ is a real number is:
1  cos  1 1
1
(a) 1 (b) (c) 3 (d) -1
2
1
Ans: (b)
2
1 1 1
We have, Δ = 1 1  sin  1
1  cos  1 1
= 1(1 + sin θ – 1) – 1(1 – 1 – cos θ) + (1 – (1 + sin θ) (1 + cos θ))
= sin θ + cos θ + [1 – 1 – sin θ – cos θ – sin θ cos θ]
= sin θ + cos θ – sin θ – cos θ – sin θ cos θ
= -sin θ cos θ
1
= − sin 2θ
2
1 1
Max. Δ = − (-1) = [∵ 1 ≤ sin θ ≤ 1]
2 2

14. If A is a symmetric matrix, then A3 is: [1]


(a) symmetric matrix (b) skew-symmetric matrix
(c) Identity matrix (d) row matrix
Ans: (a) Symmetric matrix
Since A is a symmetric matrix.
∴ A’ = A
Now (A3)’ = (A’)3 = A3 [∵ (A’)n = (An)’]
Hence, A3 is a symmetric matrix.
 2 a 5
15. If the matrix B =  1 4 b  is a symmetric matrix, then a + b + c is: [1]
 c 4 9 
(a) 0 (b) 5 (c) 4 (d) -1
Ans: (a) 0
 2 a 5
Given, B =  1 4 b  is a symmetric matrix.
 c 4 9 
∴ BT = B
 2 1 c   2 a 5 
⇒  a 4 4    1 4 b 
 5 b 9   c 4 9 

Prepared by: M. S. KumarSwamy, TGT(Maths) Page - 4-


∴ a = -1, b = -4, c = 5
⇒ a + b + c = -1 – 4 + 5 = 0

16. If set A contains 5 elements and the set B contains 6 elements, then the number of one-one and
onto mapping from A to B is:
(a) 600 (b) 56 (c) 65 (d) 0
Ans: (d) 0
Since A and B are two non-empty finite sets containing m and n elements respectively, then the
number of one-one and onto mapping from A to B is n, if m = n and 0, If m ≠ 0
there, m = 5 and n = 6
∴m≠n
So, the number of mapping = 0

17. The principal value branch of cosec-1x is:


(a) {−π/2, π/2} (b) {−π/2, π/2} – {0}
(c) {-∞, ∞} (d) {-π, π} – {0}
Ans: (b) {−π/2, π/2} – {0}

a b c a d g
18. If d e f = P, then what is the value of b e h , given P = 17?
g h i c f i
(a) 17 (b) -17 (c) 1/17 (d) −1/17
Ans: (a) 17
a b c
Consider ∆ = d e f = 17
g h i
We know that value of the determinant is the same if we take transpose.
a d g
∴ b e h = P = 17
c f i

ASSERTION-REASON BASED QUESTIONS


In the following questions, a statement of assertion (A) is followed by a statement of Reason (R).
Choose the correct answer out of the following choices.
(a) Both A and R are true and R is the correct explanation of A.
(b) Both A and R are true but R is not the correct explanation of A.
(c) A is true but R is false.
(d) A is false but R is true.

19. Assertion (A): The value of iˆ  ( ˆj  kˆ)  ˆj  (iˆ  kˆ)  kˆ  (iˆ  ˆj ) is 1.


Reason (R): Since, iˆ  iˆ  ˆj  ˆj  kˆ  kˆ  0
Ans: (c) A is true but R is false.
As iˆ  ( ˆj  kˆ)  ˆj  (iˆ  kˆ)  kˆ  (iˆ  ˆj )  iˆ  iˆ  ˆj  ( ˆj )  kˆ  kˆ
=1–1+1=1

Prepared by: M. S. KumarSwamy, TGT(Maths) Page - 5-


1
20. Assertion (A): If  (3 x 2  2 x  k )  dx  0 , then the value if k is -1.
0

x n1
Reason (R): x n  dx  .
n 1
Ans: (d) A is false but R is true.
1
We have,  (3 x 2  2 x  k )  dx  0
0

⇒ [ x  x  kx ]10 ⇒ [1 + 1 + k] – 0 = 0 ⇒ 2 + k = 0
3 2
⇒ k = -2

SECTION – B
Questions 21 to 25 carry 2 marks each.

21. Let S be the set of points in a plane and R be a relation in S defined as R = {(A, B) : d(A, B) < 2}
where d(A, B) represents the distance between the points A and B. Is R an equivalence relation?
Ans: Relation R = {(A, B): d(A, B) < 2}
For reflexive: For every A ∈ S, d(A, A) = 0 < 2
Hence, (A, A) ∈ R, ∀ A ∈ S.
Hence, reflexive.
For symmetric: Let (A, B) ∈ R
⇒ d(A, B) < 2
⇒ d(B, A) < 2
⇒ (B, A) ∈ R, for all A, B ∈ S.
Hence, symmetric.
For transitive: Let (A, B) ∈ R and (B, C) ∈ R
Let d(A, B) = 1.5 and d(B, C) = 1.7 and A, B, C are collinear
Let d(A, C) = 1.5 + 1.7 = 3.2 > 2
Hence (A, B) ∈ R, (B, C) ∈ R
⇒ (A, C) ∈ R
Hence, not transitive
∴ Relation R is not an equivalence relation.
OR
 
Find the value of cot   2 cot 1 3 
4 
Ans:

4
1
= 3 7
4
1
3
22. In a linear programming problem, objective function, z = x + 2y. The subjective the constraints
x + 2y ≥ 100, 2x – y ≤ 0, 2x + y ≤ 200, x ≥ 0, y ≥ 0

Prepared by: M. S. KumarSwamy, TGT(Maths) Page - 6-


The graph of the following equations is shown below.

Name the feasible region, and find the corner point at which the objective function is minimum.
Ans: Here the feasible region is ABCDEA
So, corner points are A(0, 50), B(20, 40), C(50, 100), E(0, 200)

Corner Points Z = x + 2y
A(0, 50) 100 Minimum
B(20, 40) 100 Minimum
C(50, 100) 250
D(0, 200) 400 Maximum

The minimum value of z is 100 at all the points on the line segment joining the points (0, 50) and
(20, 40).
 
23. Evaluate:  1  sin 2 xdx, x
4 2
Ans:

OR
dx
Evaluate:  sin 2
xcos 2 x
Ans:

24. Find the vector in the direction of the vector i  2 j  2k that has magnitude 9.
Ans:

Prepared by: M. S. KumarSwamy, TGT(Maths) Page - 7-


25. For the differential equation, find a particular solution satisfying the given condition (1 + sin2x)
dy + (1 + y2) cos x dx = 0, given that when x = π/2, y = 0
Ans:

SECTION – C
Questions 26 to 31 carry 3 marks each.
26. Show that the relation S in the set R of real numbers defined as S = {(a, b): a, b ∈ R and a ≤ b3}
is neither reflexive nor symmetric and nor transitive.
Ans: The given relation is:
S = {(a, b) : a, b ∈ R and a ≤ b3}
3
1 1 1
Reflexive: As    where ∈ R, is not true
2 2 2
1 1
∴  ,  ∉S
2 2
Thus, S is not reflexive.
Symmetric: As (-2) ≤ 33, where -2, 3 ∈ R, is true
but 3 ≤ (-2)3 is not true
i.e. (-2, 3) ∈ S but (3, -2) ∉ S.
Thus, S is not symmetric
3 3 3
3 3 4 3 4 4
Transitive: As 3 ≤   and    , where 3, , ∈ R are true but 3 ≤   is not true
2 2 3 2 3 3
 3 3 4  4
i.e.,  3,  ∈ S and  ,  ∈ S but  3,  ∉ S
 2 2 3  3
Thus, R is not transitive.
Hence, S is neither reflexive nor symmetric nor transitive.
Prepared by: M. S. KumarSwamy, TGT(Maths) Page - 8-
x 1
27. Evaluate:  ( x  2)( x  3) dx
Ans:

OR

x sin x
Evaluate:  1  cos
0
2
x
dx

Ans:

dy  y
28. Find the particular solution of the differential equation: x  y  x tan   , x ≠ 0. Given that
dx x

y , when x = 1.
4
Ans:

Prepared by: M. S. KumarSwamy, TGT(Maths) Page - 9-


OR
dy
Find the particular solution of the differential equation:  2 y tan x  sin x , given that y = 0
dx

when x  .
3
Ans:

29. A class has 15 students whose ages are 14, 17, 15, 14, 21, 17, 19, 20, 16, 18, 20, 17, 16, 19, 20
years one student is selected in such a manner that each has the same chance of being chosen and
the age X of the selected student is recorded. What is the probability distribution of random
variable X.
Ans: Here, the total number of students = 15
The ages of students in ascending order are 14, 14, 15, 16, 16, 17, 17, 17, 18, 19, 19, 20, 20, and
21.
2 1 2 3
Now, P ( X  14)  , P( X  15)  , P( X  16)  , P( X  17)  ,
15 15 15 15
1 2 3 1
P( X  18)  , P( X  19)  , P( X  20)  , P( X  21)  ,
15 15 15 15
Therefore, the probability distribution of random variable X is as follows:

X 14 15 16 17 18 19 20 21
No. of
2 1 2 3 1 2 3 1
students
2 1 2 3 1 2 3 1
P(X)
15 15 15 15 15 15 15 15

Prepared by: M. S. KumarSwamy, TGT(Maths) Page - 10-


30. The feasible region of a ∠PR is given as follows:

(i) Write the constraints with respect to the above in terms of x and y.
(ii) Find the coordinate of B and C and maximize, z = x + y.
Ans:
x y
(i) Equation of line is:  ≤1
25 50
⇒ 2x + y ≤ 50
x y
Equation of second line is:  ≤1
40 20
⇒ x + 2y ≤ 40
∴ Constraint are 2x + y ≤ 50, x + 2y ≤ 40, x ≥ 0, y ≥ 0
(ii) Coordinates of B are (20, 10) and C(0, 20)
∴ For z = x + y
Corner points z=x+y
(25, 0) 25
(20, 10) 30 Max
(0, 20) 20
(0, 0) 0

Hence, z is the maximum at the point (20, 10).

31. Find the shortest distance between the lines


 
r  (i  2 j  k )   (i  j  k ) and r  (2i  j  k )   (2i  j  2k )
Ans:

Prepared by: M. S. KumarSwamy, TGT(Maths) Page - 11-


SECTION – D
Questions 32 to 35 carry 5 marks each.

x2 y2
32. Find the area of the smaller region bounded by the ellipse   1 and the straight line 3x +
16 9
4y = 12.
Ans:

OR
Using integration, find the area bounded by the tangent to the curve 4y = x2 at the point (2, 1)
and the lines whose equations are x = 2y and x = 3y – 3.
Ans:

 1 1 2   2 0 1 
33. Use product  0 2 3  9 2 3 to solve the system of equations
 3 2 4   6 1 2 
x – y + 2z = 1
2y – 3z = 1
3x – 2y + 4z = 2
Ans:

Prepared by: M. S. KumarSwamy, TGT(Maths) Page - 12-


1 1 2   2 0 1
Consider the product  0 2 3  9 2 3
 3 2 4   6 1 2 
 2  9  12 0  2  2 1  3  4  1 0 0
  0  18  18 0  4  3 0  6  6    0 1 0 
 6  18  24 0  4  4 3  6  8   0 0 1 
1
1 1 2   2 0 1 
Hence,  0 2 3   9 2 3
 
 3 2 4   6 1 2 
Now, given system of equations can be written, in matrix form, as follows
 1 1 2   x   1 
 0 2 3  y    1 
    
 3 2 4   z   2 
1
 x  1 1 2   1   2 0 1   1 
  y   0 2 3  1    9 2 3  1 
    
 z   3 2 4   2  6 1 2   2 
 2  0  2  0 
  9  2  6   5 
 6  1  4   3
Hence x = 0, y = 5 and z = 3
OR
 2 3
Show that the matrix A   2
 satisfies the equation A – 4A + I = O, where I is 2 × 2 identity
 1 2 
matrix and O is 2 × 2 zero matrix. Using this equation, find A–1.
Ans:
2 3
Given that A   
1 2
 2 3   2 3  7 12 
A2  AA     
1 2  1 2   4 7 
 7 12  2 3 1 0
Hence, A2  4 A  I    4  
4 7  1 2   0 1 
 7 12  8 12 1 0  7  8  1 12  12  0   0 0 
       O
 4 7   4 8  0 1   4  4  0 7  8  1   0 0 
Now, A2  4 A  I  O
 AA  4 A   I
 AA( A1 )  4 AA1   IA1 (Post multiplying by A–1 because |A| ≠ 0)
 A( AA1 )  4 I   A1
 AI  4 I   A1
1 0  2 3   4  2 0  3   2 3
 A1  4 I  A  4     
 0 1   1 2   0  1 4  2   1 2 
 2 3
 A1   
 1 2 

Prepared by: M. S. KumarSwamy, TGT(Maths) Page - 13-


34. Prove that the height of the cylinder of maximum volume that can be inscribed in a sphere of
radius R is 2R/√3. Also find the maximum volume.
Ans: Let r and h be the base radius and height of cylinder respectively.
2
h
∴    r 2  R2 ----- ( i)
2
Now, V = Volume of the cylinder inscribed in a sphere = πr2h

 h2 
 V   h  R2  
 4 
 h3 
 V    R2h  
 4
dV  3h 2 
Now differentiating w.r.t. h, we get    R2  
dh  4 
d 2V  6h 
   0  
 4 
2
dh
dV 3h 2
Now for Maximum or minimum,  0  R2  0
dh 4
4 2 2R
 h2  R h
3 3
2
dV 3h 3 2R
For this value of h, 2
     .   3 R  0
dh 2 2 3
⇒ V is maximum
2R  2 1 4 2  2 R 2 2 4 3
Also maximum value of V   .  R  . R   . . R  R cu. units
3 4 3  3 3 3 3

35. Find the equation of a line passing through the point P(2, -1, 3) and perpendicular to the lines:
 
r  (iˆ  ˆj  kˆ)   (2iˆ  2 ˆj  kˆ) and r  (2iˆ  ˆj  3kˆ)   (iˆ  2 ˆj  2kˆ)
Ans: Line L is passing through point  (2iˆ  ˆj  3k) ˆ
 
L1  r  (iˆ  iˆ  kˆ)   (2iˆ  2 ˆj  kˆ) and L2  r  (2iˆ  ˆj  3kˆ)   (iˆ  2 ˆj  2kˆ)
Given that line is perpendicular to L1 and L2
Let the line L = (a1, a2, a3)
The equation of L in vector form ⇒
p is any constant.
So by condition that L is perpendicular to L1

Prepared by: M. S. KumarSwamy, TGT(Maths) Page - 14-


2a1 − 2a2 + a3 = 0 --- (1)
Now, L ⊥ L2
So, a1 + 2a2 + 2a3 = 0 --- (2)
Solve (1) and (2)
3a1 + 3a3 = 0 ⇒ a3 = − a1
Put it in (2), we get a1 + 2a2 − 2a1 = 0
⇒ a2 = a1/2
 a 
So, L   a1 , 1 , a1 
 2 
 1 
So, we can say DR of L= L   1, , 1
 2 
So, equation of L in vector form:
 ˆj
r  (2iˆ  ˆj  3kˆ)  k (iˆ   kˆ)
2
x  2 y 1 z  3
Cartesian form is   .
1 1 1
2

SECTION – E(Case Study Based Questions)


Questions 36 to 38 carry 4 marks each.

36. Case-Study 1: Read the following passage and answer the questions given below.
An electric circuit includes a device that gives energy to the charged particles constituting the
current, such as a battery or a generator; devices that use current, such as lamps, electric motors,
or computers; and the connecting wires or transmission lines.

An electric circuit consists of two subsystems say A and B as shown below:

For previous testing procedures, the following probabilities are assumed to be known.
P(A fails) = 0.2, P(B fails alone) = 0.15, P(A and B fail) = 0.15
Based on the above information answer the following questions:
(a) What is the probability that B fails? [1]
(b) What is the probability that A fails alone? [1]
(c) Find the probability that the whole of the electric system fails? [2]
OR
Find the conditional probability that B fails when A has already failed. [2]
Ans:
(a) Consider the following events
E = A fails, F = B fails

Given P(E) = 0.2, P ( E ∩ F) = 0.15, F(E ∩ F) = 0.15

Prepared by: M. S. KumarSwamy, TGT(Maths) Page - 15-



Since, P( E ∩ F) = 0.15
⇒ P(F) – P(E ∩ F) = 0.15
⇒ P(F) = 0.15 + P(E ∩ F)
⇒ P(F) = 0.15 + 0.15
⇒ P(F) = 0.30
(b) P(E ∩ F ) = P(E) – P(E ∩ F)
= 0.2 – 0.15
= 0.05
(c) If the electric system fails, we mean that A is also failed and B is also failed.
i.e., we have to find P(E ∪ F), where E is an event when A fails and B is an event when B fails.
∴ P(E ∪ F) = P(E) + P(F) – P(E ∩ F)
= 0.2 + 0.3 – 0.15
= 0.5 – 0.15
= 0.35
OR
Let E = A fail, F = B fail

∴ P(E) = 0.2, P(E ∩ F) = 0.15, P( F ∩ F) = 0.15
 B fails   F  P ( F  E ) 0.15 3
P   P    
 A fails  E P( E ) 0.20 4

37. Case-Study 3:
Mohini purchased a rectangular parallelopiped shaped box and a spherical ball inside it as a
showpiece. The sides of the box are x, 2x and x/3, and the radius of the sphere is y.

The sum of the surface area of the parallelopiped and sphere is given to be constant.
Based on the above information, answer the following questions:
(a) Let the constant surface area given to be S, then what is the relation between x and y? [1]
(b) If the combined volume is denoted by V, then what is the value of V? [1]
(c) If volume V is minimum, then how are x and y is related to each other? [2]
OR
If the shape has minimum volume when x = 2y, then what is the difference in the volume and
surface area of the shape? [2]
Ans: (a)

Prepared by: M. S. KumarSwamy, TGT(Maths) Page - 16-


(b)

(c)

OR

38. Case-Study 2: Read the following passage and answer the questions given below.
A general election of Lok Sabha is a gigantic exercise. About 911 million people were eligible to
vote and voter turnout was about 67%, the highest ever.

Let f(x) be the set of all citizens of India who were eligible to exercise their voting right in the
general election held in 2019. A relation ‘R’ is defined on I as follows:
b b
If f(x) is a continuous function defined on [a, b]  f ( x)dx   f (a  b  x)dx on the basis of the
a a

above information answer the following equations:



2
cos x
(a) Evaluate:  1 e

x
dx [2]
2

cos 2 x
(b) Find the value of  1  a x dx , a > 0. [2]


Ans: (a)

Prepared by: M. S. KumarSwamy, TGT(Maths) Page - 17-


(b)

Prepared by: M. S. KumarSwamy, TGT(Maths) Page - 18-


KENDRIYA VIDYALAYA GACHIBOWLI, GPRA CAMPUS, HYD–32
SAMPLE PAPER TEST 06 FOR BOARD EXAM (2022-23)
(ANSWERS)
SUBJECT: MATHEMATICS (041) MAX. MARKS : 80
CLASS : XII DURATION: 3 HRS
General Instructions:
1. This Question paper contains - five sections A, B, C, D and E. Each section is compulsory.
However, there are internal choices in some questions.
2. Section A has 18 MCQ’s and 02 Assertion-Reason based questions of 1 mark each.
3. Section B has 5 Very Short Answer (VSA)-type questions of 2 marks each.
4. Section C has 6 Short Answer (SA)-type questions of 3 marks each.
5. Section D has 4 Long Answer (LA)-type questions of 5 marks each.
6. Section E has 3 source based/case based/passage based/integrated units of assessment (4
marks each) with sub parts.

SECTION – A
Questions 1 to 20 carry 1 mark each.

1. If A is a square matrix such that A2 = A, then (I + A)2 – 3A is


(a) I (b) 2A (c) 3I (d) A
Ans: (a) I, as (I + A)2 – 3A = I2 + IA + AI + A2 – 3A = I + A + A + A – 3A = I

 cos x sin x 
2. If F(x) =   then F(x) F(y) is equal to
  sin x cos x 
(a) F(x) (b) F(xy) (c) F(x + y) (d) F(x – y)
Ans: (c) F(x + y)

2 x 1 3 0
3. If  , then find the value of x
4 2 2 1
(a) 3 (b) 2/3 (c) 3/2 (d) -1/4
Ans: (d) -1/4

4. Let A be a square matrix of order 2 × 2, then |KA| is equal to


(a) K|A| (b) K2|A| (c) K3|A| (d) 2K|A|
Ans: (b) K2|A|

5. A and B are invertible matrices of the same order such that |(AB)–1| = 8, If |A| = 2, then |B| is
(a) 16 (b) 4 (c) 6 (d) 1/16
Ans: (d) 1/16

Prepared by: M. S. KumarSwamy, TGT(Maths) Page - 1-


 sin x
  cos x, x  0
6. A function f (x) =  x is continuous at x = 0 for
 2k , x0
(a) k = 1 (b) k = 2 (c) k = 1/2 (d) k = 3/2
Ans: (a) k = 1
lim  sin x 
  cos x   1  1  2  2k  k  1
x  0 x 

d2y
7. If y = Ae5x + Be–5x then is equal to
dx 2
(a) 25y (b) 5y (c) –25y (d) 10y
Ans: (a) 25y
y′ = 5Ae5x – 5Be–5x
and y″ = 25Ae5x + 25Be–5x = 25y

 cos
3
8. x.e log(sin x ) dx is equal to
cos4 x sin 4 x esin x
(a)   C (b)  C (c) C (d) none of these
4 4 4
cos4 x
Ans: (a)  C
4

 /2
1
9. The value of  1  tan
0
3
x
dx is

(a) 0 (b) 1 (c) π/4 (d) π/2


Ans: (c) π/4

2
 d2y  dy d 3 y
10. If p and q are the degree and order of the differential equation  2   3  3  4 , then the
 dx  dx dx
value of 2p – 3q is
(a) 7 (b) –7 (c) 3 (d) –3
Ans: (b) –7
degree p = 1 and order q = 3
∴ 2p – 3q = 2 – 9 = –7

dy 2
11. General solution of differential equation  x 5  x 3  is
dx x
x6 x4 x 6
x 4
(a) y    2log | x | (b) y    2 log | x | 1
6 4 6 4
2 x6 x4
(c) y  5 x 4  3x 2  2  C (d) y    2log | x | C
x 6 4
6
x x4
Ans: (d) y    2log | x | C
6 4

Prepared by: M. S. KumarSwamy, TGT(Maths) Page - 2-


     
12. If for non zero vectors a and b , a  b is a unit vector and | a || b | 2 , then angle θ between
 
vectors a and b is
(a) π/2 (b) π/3 (c) π/6 (d) - π/2
Ans: (c) π/6

13. The area of a parallelogram whose one diagonal is 2i  j  2k and one side is 3i  j  k is
(a) i  4 j  k (b) 3√2 sq units (c) 6√2 sq units (d) 6 sq units
Ans: (b) 3√2 sq units

    
14. If | a | = 5, | b | = 13 and | a  b | = 25, then a.b is equal to
(a) 12 (b) 5 (c) 13 (d) 60
Ans: (d) 60

15. Direction ratios of a line are 2, 3, –6. Then direction cosines of a line making obtuse angle with
the y-axis are
2 3 6 2 3 6 2 3 6 2 3 6
(a) , , (b) , , (c) , , (d) , ,
7 7 7 7 7 7 7 7 7 7 7 7
2 3 6
Ans: (c) , ,
7 7 7

16. Feasible region is the set of points which satisfy


(a) the objective functions (b) some of the given constraints
(c) all of the given constraints (d) none of these
Ans: (c) all of the given constraints

17. Solution of LPP


To maximise Z = 4x + 8y
subject to constraints : 2x + y ≤ 30, x + 2y ≤ 24, x ≥ 3, y ≤ 9, y ≥ 0 is
(a) x = 12, y = 6 (b) x = 6, y = 12 (c) x = 9, y = 6 (d) none of these
Ans: (a) x = 12, y = 6

18. If events A and B are independent, P(A) = 0.35, P(A ∪ B) = 0.60 then P(B) is
(a) 0.25 (b) 0 (c) 0.95 (d) none of these
Ans: (d) none of these
P(A ∪ B) = P(A) + P(B) – P(A) (B)
for independent and events
⇒ 0.6 = 0.35 + P(B) – 0.35P(B) = 0.35 + 0.65 P(B)
⇒ P(B) = 0.25/0.65 = 0.38

Prepared by: M. S. KumarSwamy, TGT(Maths) Page - 3-


ASSERTION-REASON BASED QUESTIONS
In the following questions, a statement of assertion (A) is followed by a statement of Reason (R).
Choose the correct answer out of the following choices.
(a) Both A and R are true and R is the correct explanation of A.
(b) Both A and R are true but R is not the correct explanation of A.
(c) A is true but R is false.
(d) A is false but R is true.

19. Assertion (A): We can write sin–1x = (sin x) –1.


Reason (R): Any value in the range of principal value branch is called principal value of that
inverse trigonometric function.
Ans: (d) A is false but R is true.
Assertion: sin–1x should not be confused with (sin x) –1. In fact (sin x) –1; = 1/sin x and similarly
for other trigonometric functions.
Reason: The value of an inverse trigonometric function which lies in the range of principal
branch, is called the principal value of that inverse trigonometric function.
Hence, we can say that Assertion is false and Reason is true.

x 1 y  2 z  3
20. Assertion (A): The angle between the straight lines   and
2 5 4
x 1 y  2 z  3
  is 90°
1 2 3
Reason (R): Skew lines are lines in different planes which are parallel and intersecting.
Ans: (c) A is true but R is false.
For Assertion, a1a2 + b1b2 + c1c2 = 2(1) + 5(2) + 4(-3) = 0. Both lines are perpendicular
Reason (R) is wrong. In the space, there are lines neither intersecting nor parallel, such pairs of
lines are non-coplanar and are called skew lines.

SECTION – B
Questions 21 to 25 carry 2 marks each.

21. If sin [cot–1 (x + 1)] = cos (tan–1x), then find x.


Ans:

OR
Prove that the Greatest Integer Function f : R → R, given by f(x) = [x] is neither one-one nor
onto. Where [x] denotes the greatest integer less than or equal to x.
Ans: Given f : R → R defined by f(x) = [x]
For one one: We know by definition that for a ≤ x < a + 1, f(x) = a, a is an integer,
i.e. for x1, x2 ∈ [a, a + 1), x1 ≠ x2 ⇒ f(x1) = f(x2) = a.
Hence, not one-one.
For onto: For y (non integer) ∈ R in codomain there does not exist x ∈ R in domain such that
f(x) = y.
Hence, not onto.
Prepared by: M. S. KumarSwamy, TGT(Maths) Page - 4-
Alternative method: Given that f(x) = [x]
It is seen that f(1.4) = [1.4] = 1, f(1.8) = [1.8] = 1.
∴ f(1.4) = f(1.8), but 1.4 ≠ 1.8
∴ f is not one-one
Now, consider 0.7 ∈ R
It is known that f(x) = [x] is always an integer. Thus, there does not exist any element x ∈ R such
that f(x) = 0.7
∴ f is not onto.
Hence, the greatest integer function is neither one-one nor onto.

7x –7x d2y
22. If y = 500e + 600e , show that = 49y.
dx 2
Ans: Given that y = 500e7x + 600e–7x
dy
⇒ = 7 × 500e7x – 7 × 600e–7x
dx
d2y
⇒ = 49 × 500e7x + 49 × 600e–7x = 49(500e7x + 600e–7x)
dx 2
d2y
⇒ = 49y.
dx 2
     
23. If a  i  2 j  3k , b  3i  j  2k show that (a  b) and (a  b) are perpendicular to each other.
Ans:

OR
x 1 y  2 z  3 1 x y  2 3  z
Find the angle between the straight lines   and   .
2 5 4 1 2 3
Ans:

24. Show that the function f(x) = x3 – 3x2 + 6x – 100 is increasing on R.


Ans: We have, f(x) = x3 – 3x2 + 6x – 100 ...(i)
Differentiating (i) w.r.t. x, we get
f ′(x) = 3x2 – 6x + 6
= 3(x2 – 2x + 1) + 3
= 3(x – 1)2 + 3 > 0
(∵ For all values of x, (x – 1)2 is always positive)
∴ f ′ (x) > 0
So, f(x) is an increasing function on R.

Prepared by: M. S. KumarSwamy, TGT(Maths) Page - 5-


  
25. If a  i  j  k , b  4i  2 j  3k and c  i  2 j  k , find a vector of magnitude 6 units which is
  
parallel to the vector 2a  b  3c .
Ans:

SECTION – C
Questions 26 to 31 carry 3 marks each.
sin x
26. Evaluate:  (1  cos x)(2  cos x) dx
Ans:

 sin 4 x  4 
27. Evaluate:  e x   dx
 1  cos 4 x 
Ans:

 /3
1
28. Evaluate the following integral:  1
 /6 tan x
dx

Ans:

OR

Prepared by: M. S. KumarSwamy, TGT(Maths) Page - 6-



x
Evaluate the following integral:  1  sin x dx
0

Ans:

dy
29. Find the particular solution of the differential equation = 1 + x + y + xy, given that y = 0
dx
when x = 1.
dy
Ans: Consider equation = 1 + x + y + xy
dx
= 1(1 + x) + y(1 + x) = (1 + x) (1 + y)
dy
⇒ = (1 + x) dx
(1  y )
dy x2
Integrating both sides, we get  1 y 
 (1  x ) dx  log |1  y | x 
2
C

Given y = 0, when x = 1
1 3
log |1 + 0| = 1 + + C ⇒ C = 
2 2
x2 3
Substituting in (i), we get log |1  y | x   is the required solution.
2 2
OR
Find the particular solution of the differential equation (1 + e2x) dy + (1 + y2) ex dx = 0, given that
when x = 0, y = 1.
Ans:

30. Solve the following problem graphically: Minimise and Maximise Z = 3x + 9y subject to the
constraints: x + 3y ≤ 60; x + y ≥ 10; x ≤ y; x ≥ 0, y ≥ 0
Ans: Plotting the inequations x + 3y ≤ 60, x + y ≥ 10, x ≤ y, x ≥ 0, y ≥ 0.

Prepared by: M. S. KumarSwamy, TGT(Maths) Page - 7-


We notice common shaded portion is the feasible solution. Possible points for maximumand
minimum Z are A(5, 5),B(15, 15), C(0, 20), D(0, 10)

Minimum Z is at A(5, 5), i.e. x = 5, y = 5, Minimum Z = 60.


Maximum Z is at B(15, 15), i.e. x = 15, y = 15 and C(0, 20),
i.e. x = 0, y = 20, Maximum Z = 180.

31. A family has 2 children. Find the probability that both are boys, if it is known that (i) at least one
of the children is a boy. (ii) the elder child is a boy.
Ans:
S = {BB, BG, GB, GG}
(i) A: at least one of the children is a boy = BB, BG, GB
B: both are boys = BB
A ∩ B : BB
Probability of both boys when at least one of the children is a boy.
1
P( A  B) 4 1
P( B / A)   
P( A) 3 3
4
(ii) A: the elder child is a boy = BB, BG
B: both are boys = BB
A ∩ B : BB
Probability of the elder child is a boy.
1
P( A  B) 4 1
P( B / A)   
P( A) 2 2
4
OR
Bag I contains 3 red and 4 black balls and bag II contains 4 red and 5 black balls. One ball is
transferred from bag I to bag II and then a ball is drawn from bag II at random. The balls so
drawn is found to be red in colour. Find the probability that the transferred ball is black.
Ans: Bag I: 3 red + 4 black,
Bag II: 4 red + 5 black

Prepared by: M. S. KumarSwamy, TGT(Maths) Page - 8-


Case I : when ball transferred is black.
4
P(B/I) =
7
Total balls in bag II are 4 red + 6 black;
4
P(R/II) =
10
4 4
Probability in this case =  .
7 10
Case II: When ball transferred is red.
3
P(R/I) =
7
Total balls in bag II are 5 red + 5 black,
5
P(R/II) =
10
3 5
Probability in this case = 
7 10
Using Bayes’ Theorem, probability that the ball transferred is black
4 4

7 10 16 16
  
4 4 3 5 16  15 31
  
7 10 7 10

SECTION – D
Questions 32 to 35 carry 5 marks each.

32. Show that the relation S in the set R of real numbers defined as S = {(a, b): a, b ∈ R and a ≤ b3}
is neither reflexive nor symmetric and nor transitive.
Ans: The given relation is:
S = {(a, b) : a, b ∈ R and a ≤ b3}
3
1 1 1
Reflexive: As    where ∈ R, is not true
2 2 2
1 1
∴  ,  ∉S
2 2
Thus, S is not reflexive.
Symmetric: As (-2) ≤ 33, where -2, 3 ∈ R, is true
but 3 ≤ (-2)3 is not true
i.e. (-2, 3) ∈ S but (3, -2) ∉ S.
Thus, S is not symmetric
3 3 3
3 3 4 3 4 4
Transitive: As 3 ≤   and    , where 3, , ∈ R are true but 3 ≤   is not true
2 2 3 2 3 3
 3 3 4  4
i.e.,  3,  ∈ S and  ,  ∈ S but  3,  ∉ S
 2 2 3  3
Thus, R is not transitive.
Hence, S is neither reflexive nor symmetric nor transitive.

33. Using integration, find the area bounded by the lines x + 2y = 2, y - x = 1 and 2x + y = 7.
Ans: Given, x + 2y = 2 ...(i)
y – x = 1 ...(ii)
2x + y = 7 ...(iii)
On plotting these lines, we have

Prepared by: M. S. KumarSwamy, TGT(Maths) Page - 9-


 1 1 0   2 2 4 
34. Given A =  2 3 4  and B =  4 2 4  , verify that BA = 6I, use the result to solve the
 0 1 2   2 1 5 
system x – y = 3, 2x + 3y + 4z = 17, y + 2z = 7.
Ans:

2 2 4 1 1 0   2  4  0 2  6  4 0  8  8 
Consider AB =  4 2 4  2 3 4    4  4  0 4  6  4 0  8  8 

 2 1 5   0 1 2   2  2  0 2  3  5 0  4  10
6 0 0 1 0 0
⇒ BA   0 6 0   6 0
 1 0  6 I ------ (1)
 0 0 6  0 0 1 
Given equations x – y = 3;
2x + 3y + 4z = 17;
y + 2z = 7

Prepared by: M. S. KumarSwamy, TGT(Maths) Page - 10-


 1 1 0   x   3 
Matrix equation is  2 3 4   y   17  ⇒ AX = C ⇒ X = A–1C.
 0 1 2   z   7 
1
From (i), we have BA = 6I ⇒ B = 6IA–1 ⇒ A–1 = BC
6
 x  2 2 4   3  12   2 
  1    1   
X   y    4 2 4 17    6   1
6 6
 z   2 1 5   7   24   4 
⇒ x = 2, y = – 1, z = 4.
OR
The sum of three numbers is 6. If we multiply third number by 3 and add second number to it,
we get 11. By adding first and third numbers, we get double of the second number. Represent it
algebraically and find the numbers using matrix method.
Ans: Let the first , second and third number be x, y, z respectively.
Then, according to the given condition, we have
x+y+z=6
y + 3z = 11
x + z = 2y or x − 2y + z = 0
This system of equations can be written as AX = B, where
1 1 1 x  6 
A   0 1 3 , X   y  & B  11
1 2 1  z   0 
A = 1(1 + 6) – 0 + 1(3 − 1) = 9
⇒ |A| ≠ 0
∴ The system of equation is consistent and has a unique solution.
Now, we find adj(A)
A11  7, A12  3, A13  1 ,
A21  3, A22  0, A23  3 ,
A31  2, A32  3, A33  1
 7 3 2 
Hence, adj ( A)   3 0 3
 1 3 1 
 7 3 2 
adj ( A)   3 0 3
1 1 1
Thus, A 
| A| 9
 1 3 1 
Since, AX=B
∴ X = A−1B
 7 3 2   6  x   9  1 
1      1   
 X   3 0 3 11   y   18    2 
9 9
 1 3 1   0   z   27  3 
⇒ x = 1, y = 2, z = 3

35. Find the shortest distance between the lines whose vector equations are
 
r  (1  t )i  (t  2) j  (3  2t )k and r  ( s  1)i  (2s  1) j  (2 s  1)k
Ans:

Prepared by: M. S. KumarSwamy, TGT(Maths) Page - 11-


OR

Find the equation of a line passing through the point (1, 2, –4) and perpendicular to two lines
 
r  (8i  19 j  10k )   (3i  16 j  7 k ) and r  (15i  29 j  5k )   (3i  8 j  5k )
Ans:

SECTION – E(Case Study Based Questions)


Questions 36 to 38 carry 4 marks each.

36. Case-Study 1:
Let d1, d2, d3 be three mutually exclusive diseases.
Let S = {S1, S2, S3, S4, S5, S6} be the set of observable symptoms of these diseases. For example,
S1 is the shortness of breath, S2 is the loss of weight, S3 is the fatigue, etc. Suppose a random
sample of 10,000 patients contains 3200 patients with disease d1, 3500 patients with disease d2,
and 3300 patients with disease d3. Also, 3100 patients with disease d1, 3300 patients with disease
d2, and 3000 patients with disease d3 show the symptom S.

Prepared by: M. S. KumarSwamy, TGT(Maths) Page - 12-


Based on the above information answer the following questions:
(a) A person is chosen at random from the sample of 10,000. What is The probability that the
person chosen does not suffer from disease d3? [1]
(b) Find the conditional probability that the patient shows the symptom S given that he suffers
from disease d1 and also calculate the conditional probability that the patient shows the symptom
S given that he suffers from disease d2.
OR
If a person chosen at random shows the symptom S, then what is the probability that he does
suffer from disease d1? [2]
(c) Let Di denote the event that the patient has disease di (i = 1, 2, 3) and S be the event that the
3
d 
patient shows the symptom S. Then find the value of P  i  . [1]
1  s 
3300
Ans: (a) P(d 3 )   0.33
10000
P( S  d1 ) 3000 P( S  d 2 ) 3300
(b) P( S / d1 )    0.97 and P( S / d 2 )    0.94
P(d1 ) 3300 P(d 2 ) 3500
OR
P(d1 ) P(S / d1 )
P( D1 / S ) 
P(d1 ) P(S / d1 )  P(d 2 ) P( S / d 2 )  P(d3 ) P( S / d 3 )
0.32  0.97 0.3104
   0.33
0.32  0.97  0.35  0.94  0.33  0.91 0.937

(c) Total probability is equal to 1.

37. Case-Study 2:
Anil is the owner of a high rise residential society having 50 apartments. When he set rent at
Rs.10000/month, all apartments are rented. If he increases rent by Rs.250/ month, one fewer
apartment is rented. The maintenance cost for each occupied unit is Rs.500/month. Anil
represented the rent price per apartment by P and the number of rented apartments represented
by N.

Prepared by: M. S. KumarSwamy, TGT(Maths) Page - 13-


Based on the above information answer the following questions.
(a) If P = 10500, then find N [1]
(b) If P = 11,000, then find the profit. [1]
(c) Find the rent that maximizes the total amount of profit. [2]
Ans: (a) If P is the rent price per apartment and N is the number of rented apartment, the profit is
given by NP – 500 N = N(P – 500) [∵ Rs. 500/month is the maintenance charges for each
occupied unit]
Clearly, if P = 10500, then 10500 = 10000 + 250 x ⇒ x = 2 ⇒ N = 48
(b) If P = 11000, then 11000 = 10000 + 250 x ⇒ x = 4 and so profit P(4) = 250(50 – 4) (38 + 4)
= Rs. 4,83,000
(c) If x be the number of non-rented apartments, then N = 50 – x and P = 10000 + 250 x
Thus, profit = N(P – 500) = (50 – x) (10000 + 250 x – 500) = (50 – x) (9500 + 250 x) = 250(50
– x) (38 + x)
Now, P′(x) = 250[50 – x – (38 + x)] = 250[12 – 2x]
For maxima/minima, put P′(x) = 0
⇒ 12 – 2x = 0 ⇒ x = 6
Thus, price per apartment is, P = 10000 + 1500 = 11500
Hence, the rent that maximizes the profit is Rs. 11500.

38. Case-Study 3:
Shalini wants to prepare a handmade gift box for her friend's birthday at home. For making lower
part of box, she takes a square piece of cardboard of side 20 cm.

If x cm be the length of each side of the square cardboard which is to be cut off from corners of
the square piece of side 20 cm and Volume of the box is V then, answer the following questions.
dV
(a) Find the value of V for which = 0 [1]
dx
(b) Shalini is interested in maximising the volume of the box. So, what should be the side of the
square to be cut off so that the volume of the box is maximum? [2]
(c) Find the maximum value of the volume. [1]
Ans: (a) Height of open box = x cm
Length of open box = 20 - 2x and width of open box = 20 - 2x
∴ Volume (V) of the open box = x (20 - 2x)(20 - 2x)
dV
  x  2(20  2 x )(2)  (20  2 x) 2 = (20 – 2x)( -4x + 20 – 2x) = (20 – 2x)(20 – 6x)
dx
dV 10
Now,  0  20  2 x  0 or 20  6 x  0  x  10 or
dx 3
dV
(b)  (20  2 x)(20  6 x)
dx
d 2V
 2  (20  2 x )(6)  (20  6 x )(2) = (-2)[60 – 6x + 20 – 6x] = (-2)[80 – 12x] = 24x – 160
dx
10 d 2V d 2V
For x  , 2  0 and for x  10, 2  0
3 dx dx

Prepared by: M. S. KumarSwamy, TGT(Maths) Page - 14-


10
So, volume will be maximum when x 
3
2
10  10 
(c) Maximum volume   20  2  
3 3
10 40 40 16000 3
    cm
3 3 3 27

Prepared by: M. S. KumarSwamy, TGT(Maths) Page - 15-


KENDRIYA VIDYALAYA GACHIBOWLI, GPRA CAMPUS, HYD–32
SAMPLE PAPER TEST 07 FOR BOARD EXAM (2022-23)
(ANSWERS)
SUBJECT: MATHEMATICS (041) MAX. MARKS : 80
CLASS : XII DURATION: 3 HRS
General Instructions:
1. This Question paper contains - five sections A, B, C, D and E. Each section is compulsory.
However, there are internal choices in some questions.
2. Section A has 18 MCQ’s and 02 Assertion-Reason based questions of 1 mark each.
3. Section B has 5 Very Short Answer (VSA)-type questions of 2 marks each.
4. Section C has 6 Short Answer (SA)-type questions of 3 marks each.
5. Section D has 4 Long Answer (LA)-type questions of 5 marks each.
6. Section E has 3 source based/case based/passage based/integrated units of assessment (4
marks each) with sub parts.

SECTION – A
Questions 1 to 20 carry 1 mark each.

dy
1. If y = a 2  x 2 , then y is:
dx
(a) 0 (b) x (c) -x (d) 1
Ans: (c) -x

 1 1 2 
2. If Cij denotes the cofactor of element Pij of the matrix P =  0 2 3 , then the value of
 
 3 2 4 
C31.C23 is:
(a) 5 (b) 24 (c) –24 (d) –5
Ans: (a) 5
1 2 1 1
C31  (1)31  3  4  1 and C23  (1)2 3  (2  3)  5
2 3 3 2
 C31C23  (1)(5)  5

2
dx
3. The value of is x
1 x2  1
:

(a) π/3 (b) π/2 (c) π/4 (d) π/6


Ans: (a) π/3

sin 2 x  cos2 x
4. Evaluate:  sin 2 x cos 2 x dx
(a) tanx – cotx + C (b) –tanx + cotx + C

Prepared by: M. S. KumarSwamy, TGT(Maths) Page - 1-


(c) tanx + cotx + C (d) –tanx – cotx +C
Ans: (c) tanx + cotx + C

5. Let f: R → R be defined as f(x) = 3x. Choose the correct answer.


(a) f is one-one onto (b) f is many-one onto
(c) f is one-one but not onto (d) f is neither one-one nor onto
Ans: (a) f is one-one onto
f: R → R is defined as f(x) = 3x
Let x, y ∈ R such that f(x) = f(y)
⇒ 3x = 3y ⇒ x = y
∴ f is one-one.
y  y  y
Also, for any real number y in co-domain R, there exists in R such that f    3    y
3 3 3
∴ f is onto.
Hence, function f is one-one and onto.

 x2  d2y
6. If y  log e  2  then equals to
e  dx 2
1 1 2 2
(a)  (b)  2 (c) (d) 
x x x2 x2
2
Ans: (d)  2
x
 x2 
Given y  log e  2 
e 

7. The function f(x) = tan x – x:


(a) always increases (b) always decreases
(c) never increases (d) sometimes increases and sometimes decreases
Ans: (a) always increases
We have, f(x) = tan x – x
On differentiating with respect to x, we get
f'(x) = sec2x – 1
 f '( x )  0, x  R
So, f(x) always increases.

2
 d 2 y   dy 
2
 dy 
8. The degree of the differential equation  2      x sin   is
 dx   dx   dx 
(a) 1 (b) 2 (c) 3 (d) not defined
Ans: (d) not defined

2
dx
9. The value of e
0
1
sin x
is

(a)  (b) 0 (c) 3  (d) /2


Ans: (a) 

Prepared by: M. S. KumarSwamy, TGT(Maths) Page - 2-


10. The derivative of sin x with respect to log x is
cos x 1
(a) cos x (b) x cos x (c) (d) cos x
log x x
Ans: (b) x cos x
dy dt 1
y  sin x, t  log x   cos x, 
dx dx x
dy dy dx
Now,    x cos x
dt dx dt

11. The value of tan2(sec–12) + cot2(cosec–13) is


(a) 5 (b) 11 (c) 13 (d) 15
Ans: (b) 11
tan2(sec–12) + cot2(cosec–13)
= sec2(sec–12) – 1 + cosec2(cosec–13) – 1
= (2)2 –1 + (3)2 – 1 = 4 – 1 + 9 – 1 = 11.

x 3 y  2 z 5
12. If the equation of a line AB is   , find the direction ratios of a line parallel to
1 2 4
AB.
(a) 1, 2, 4 (b) 1, 2, –4 (c) 1, –2, –4 (d) 1, –2, 4
Ans: (d) The direction ratios of line parallel to AB is 1, –2 and 4.

1  sin x
13. If y = tan 1 , then value of dy/dx at x = π/6 is:
1  sin x
(a) 1/2 (b) −1/2 (c) 1 (d) -1
Ans: (b) −1/2

14. Area of parallelogram, whose diagonals are along vectors i  2k and 2 j  3k is
1
(a) 29 (b) 4i  3 j  2k (c) 29 (d) none of these
2

Prepared by: M. S. KumarSwamy, TGT(Maths) Page - 3-


1
Ans: (c) 29
2
1   1
Area of Parallelogram = (i  2k )  (2 j  3k )  29
2 2

15. Unit vectors along vector i  2 j  2k are


1 2 2 
(c)   i  j  k 
1 2  2 
(a)  (i  2 j  2k ) (b) i j k (d) none of these
3 3 3 3 3 3 
1 2 2 
Ans : (c)   i  j  k 
3 3 3 

4  x y 1 z
16. Direction ratios of the line   are
2 6 3
(a) 2, 6, 3 (b) –2, 6, 3 (c) 2, – 6, 3 (d) none of these
Ans : (c) 2, – 6, 3

5
17. The value of  e x dx is
4
4
(a) e (e + 1) (b) e4 (e – 1) (c) e2 (e – 1) (d) e2 (e +1)
Ans: (b) e4 (e – 1)

18. The solution of differential equation xdy − ydx =0 represents:


(a) a rectangular hyperbola (b) parabola whose vertex is at origin
(c) straight line passing through origin (d) a circle whose centre is at origin

ASSERTION-REASON BASED QUESTIONS


In the following questions, a statement of assertion (A) is followed by a statement of Reason (R).
Choose the correct answer out of the following choices.
(a) Both A and R are true and R is the correct explanation of A.
(b) Both A and R are true but R is not the correct explanation of A.
(c) A is true but R is false.
(d) A is false but R is true.

19. Assertion (A): Given that E and F are events such that P(E) = 0.6, P(F) = 0.3 and P(E ∩ F) =
0.2, then P(E|F) = 2/3
Reason (R): Given that E and F are events such that P(E) = 0.6, P(F) = 0.3 and P(E ∩ F) = 0.2,
then P(E|F) = 1/3
Ans. (c) A is true but R is false.
Given that, P(E) = 0.6, P(F) = 0.3 and P(E ∩ F) = 0.2
P( E  F ) 0.2 2
P( E / F )   
P(F ) 0.3 3
Hence, Assertion is true and Reason is false.
3

 (x  5)dx  30
3
20. Assertion(A):
3

Reason(R): f(x) = x³ + 5 is an odd function.


Ans. (c) A is true but R is false.

Prepared by: M. S. KumarSwamy, TGT(Maths) Page - 4-


Let f(x) = x³ +5
f(-x) = (-x)³ +5 = -x³ +5
f(x) is neither even nor odd.
Hence R is false.
3 3 3
3
 ( x  5)dx   x dx   5dx  0  5  x   30
3 3

3 3 3
3
Hence A is true.

SECTION – B
Questions 21 to 25 carry 2 marks each.

 2  1  2 
21. Find the value of cos1  cos   sin  sin 
 3   3 
Ans:

22. Two tailors A and B earn Rs. 150 and Rs. 200 per day respectively. A can stich 6 shirts and 4
pants per day while B can stich 10 shirts and 4 pants per day. Form a linear programming
problem to minimise the labour cost to produce at least 60 shirts and 52 pants.
Ans : Let A works for x days and B works for y days
Then LPP is
To Minimise cost Z = 150x + 200y subject to constraints,
x ≥ 0, y ≥ 0
6x + 10y ≥ 60
4x + 4y ≥ 52

 1  x  1 x  dy 1
23. If y = sin 1   , then show that 
 2  dx 2 1  x 2
Ans:

OR
 sin 5 x
  cos x, if x  0
For what value of ‘k’ is the function f ( x)   3 x continuous at x = 0?
 k , if x  0

Prepared by: M. S. KumarSwamy, TGT(Maths) Page - 5-


Ans:

24. Show that the line through the points (1, –1, 2), (3, 4, –2) is perpendicular to the line through the
points (0, 3, 2) and (3, 5, 6).
Ans: Let A (1, –1, 2) and B (3, 4, – 2) be given points.
Direction ratios of AB are
(3 – 1), {(4 – (–1)}, (–2 –2) i.e., 2, 5, – 4.
Let C (0, 3, 2) and D (3, 5, 6) be given points.
Direction ratios of CD are
(3 – 0), (5 – 3), (6 – 2) i.e., 3, 2, 4.
We know that two lines with direction ratios a1, b1, c1 and a2, b2, c2 are perpendicular if
a1a2 + b1b2 + c1c2 = 0.
∴ 2 × 3 + 5 × 2 + (– 4) × 4 = 6 +10 –16 = 0, which is true.
It will shows that lines AB and CD are perpendicular.

dy
25. Find the general solution of the differential equation  2 y  e3 x
dx
Ans:

OR
dy
Show that differential equation x  y (log y  log x  1) is a homogenous equation.
dx
Ans:

Prepared by: M. S. KumarSwamy, TGT(Maths) Page - 6-


SECTION – C
Questions 26 to 31 carry 3 marks each.
dy sin 2 (a  y )
26. If sin y = xsin(a + y), prove that: 
dx sin a
sin y dx cos y sin(a  y )  sin y cos(a  y )
Ans: sin y  x sin(a  y )  x   
sin(a  y ) dy sin 2 (a  y )
dx cos y (sin a cos y  cos a sin y )  sin y (cos a cos y  sin a sin y )
 
dy sin 2 (a  y )
dx cos2 y sin a  cos a sin y cos y  sin y cos a cos y  sin a sin 2 y
 
dy sin 2 (a  y )
dx cos 2 ysin a  sin a sin 2 y dx sin a(cos 2 y  sin 2 y )
   
dy sin 2 (a  y) dy sin 2 (a  y )
dx sin a dy sin 2 (a  y)
   
dy sin 2 (a  y) dx sin a


x sin x
27. Evaluate: 0 1  cos2 xdx

OR
3
Evaluate:  | x 2  2 x | dx.
1
Ans:

Prepared by: M. S. KumarSwamy, TGT(Maths) Page - 7-


28. If the position vectors of the vertices of a triangle are i  2 j  3k , 2i  3 j  k and 3i  j  2k ,
show that triangle is equilateral.
Ans:

OR
    
Find a unit vector perpendicular to each of the vectors a  b and a  b , where a  3i  2 j  2k

and b  i  2 j  2k .
Ans:

29. Find the general solution of differential equation:


 y  y    y  y 
 x cos  x   y sin  x   ydx   y sin  x   cos  x   xdy
         
Ans:

Prepared by: M. S. KumarSwamy, TGT(Maths) Page - 8-


OR
2
Solve the differential equation: ydx + (x – y ) dy = 0
Ans:

30. Solve the following LPP graphically:


Maximise Z = 3x + 4y
Subject to x + y ≤ 4, x ≥ 0 and y ≥ 0.
Ans: Given, Max. Z = 3x + 4y
Subject to x + y ≤ 4, x, y ≥ 0
for x + y = 4
x 0 4
y 4 0
Also, x = 0 and y = 0

The feasible region is a triangle with vertices O(0, 0), A(4, 0) and B(0, 4)
ZO = 3 × 0 + 4 × 0 = 0
ZA = 3 × 4 + 4 × 0 = 12
ZB = 3 × 0 + 4 × 4 = 16
Thus, maximum of Z is at B(0, 4) and the maximum value is 16

Prepared by: M. S. KumarSwamy, TGT(Maths) Page - 9-


31. Find the mean number of defective items in a sample of two items drawn one-by-one without
replacement from an urn containing 6 items, which include 2 defective items. Assume that the
items are identical in shape and size.
Ans: Let X denote the Random Variable defined by the number of defective items.

SECTION – D
Questions 32 to 35 carry 5 marks each.

32. The management committee of a residential colony decided to award some of its members
(say x) for honesty, some (say y) for helping others and some others (say z) for supervising
the workers to keep the colony neat and clean. The sum of all the awardees is 12. Three
times the sum of awardees for cooperation and supervision added to two times the number
of awardees for honesty is 33. If the sum of the number of awardees for honesty and
supervision is twice the number of awardees for helping others, using matrix method, find
the number of awardees of each category. Apart from these values, namely, honesty,
cooperation and supervision, suggest one more value which the management of the colony
must include for awards.
Ans:
According to question
x + y + z = 12
2x + 3y + 3z = 33
x – 2y + z = 0
The above system of linear equation can be written in matrix form as AX = B where
 1 1 1 x 12 
A   2 3 3 , X   y  and B  33
   
 1 2 1  z   0 
1 1 1
Here,| A | 2 3 3
1 2 1
= 1 (3 + 6) –1 (2–3) + 1 (– 4 –3) = 9 + 1 – 7 = 3
 A–1 exists.
A11 = 9, A12 = 1, A13 = –7
A21 = –3, A22 = 0, A23 = 3
A31 = 0, A32 = –1, A33 = 1
T
 9 1 7   9 3 0 
adj ( A)   3 0 3    1 0 1
 
 0 1 1   7 3 1 

Prepared by: M. S. KumarSwamy, TGT(Maths) Page - 10-


 9 3 0 
1
0 1
1 1
A  (adjA)   1
| A| 3
 7 3 1 
x  9 3 0  12 
Now, X  A B   y    1 0 1 33
 
1 1
3
 z   7 3 1   0 
x  108  99   9  3
  1  1   
  y   12  0  0   12    4 
3 3
 z   84  99  15  5 
 x = 3, y = 4, z = 5
No. of awards for honesty = 3
No. of awards for helping others = 4
No. of awards for supervising = 5.
The persons, who work in the field of health and hygiene should also be awarded.

33. Using integration, find the area of the region.


{(x, y) : x2 + y2 ≤ 2ax , y2 ≥ ax, x, y ≥ 0}
Ans: {(x, y) : x2 + y2 ≤ 2ax, y2 ≥ ax, x, y ≥ 0}
Considering the inequations as equations
x2 + y2 = 2ax ……(i)
⇒ x2 – 2ax + y2 = 0
⇒ x2 – 2ax + a2 + y2 = a2
⇒ (x – a)2 + y2 = a2 ..….(ii)

It represents a circle whose centre is (a, 0) and radius r = a


y2 = ax …..(iii)
Vertex (0, 0)
Axis along x-axis.
Point of intersection, from (i) and (iii)
x2 + ax = 2ax
⇒ x2 – ax = 0
⇒ x(x – a) = 0
⇒ x = 0, x = a
⇒ y = 0, y = ± a
Point of intersection are (0, 0) (a, a), (a, – a).
x2 + y2 ≤ 2ax
Area of shaded part: Required area

Prepared by: M. S. KumarSwamy, TGT(Maths) Page - 11-


OR
Make a rough sketch of the region {(x, y): 0 ≤ y ≤ x2, 0 ≤ y ≤ x, 0 ≤ x ≤ 2} and find the area of
the region using integration.
Ans: The points of intersection of the parabola y = x2 and the line y = x are (0, 0) and (1, 1).

34. Show that the rectangle of maximum area that can be inscribed in a circle of radius 'r' is a square
of side √2r.
Ans: Let ABCD be the rectangle inscribed in a circle of radius r
AC and BD are diameters of length 2r as angle is semicircle is always 90⁰
let x be the length and y be the breadth
x 2  y 2  (2r )2  x 2  y 2  ar 2  y  4r 2  x 2
Area of rectangle =xy
 A  x 4r 2  x 2

Prepared by: M. S. KumarSwamy, TGT(Maths) Page - 12-


dA 2r 2
  4r  x 
2 2

dx 2 4r 2  x 2
dA 4r 2  x 2  x 2 4r 2  2x 2
  
dx 4r 2  x 2 4r 2  x 2
dA a2  2x2
For maximum, 0 0
dx 4r 2  x 2
 4r 2  2x 2  0  x  2r
Since length and breadth are same,
So required rectangle of length and breadth is a square of √2r

35. Find the co-ordinates of the foot of the r and the length of the perpendicular drawn from the point

P(5, 4, 2) to the line r  (i  3 j  k )   (2i  3 j  k ) . Also, find the image of P in this line.
Ans: Any point on the line can be written in parametric form as (2λ − 1, 3λ + 3, −λ + 1)
Assuming this as the foot of perpendicular from (5,4,2), we can equate the dot product of this
vector and the line direction to zero.
 ((2  1  5)iˆ  (3  3  4) ˆj  (  1  2)kˆ)  (2iˆ  3 ˆj  kˆ)  0
 (2  6)  2  (3  1)  3  (  1)  (1)  0
 4  12  9  3    1  0
 14  14  0    1
The coordinates of the point are thus (1,6,0)
The length of the perpendicular can be found out by
(5  1)2  (4  6)2  (2  0)2  16  4  4  24
The foot of perpendicular would be the midpoint of P and the image of P in the line.
 (5iˆ  4 ˆj  2kˆ)  ( xiˆ  yjˆ  zkˆ)  2  (iˆ  6 ˆj )
 x  2  5  3, y  12  4  8, z  0  2  2
The image of point P is thus (−3,8,−2)
OR
Find the shortest distance between the following lines :
x 3 y 5 z 7 x 1 y 1 z 1
  and  
1 2 1 7 6 1
x 3 y 5 z 7 x 1 y 1 z 1
Ans: Let     and   k
1 2 1 7 6 1

Prepared by: M. S. KumarSwamy, TGT(Maths) Page - 13-


Now, let’s take a point on first line as
A(λ + 3, -2λ + 5, λ + 7) and let
B(7k - 1, - 6k - 1, k - 1) be point on the second line
The direction ratio of the line AB
7k - λ - 4, - 6k + 2λ - 6, k - λ - 8
Now as AB is the shortest distance between line 1 and line 2 so,
(7k - λ - 4)  1 + (- 6k + 2λ - 6)  (-2) + (k - λ - 8)  1 = 0 ...(i)
and (7k - λ - 4)  7 + (- 6k + 2λ - 6)  (-6) + (k - λ - 8)  1 = 0 ...(ii)
Solving equation (i) and (ii) we get
λ = 0 and k = 0
∴ A = (3, 5, 7) and B = (- 1, - 1, - 1)
∴ AB =  3  1 2   5  1 2   7  1 2  16  36  64  116  2 29

SECTION – E(Case Study Based Questions)


Questions 36 to 38 carry 4 marks each.

36. Case-Study 1:
A trust having a fund of ₹ 30000 invests in two different types of bonds. The first bond pays 5%
interest per annum which will be given to an orphanage and the second bond pays 7% interest
per annum which will be given to ‘Cancer Aid Society’ an NGO. The trust wishes to divide ₹
30000 among two types of bonds in such a way that they earn an annual total interest of ₹ 1800.

Based on the above information, answer the following questions:


(a) If the amount invested in the first bond by ₹ x and in the second bond is ₹ y, then what is the
system of equations formed? [1]
(b) Write the system of equations in matrix form. [1]
(c) Find the values of ‘x’ and ‘y’. [2]
OR
 1 1 
(c) What is the inverse of the matrix   ? [2]
5 7 
Ans:
(a) Let x be invested in the first bond and ₹ y is invested in the second bond, and the total amount
invested is ₹ 30,000
⇒ x + y = 30000
5% of x + 7% of y = 1800
⇒ 5x + 7y = 18000
(b) We have equations,
x + y = 30000 and 5x + 7y = 1800000
1 1   x   30000 
Then their Matrix form is      
5 7   y  180000 

Prepared by: M. S. KumarSwamy, TGT(Maths) Page - 14-


(c) We have,
x + y = 30000 …….(i)
and 5x + 7y = 180000 ……(ii)
Multiplying equation (i) by 5 and subtracting equations (ii) from (i),
(5x + 5y) – (5x + 7y) = 150000 – 180000
⇒ -2y = -30000
⇒ y = 15000
From (i), x = 30000 – 15000 = 15000
Thus, x = 15000, y = 15000
OR

37. Case-Study 2:
In a test, you either guesses or copies or knows the answer to a multiple-choice question with
four choice. The probability that you make a guess is 1/3, you copy the answer is 1/6. The
probability that your answer is correct, given that you guess it, is 1/8. And also, the probability
that you answer is correct, given that you copy it, is 1/4.

(i) The probability that you know the answer. [1]


(ii) Find the probability that your answer is correct given that you guess it and the probability
that your answer is correct given that you know the answer . [1]
(iii) Find the probability that you know the answer given that you correctly answered it. [2]
OR
(iii) Find the total probability of correctly answered the question. [2]
Ans: (i) Let E1 be the event that he guesses
E2 be the event that he copies
E3 be the event that he knows the answer.
Let A be the event that he answered correctly.
1 1 1
Given, P( E1 )  , P( E2 )  , P( E3 ) 
3 6 2
1 1 1 1
P( E3 )  1      1  
3 6 2 2

Prepared by: M. S. KumarSwamy, TGT(Maths) Page - 15-


1
(ii) P( A / E1 )  , P( A / E3 )  1
8
P( E3 )  P( A / E3 )
(iii) P( E3 / A) 
P ( E1 )  P( A / E1 )  P ( E2 )  P( A / E2 )  P( E3 )  P( A / E3 )
1
1 6
 P( E3 / A)  2 
1 1 1 1 1
    1 7
3 8 6 4 2
OR
1 1 1 1 1 7
P( A)  P( E1 )  P( A / E1 )  P( E2 )  P( A / E2 )  P( E3 )  P( A / E3 )       1 
3 8 6 4 2 12

38. Case-Study 3:
Sherlin and Danju are playing Ludo at home during Covid-19. While rolling the dice, Sherlin’s
sister Raji observed and noted the possible outcomes of the throw every time belongs to set {1, 2,
3, 4, 5, 6}. Let A be the set of players while B be the set of all possible outcomes.
A = {S, D}, B = {1, 2, 3, 4, 5, 6}

(i) Show that relation R : B ∈ B be defined by R = {(x, y) : y is divisible by x} is reflexive and


symmetric but not transitive.
(ii) Let R be a relation on B defined by R = {(1, 2), (2,2), (1, 3), (3, 4), (3, 1), (4, 3), (5, 5)}. Then
R is show that R is neither reflexive nor symmetric nor transitive.
Ans: (i) R is reflexive, since every element of B i.e.,
B = {1, 2, 3, 4, 5, 6} is divisible by itself.
i.e., (1, 1), (2, 2), (3, 3), (4, 4), (5, 5), (6, 6) ⋴ R
further, (1, 2) ⋴ R
but (2, 1) ⋴ R
Moreover, (1, 2), (2, 4) ⋴ R
⇒ (1, 4) ⋴ R
⇒ R is transitive.
Therefore, R is reflexive and transitive but not symmetric.
(ii) R = {(1, 2), (2, 2), (1, 3), (3, 4), (3, 1), (4, 3), (5, 5)}
R is not reflexive.
Since, (1, 1), (3, 3), (4, 4), (6, 6) ⋴ R
R is not symmetric.
Because, for (1, 2) ⋴ R there (2, 1) ⋴ R.
R is not transitive.
Because for all element of B there does not exist, (a, b)(b, c) ⋴ R and (a, c) ⋴ R.

Prepared by: M. S. KumarSwamy, TGT(Maths) Page - 16-


KENDRIYA VIDYALAYA GACHIBOWLI, GPRA CAMPUS, HYD–32
SAMPLE PAPER TEST 08 FOR BOARD EXAM (2022-23)
(ANSWERS)
SUBJECT: MATHEMATICS (041) MAX. MARKS : 80
CLASS : XII DURATION: 3 HRS
General Instructions:
1. This Question paper contains - five sections A, B, C, D and E. Each section is compulsory.
However, there are internal choices in some questions.
2. Section A has 18 MCQ’s and 02 Assertion-Reason based questions of 1 mark each.
3. Section B has 5 Very Short Answer (VSA)-type questions of 2 marks each.
4. Section C has 6 Short Answer (SA)-type questions of 3 marks each.
5. Section D has 4 Long Answer (LA)-type questions of 5 marks each.
6. Section E has 3 source based/case based/passage based/integrated units of assessment (4
marks each) with sub parts.

SECTION – A
Questions 1 to 20 carry 1 mark each.

1 5 
1. For the matrix A =   , a symmetric matrix is:
6 7
1 7  1 5  5 1 5 1 
(a)   (b)   (c)   (d)  
5 6  5 2  6 7 5 2 
1 5 
Ans: (b)  
5 2 

 1 1
2. The inverse of the matrix   is:
2 3 
 1 1   3 1
5 5   5 5  3 2  1 1 
(a)   (b)   (c)   (d)  
2 3   2 1   1 1   3 2
 5 5   5 5 
 3 1
 5 5
Ans: (b)  
 2 1 
 5 5 

3. The area of a triangle with vertices A, B, C is given by


  1   1   1  
(a) | AB  AC | (b) | AB  AC | (c) | AC  AB | (d) | AC  AB |
2 4 8
1  
Ans: (b) | AB  AC |
2
1  cos 4 x
 , if x  0
4. The value of ‘k’ for which the function f ( x )   8 x 2 is continuous at x = 0 is
 k , if x  0
(a) 0 (b) -1 (c) 1. (d) 2
Ans: (c) 1

Prepared by: M. S. KumarSwamy, TGT(Maths) Page - 1-


5. The scalar projection of the vector 3i  j  2k ℎ i  2 j  3k is
7 7 6 7
(a) (b) (c) (d)
14 14 13 2
7
Ans: (a)
14
1
6. The value of  ( x  [ x])dx is:
1
(a) -1 (b) 0 (c) 1 (d) 2
Ans: (c) 1

1 2 4
7. The sum of cofactors of 7 and 12 in the determinant 5 7 8 is:
9 10 12
(a) -27 (b) -24 (c) -18 (d) 0
Ans: (a) -27

8. The corner points of the shaded unbounded feasible region of an LPP are (0, 4), (0.6, 1.6) and (3,
0) as shown in the figure. The minimum value of the objective function Z = 4x + 6y occurs at

(a)(0.6, 1.6) (b) (3, 0) only (c) (0.6, 1.6) and (3, 0) only
(d) at every point of the line-segment joining the points (0.6, 1.6) and (3, 0)
Ans: (d) at every point of the line-segment joining the points (0.6, 1.6) and (3, 0)

 x 1
9. If A =   and A2 is the identity matrix, then x is equal to:
1 0
(a) 0 (b) -1 (c) 1 (d) 2
Ans: (a) 0

Prepared by: M. S. KumarSwamy, TGT(Maths) Page - 2-


1
10. If '( ) = +
, then ( ) is
x
x2 x x
2
(a) + log | | + (b) + log | | + (c) + log | | + (d) − log | | +
2 2 2
x2
Ans: (b) + log | | +
2

11. If A is a square matrix of order 3 and |A| = 5, then | |=


1
(a) 5 (b) 25 (c) 125 (d)
5
Ans: (b) 25

12. If the following table represents a probability distribution for a random variable X:
X 1 2 3 4 5 6
P(X) 0.1 2k K 0.2 3k 0.1
The value of k is:
(a) 0.01 (b) 0.1 (c) 1/1000 (d) 25
Ans: (b) 0.1
In the probability distribution of X, ΣP(X) = 1
⇒ (0.1) + 2k + k + (0.2) + 3k + (0.1) = 1
⇒ 6k = 0.6 ⇒ k = 0.1

13. If m and n, respectively, are the order and the degree of the differential equation
4
d  dy  
   0 , then m + n =
dx  dx  
(a) 1 (b) 2 (c) 3 (d) 4
Ans: (c) 3

d2y
14. The solution of  9 y  6cos 3 x  0 is:
dx 2
(a) y = x cos 3x (b) y = x sin 3x (c) y = x cos 2x (d) y = x sin 2x
Ans: (b) y = x sin 3x

1  sin x
15. If y = tan 1 , then value of dy/dx at x = π/6 is:
1  sin x
(a) 1/2 (b) −1/2 (c) 1 (d) -1
Ans: (b) −1/2
    
16. If two vectors a b are such that | a | = 2 , | b | = 3 a.b = 4, ℎ is equal to
(a) √2 (b) 2√6 (c) 24 (d) 2√2
Ans: (b) 2√6

17. The solution set of the inequality 3x + 5y < 4 is


(a) an open half-plane not containing the origin.
(b) an open half-plane containing the origin.
(c) the whole XY-plane not containing the line 3x + 5y = 4.
(d) a closed half plane containing the origin.
Ans: (b) an open half-plane containing the origin.

18. The acute angle between two lines whose direction ratios are 2, 3, 6 and 1, 2, 2 is:

Prepared by: M. S. KumarSwamy, TGT(Maths) Page - 3-


20 19 1 1
(a) cos 1 (b) cos 1 (c) cos 1 (d) cos 1
21 21 3 7
20
Ans: (a) cos 1
21

ASSERTION-REASON BASED QUESTIONS


In the following questions, a statement of assertion (A) is followed by a statement of Reason (R).
Choose the correct answer out of the following choices.
(a) Both A and R are true and R is the correct explanation of A.
(b) Both A and R are true but R is not the correct explanation of A.
(c) A is true but R is false.
(d) A is false but R is true.

−1  1 1 
19. Assertion (A): The domain of the function 2 is  ,     ,  
 2 2 
−1 
Reason (R): (−2) = −
4
Ans: (c) A is true but R is false.

 
20. Assertion (A): The acute angle between the line r  i  j  2k   (i  j ) and the x-axis is
4

Reason(R): The acute angle between the lines r  x1i  y1 j  z1 k   (a1 i  b1 j  c1 k ) and
 | a1a2  b1b2  c1c2 |
r  x2 i  y2 j  z2 k   (a2 i  b2 j  c2 k ) is given by cos   .
a12  b12  c12 a2 2  b2 2  c2 2
Ans: (a) Both A and R are true and R is the correct explanation of A.

SECTION – B
Questions 21 to 25 carry 2 marks each.

 x 
21. Write in the simplest form of tan 1  
 a x 
2 2

Ans:

OR
Prove that the function f is surjective, where : → such that
n 1
 2 , if n is odd
f ( n)  
 n , if n is even
 2
Is the function injective? Justify your answer.

Prepared by: M. S. KumarSwamy, TGT(Maths) Page - 4-


Ans: Let y  N (codomain). Then  2y  N (domain)
2y
such that f(2y) = y
2
Hence, f is surjective.
1, 2  N (domain) such that f(1) = 1 = f(2) 1
Hence, f is not injective.

22. A stone is dropped into a quiet lake and waves move in circles at a speed of 4cm per second. At
the instant, when the radius of the circular wave is 10 cm, how fast is the enclosed area
increasing?
Ans: The area A of a circle with radius r is given by A = πr2. Therefore, the rate of change of
area A with respect to time t is
dA d d dr dr
 ( r 2 )  ( r 2 )  2 r
dt dt dr dt dt
 dr 
Therefore, when r = 10 cm, dA/dt = 2π (10) (4) = 80π   4cm / s 
 dt 
Thus, the enclosed area is increasing at the rate of 80π cm2/s, when r = 10 cm.
 
23. Find the angle between the vectors a  iˆ  ˆj  k and b  iˆ  ˆj  k .
Ans:

OR
   
Find the value of λ and µ, if a  b  0 , where a  2iˆ  6 ˆj  27k and b  iˆ   ˆj   k .
Ans:

 
24. If f (x) = sin 2x – cos 2x, find f '  
6
Ans:

     
25. Find | x | if ( x  a )( x  a)  12 , where a is a unit vector.
Ans:

Prepared by: M. S. KumarSwamy, TGT(Maths) Page - 5-


SECTION – C
Questions 13 to 22 carry 3 marks each.
2x
26. Find:  (x 2
 1)( x 2  2)
dx

Ans:

OR
Find:  e .sin 2 xdx
x

Ans:

27. There are two boxes, namely box-I and box-II. Box-I contains 3 red and 6 black balls. Box-II
contains 5 red and 5 black balls. One of the two boxes, is selected at random and a ball is drawn
at random. The ball drawn is found to be red. Find the probability that this red ball comes out
from box-II.
Ans: Let E1 = Selecting Box-I and E2 = Selecting Box-II
A = getting a red ball from the selected box
Here, P(E1) = 1/2, P(E2) = 1/2

Prepared by: M. S. KumarSwamy, TGT(Maths) Page - 6-


P(A/E1) = 3/9 = 1/3
P(A/E2) = 5/10 = 1/2
Required probability = P(E/A2) = P(Red ball comes out from Box-II)
P(E 2 )  P(A / E 2 )
Using Bayes' theorem, P(E 2 / A) 
P(E1 )  P(A / E1 )  P(E 2 )  P(A / E 2 )
1 1 1 1
 1 24 3
 2 2  4  4   
1 1 1 1 1 1 10 4 10 5
   
2 3 2 2 6 4 24

OR
Find the mean number of defective items in a sample of two items drawn one-by-one without
replacement from an urn containing 6 items, which include 2 defective items. Assume that the
items are identical in shape and size.
Ans: Let X denote the Random Variable defined by the number of defective items.
4 3 2
P(X = 0) =  
6 5 5
2 4 8
P(X = 1) = 2     
 6 5  15
2 1 1
P(X = 2) =  
6 5 15
xi 0 1 2
pi 2/5 8/15 1/15
pixi 0 8/15 2/15

10 2
Mean   pi xi  
15 3
 /4
dx
28. Evaluate:  1  tan x
0

Ans:

Prepared by: M. S. KumarSwamy, TGT(Maths) Page - 7-


dy
29. Find the particular solution of the differential equation x  y  x 2 .e x , given y(1) = 0.
dx
dy
Ans: Given differential equation is x  y  x 2 .e x
dx

OR
dy
Find the general solution of the differential equation x  y (log y  log x  1)
dx
dy
Ans: Given differential equation is x  y (log y  log x  1)
dx

30. Solve the following Linear Programming Problem graphically:


Maximize Z = 400x + 300y subject to + ≤ 200, ≤ 40, ≥ 20, ≥0
Ans: We have Z = 400x + 300y subject to
x + y < 200, x < 40, x > 20, y > 0
The corner points of the feasible region are
C(20, 0) D(40, 0), B(40, 160), A(20, 80)

Prepared by: M. S. KumarSwamy, TGT(Maths) Page - 8-


Maximum profit occurs at x = 40, y = 160
and the maximum profit = Rs. 64, 000
1
31. Find 
2
5  4x  x 2 dx

Ans:

SECTION – D
Questions 32 to 35 carry 5 marks each.

32. Find the area of the region bounded by curve 4x2 = y and the line y = 8x + 12, using integration.
Ans: Given curve is 4x2 = y and line is y = 8x + 12
On solving both equations, we get 4x2 = 8x + 12
⇒ x2 = 2x + 3
⇒ x2 – 2x – 3 = 0
⇒ x = 3, –1

Prepared by: M. S. KumarSwamy, TGT(Maths) Page - 9-


33. Let R be a relation on the set A of ordered pairs of positive integers defined as (x, y) R(u, v) if
and only if xv = yu. Show that, R is an equivalence relation.
Ans: Clearly, (x, y) R (u, v) for all (x, y) ∈ A,
since xy = yx for all positive integers x and y.
This show that R is reflexive.
Further, (x, y) R (u, v)
⇒ xv = yu ⇒ uy = vx ⇒ (u, v) R (x, y)
and hence (u, v) R (x, y)
This shows that R is symmetric.
Suppose, (x, y) R (u, v) and (u, v) R (a, b)
So, xv = yu and ub = va
a a
⇒ xv  yu
u u
b a
⇒ xv   yu [∵ ub = va ⇒ a/u=b/v]
v u
⇒ xb = ya
⇒ (x, y) R (a, b)
This shows that R is transitive.
Hence, R is an equivalence relation.
OR
Given a non-empty set X, define the relation R in P(X) as follows:
For A, B ∈ ( ), ( , ) ∈ iff ⊂ . Prove that R is reflexive, transitive and not symmetric.
Ans: Let A  P(X). Then A  A
 (A, A)  R
Hence, R is reflexive.
Let A, B, C,  P(X) such that 1
(A, B), (B, C)  R
 A  B, B,  C
A C
 (A, C)  R
Hence, R is transitive. 2
, X  P(X) such that   X. Hence, (, X)  R.
But, X  
which implies that (X, )  R.
Thus, R is not symmetric.

34. Find the shortest distance between the lines:


 
r  3i  5 j  7k   (i  2 j  k ) and r  i  j  k   (7i  6 j  k )
 
Ans: Given lines are r  3i  5 j  7k   (i  2 j  k ) and r  i  j  k   (7i  6 j  k )

Prepared by: M. S. KumarSwamy, TGT(Maths) Page - 10


-
OR
The equations of motion of a rocket are: = 2 , = −4 , = 4 , where the time t is given in
seconds, and the coordinates of a moving point in km. What is the path of the rocket? At what
distances will the rocket be from the starting point O(0, 0, 0) and from the following line in 10
seconds?
x y z
Ans: Eliminating t between the equations, we obtain the equation of the path   , which
2 4 4
are the equations of the line passing through the origin having direction ratios <2, -4, 4>.
This line is the path of the rocket.
When t = 10 seconds, the rocket will be at the point (20, -40, 40). Hence, the required distance
from the origin at 10 seconds = 20 2  402  402 km  20  3km  60km
The distance of the point (20, -40, 40) from the given line

3 1 2 
35. If A   3 2 3 , find –1. Use –1 to solve the following system of equations:
 2 0 1
3x + 3y + 2z = 1; x + 2y = 4; 2x – 3y – z = 5
Ans:

Prepared by: M. S. KumarSwamy, TGT(Maths) Page - 11


-
SECTION – E(Case Study Based Questions)
Questions 35 to 37 carry 4 marks each.

36. Case-Study 1: Read the following passage and answer the questions given below.

The temperature of a person during an intestinal illness is given by


( ) = −0.1 ² + + 98.6,0 ≤ ≤ 12, m being a constant, where f(x) is the temperature in °F at
x days.
(i) Is the function differentiable in the interval (0, 12)? Justify your answer.
(ii) If 6 is the critical point of the function, then find the value of the constant
(iii) Find the intervals in which the function is strictly increasing/strictly decreasing.
OR
(iii) Find the points of local maximum/local minimum, if any, in the interval (0, 12) as well as
the points of absolute maximum/absolute minimum in the interval [0, 12]. Also, find the
corresponding local maximum/local minimum and the absolute maximum/absolute minimum
values of the function.

Ans: (i) f(x) = – 0.1x2 + mx + 98.6, being a polynomial function, is differentiable everywhere,
hence, differentiable in (0, 12).
(ii) f'(x) = – 0.2x + m
Since, 6 is the critical point,
(iii) f(x) = – 0.1x2 + 1.2x + 98.6
f'(x) = – 0.2x + 1.2 = – 0.2(x – 6)

Prepared by: M. S. KumarSwamy, TGT(Maths) Page - 12


-
In the Interval f’(x) Conclusion
(0, 6) +ve f is strictly increasing in [0, 6]
(6, 12) -ve f is strictly decreasing in [6, 12]

OR
(iii) f(x) = – 0.1x2 + 1.2x + 98.6,
f'(x) = – 0.2x + 1.2, f'(6) = 0,
f''(x) = – 0.2
f''(6) = – 0.2 < 0
Hence, by second derivative test 6 is a point of local maximum. The local maximum value =
f(6) = − 0.1 × 62 + 1.2 × 6 + 98.6 = 102.2
We have f(0) = 98.6, f(6) = 102.2, f(12) = 98.6 6 is the point of absolute maximum and the
absolute maximum value of the function = 102.2.
0 and 12 both are the points of absolute minimum and the absolute minimum value of the
function = 98.6.

37. Case-Study 2: Read the following passage and answer the questions given below.
In an elliptical sport field the authority wants to design a rectangular soccer field with the
x2 y2
maximum possible area. The sport field is given by the graph of 2  2  1
a b

(i) If the length and the breadth of the rectangular field be 2x and 2y respectively, then find the
area function in terms of x.
(ii) Find the critical point of the function.
(iii) Use First derivative Test to find the length 2x and width 2y of the soccer field (in terms of a
and b) that maximize its area.
OR
(iii) Use Second Derivative Test to find the length 2x and width 2y of the soccer field (in terms
of a and b) that maximize its area.
Ans: (i)

Prepared by: M. S. KumarSwamy, TGT(Maths) Page - 13


-
OR

Prepared by: M. S. KumarSwamy, TGT(Maths) Page - 14


-
38. Case-Study 3:
A shopkeeper sells three types of flower seeds A1, A2, A3. They are sold is the form of a mixture,
where the proportions of these seeds are 4 : 4 : 2 respectively. The germination rates of the three
types of seeds are 45%, 60% and 35% respectively.

Based on the above information :


(a) Calculate the probability that a randomly chosen seed will germinate. [2]
(b) Calculate the probability that the seed is of type A2, given that a randomly chosen seed
germinates. [2]
Ans: (a)

(b)

Prepared by: M. S. KumarSwamy, TGT(Maths) Page - 15


-
KENDRIYA VIDYALAYA GACHIBOWLI, GPRA CAMPUS, HYD–32
SAMPLE PAPER TEST 09 FOR BOARD EXAM (2022-23)
(ANSWERS)
SUBJECT: MATHEMATICS (041) MAX. MARKS : 80
CLASS : XII DURATION: 3 HRS
General Instructions:
1. This Question paper contains - five sections A, B, C, D and E. Each section is compulsory.
However, there are internal choices in some questions.
2. Section A has 18 MCQ’s and 02 Assertion-Reason based questions of 1 mark each.
3. Section B has 5 Very Short Answer (VSA)-type questions of 2 marks each.
4. Section C has 6 Short Answer (SA)-type questions of 3 marks each.
5. Section D has 4 Long Answer (LA)-type questions of 5 marks each.
6. Section E has 3 source based/case based/passage based/integrated units of assessment (4
marks each) with sub parts.

SECTION – A
Questions 1 to 20 carry 1 mark each.

1. The vector of the direction of the vector i  2 j  2k that has magnitude 9 is
i  2 j  2k
(a) i  2 j  2k (b) (c) 3(i  2 j  2k ) (d) 9(i  2 j  2k )
3
Ans: (c) 3(i  2 j  2k )

Any vector in the direction of a vector a is given by

a iˆ  2 ˆj  2kˆ iˆ  2 ˆj  2kˆ
  
|a| 12  22  22 3

 a iˆ  2 ˆj  2kˆ
∴ Vector in the direction of a with magnitude 9 is 9   9.  3(i  2 j  2k )
|a| 3

2. If P(A) = 2/5, P(B)= 3/10 and P(A ∩ B) = 1/5, then P(A'/B').P(B'/A') is equal to
(a) 25/42 (b) 5/6 (c) 5/7 (d) 1
Ans: (a) 25/42

Prepared by: M. S. KumarSwamy, TGT(Maths) Page - 1-


3. In answering a question on a multiple-choice test, a student either knows the answer or guesses.
Let 3/4 be the probability that he knows the answer and 1/4 be the probability that he guesses.
Assuming that a student who guesses at the answer will be correct with probability 1/4. What is
the probability that the student knows the answer given that he answered it correctly?
(a) 11/13 (b) 7/13 (c) 12/13 (d) 9/13
Ans: (c) 12/13

 
4. If the position vector a of the point (5, n) is such that | a | = 13, then the value(s) of n can be
(a) ±12 (b) ±8 (c) Only 12 (d) Only 8
Ans:

 1 x 
2

5.  e 
x
2 
dx = ?
1 x 
e x ex e x ex
(a)  C (b) C (c) C (d) C
1  x2 1  x2 (1  x 2 )2 (1  x 2 )2
ex
Ans: (b) C
1  x2

x  3 y 1 z  6 x 5 y  2 z 3
6. Two-line   and   intersect at the point R. The reflection of
1 3 1 7 6 4
R in the xy plane has coordinates
(a) (2, 4, 7) (b) (-2, 4, 7) (c) (2, -4, -7) (d) (2, -4, 7)
Ans: (c) (2, -4, -7)

Prepared by: M. S. KumarSwamy, TGT(Maths) Page - 2-


 1  x  1x
7.   1  x 
x
e dx = ?
1 1
x x
(a) ( x  1)e x
C (b) xe x
C
1 1
x x
(c)  xe x
C (d) ( x  1)e x

1
x
Ans: (b) xe x
C

x2 y2
8. The area of the region bounded by the ellipse   1 is
25 16
(a) 20π² sq. units (b) 25π sq. units (c) 20π sq. units (d) 16π² sq. units
Ans: (c) 20π sq. units
The area of the standard ellipse is given by ; πab. Here, a = 5 and b = 4
Therefore, the area of curve is π(5)(4) = 20π.
   
9. If a and b are unit vectors inclined at an angle θ, then the value of | a  b | is
 
(a) 2 cos (b) 2sin (c) 2cos θ (d) 2sin θ
2 2

Ans: (b) 2sin
2

10. The area enclosed by the circle x2 + y2 = 2 is equal to


(a) 4π² sq units (b) 4π sq units (c) 2π sq units (d) 2√2π sq units
Ans: (c) 2π sq units

2
 d 2 y   dy 
2
 dy 
11. The degree of the differential equation  2      x sin   is
 dx   dx   dx 
(a) 1 (b) 2 (c) 3 (d) not defined
Ans: (d) not defined

12.  sin 3 (2 x  1)dx  ?


1 1 1 1
(a) cos(2 x  1)  cos3 (2 x  1)  C (b)  cos(2 x  1)  cos3 (2 x  1)  C
2 3 2 6

Prepared by: M. S. KumarSwamy, TGT(Maths) Page - 3-


1 4
(c) sin (2 x  1)  C (d) none of these
8
1 1
Ans: (b)  cos(2 x  1)  cos3 (2 x  1)  C
2 6

13. If A and B are invertible matrices, then which of the following is not correct.
(a) adj A = |A|. A–1 (b) det (A–1) = [det (A)]–1
–1 –1 –1
(c) (AB) = B A (d) (A + B)–1 = B–1 + A–1
–1 –1 –1
Ans: (d) (A + B) = B + A

14. Function f(x) = 2x3 – 9x2 + 12x + 29 is monotonically decreasing when


(a) x > 2 (b) 1 < x < 2 (c) x = 2 (d) x > 3
Ans: (b) 1 < x < 2

15. If A is a square matrix of order 3 and |A| = –5, then |adj A| is:
(a) 125 (b) –25 (c) 25 (d) ±25
Ans: (c) 25
We know that, |adj A| = |A|n – 1 where n is the order of the matrix
∴ |adj A| = [5]3 – 1 = (–5)2 = 25

 2 x 6
16. If A =   is a singular matrix, then x :
 1 1 
(a) 3 (b) – 3 (c) 1 (d) – 2
Ans: (b) – 3
|A| = 2x + 6 = 0
⇒x=–3

dy
17. The integrating factor of the differential equation ( x log x)  y  2 log x is
dx
(a) ex (b) log x (c) log(log x) (d) x
Ans: (b) log x

18. The value of tan2 (sec–1 2) + cot2 (cosec–1 3) is


(a) 5 (b) 11 (c) 13 (d) 15
Ans: (b) 11
tan 2 (sec 1 2)  cot 2 (cosec 1 3)
 sec2 (sec 1 2)  1  cosec 2 (cosec 1 3)  1 [ sec 2   tan 2   1; cosec 2  cot 2   1]
 [sec(sec 1 2)]2  1  [cosec(cosec 1 3)]2  1
 22  1  42  1  4  1  9  1  3  8  11

ASSERTION-REASON BASED QUESTIONS


In the following questions, a statement of assertion (A) is followed by a statement of Reason (R).
Choose the correct answer out of the following choices.
(a) Both A and R are true and R is the correct explanation of A.
(b) Both A and R are true but R is not the correct explanation of A.
(c) A is true but R is false.
(d) A is false but R is true.

Prepared by: M. S. KumarSwamy, TGT(Maths) Page - 4-


 x 
19. Assertion (A): If manufacturer can sell x items at a price of Rs.  5   each. The cost price
 100 
x 
of x items is Rs.   500  . Then, the number of items he should sell to earn maximum profit is
5 
240 items.
24 x2
Reason (R): The profit for selling x items is given by x  300 .
5 100
Ans. (c) A is true but R is false.

20. Assertion(A): Determinant of a skew-symmetric matrix of order 3 is zero.


Reason(R): For any matrix A, |A'| = |A| and |-A| = |A|.
Ans. (c) A is true but R is false.
Assertion: Determinant of a skew-symmetric matrix of odd order is zero.
∴ Assertion is true.
Reason: For any matrix A, |A'| = A
and |-A] = |A| [when A is of even order]
and |-A| = -|A| [when A is of odd order]
∴ Reason is false.

SECTION – B
Questions 21 to 25 carry 2 marks each.

2 7 1
21. Prove that tan 1  tan 1  tan 1
11 24 2
 2 7 
2
1 1 7 1
 11  24    x  y 
Ans: LHS  tan  tan  tan    tan 1 x  tan 1 y  tan 1  
11 24 2 7
 1 .    1  x. y  
 11 24 
 48  77   125   125 
      125 1
 tan 1  264   tan 1  264   tan 1  264   tan 1  tan 1  RHS
 1  14   264  14   250  250 2
 264   264   264 

22. Find the values of x and y from the following equation:


x 5   3 4   7 6 
2   
 7 y  3 1 2  15 14 
x 5   3 4   7 6 
Ans: 2    
 7 y  3 1 2  15 14 
2 x 10  3 4  7 6 
   
14 2 y  6  1 2  15 14 
2 x  3 6  7 6
 
 15 2 y  4 15 14 
or 2x + 3 = 7 and 2y – 4 = 14
or 2x = 7 – 3 and 2y = 18
4 18
or x = and y =
2 2
i.e. x = 2 and y = 9.
OR

Prepared by: M. S. KumarSwamy, TGT(Maths) Page - 5-


 3 2  1 0 
If A    and I   2
 , find k so that A = kA – 2I
 4  2   0 1 
Ans: Given than A2 = kA – 2I
 3 2   3 2   3 2  1 0 
    k  2 
 4 2   4 2   4 2  0 1 
 9  8 6  4   3k 2k   2 0 
  
12  8 8  4   4k 2k   0 2 
1 2  3k  2 2 k 
  
 4 4   4k 2k  2
By definition of equality of matrix as the given matrices are equal, their corresponding elements
are equal. Comparing the corresponding elements, we get
3k − 2 = 1  k = 1
−2k = −2  k = 1
4k = 4  k = 1
−4 = −2k − 2  k = 1
23. An urn contains 10 black and 5 white balls. Two balls are drawn from the urn one after the other
without replacement, then find the probability that both drawn balls are black.
Ans: Let E and F denote respectively the events that first and second ball drawn are black. We
have to find P(E ∩ F).

24. Solve (1 + x2) sec2 ydy + 2x tan ydx = 0, given that y = π/4 when x = 1.
Ans:


25. Find the area of parallelogram whose adjacent sides are represented by the vectors a  2i  j  k

and b  3i  j .
Ans:

Prepared by: M. S. KumarSwamy, TGT(Maths) Page - 6-


SECTION – C
Questions 26 to 31 carry 3 marks each.
 ax  1, if x  3
26. Find the relationship between a and b so that the function f defined by f ( x)  
bx  3, if x  3
is continuous at x = 3.
 ax  1, if x  3
Ans. Here, f ( x)  
bx  3, if x  3
lim lim
LHL  
f ( x)  (ax  1)
x3 x  3
Putting x = 3 − h has x → 3− when h → 0
lim lim lim
 
f ( x)  (a(3  h)  1)  (3a  ah  1)  3a  1
x3 h0 h0
lim lim
RHL  
f ( x)  (bx  3)
x3 x  3
Putting x = 3 + h as x → 3+ when h → 0
lim lim lim
 
f ( x)  (b(3  h)  3)  (3b  bh  3)  3b  3
x3 h0 h0
Also, f(3) =3 a +1 [since f(x) = ax + 1]
Since, f(x) is continuous at x = 3.
 LHL = RHL = f(3)
2
 3a  1  3b  3  3a  3b  2  a  b 
3
dy
27. Solve:  ( x  y  1)2
dx
Ans:

OR
Find the general solution of the differential equation: xdy  ydx  x 2  y 2 dx
Ans:

Prepared by: M. S. KumarSwamy, TGT(Maths) Page - 7-


x
28. Evaluate:  1  x 2  x4
dx

Ans:

OR
( x  3), x  2
2
Evaluate  f ( x)dx , if
0
f ( x)  
 ( x  3), x  2
Ans:

Prepared by: M. S. KumarSwamy, TGT(Maths) Page - 8-


29. If y = 3 cos (log x) + 4 sin (log x), show that x2 y2 + xy1 + y = 0
Ans. Given that y = 3 cos (log x) + 4 sin (log x)
Differentiating both sides w.r.t. x, we get
dy d d
 y1  3sin(log x) (log x)  4 cos(log x) (log x)
dx dx dx
1 1 1
 3sin(log x)  4 cos(log x)   3sin(log x)  4 cos(log x) 
x x x
 xy1  3sin(log x )  4 cos(log x)
Again, Differentiating both sides w.r.t. x, we get
d d
xy2  y1.1  3cos(log x) (log x)  4 sin(log x) (log x)
dx dx
1 1 1 y
 3cos(log x )  4 sin(log x )   3cos(log x)  4sin(log x)   
x x x x
 x y2  xy1   y
2

 x 2 y2  xy1  y  0

    
30. Find a unit vector perpendicular to each of the vectors a  b and a  b , where a  3i  2 j  2k

and b  i  2 j  2k
Ans:

OR
Using vectors, prove that the points (2, –1, 3), (3, –5, 1) and (–1, 11, 9) are collinear.
Ans:

Prepared by: M. S. KumarSwamy, TGT(Maths) Page - 9-


31. Find the area of the region {(x, y) : x2 + y2 ≤ 4, x + y ≥ 2}.
Ans:

SECTION – D
Questions 32 to 35 carry 5 marks each.

32. Minimize and maximize Z = 5x + 2y subject to the following constraints:


x – 2y ≤ 2, 3x + 2y ≤ 12, –3x + 2y ≤ 3, x ≥ 0, y ≥ 0
Ans: Here, objective function is Z = 5x + 2y ...(i)
Subject to the constraints:
x – 2y ≤ 2 ...(ii)
3x + 2y ≤ 12 ...(iii)
–3x + 2y ≤ 3 ...(iv)
x ≥ 0, y ≥ 0 ...(v)
For Graph for x – 2y ≤ 2, We draw graph of x – 2y = 2
[By putting x = y = 0 in the equation]
i.e., (0, 0) satisfy (ii) ⇒ feasible region lie origin side of line x – 2y = 2.
For Graph for 3x + 2y ≤ 12, We draw the graph of 3x + 2y =12.
3 × 0 + 2 × 0 ≤ 12 [By putting x = y = 0 in the given equation]
i.e., (0, 0) satisfy (iii) ⇒ feasible region lie origin side of line 3x + 2y = 12.
For Graph for –3x + 2y ≤ 3, We draw the graph of –3x + 2y = 3
–3 × 0 + 2 × 0 ≤ 3 [By putting x = y = 0]
i.e., (0, 0) satisfy (iv) ⇒ feasible region lie origin side of line –3x + 2y = 3.
x ≥ 0, y ≥ 0 ⇒ feasible region is in Ist quadrant.
Now, we get shaded region having corner points O, A, B, C and D as feasible region.

Prepared by: M. S. KumarSwamy, TGT(Maths) Page - 10-


The co-ordinates of O, A, B, C and D are O(0, 0), A(2, 0), B(7/2, 3/4), C(3/2, 15/4) and D(0,
3/2) respectively. Now, we evaluate Z at the corner points.
Corner Points Z = 5x + 2y
O (0, 0) 0
A (2, 0) 10
B (7/2, 3/4) 19
C (3/2, 15/4) 15
D (0, 3/2) 3
Hence, Z is minimum at x = 0, y = 0 and minimum value = 0
also Z is maximum at x = 7/2, y = 3/4 and maximum value = 19.

33. Show that each of the relation R in the set A  {x  Z : 0  x  12} , given by R = {(a, b): |a – b| is
a multiple of 4} is an equivalence relation. Find the set of all elements related to 1.
Ans: A  {x  Z : 0  x  12}  {0,1, 2,3, 4,5, 6, 7,8,9,10,11,12} and
R = {(a, b): |a – b| is a multiple of 4}
For any element a ∈A, we have (a, a) ∈ R ⇒ |a – a| = 0 is a multiple of 4.
∴R is reflexive.
Now, let (a, b) ∈ R ⇒|a – b| is a multiple of 4.
⇒|–(a – b)| is a multiple of 4
⇒|b – a| is a multiple of 4.
⇒ (b, a) ∈ R
∴R is symmetric.
Now, let (a, b), (b, c) ∈ R.
⇒|a – b| is a multiple of 4 and |b – c| is a multiple of 4.
⇒(a – b) is a multiple of 4 and (b – c) is a multiple of 4.
⇒(a – b + b – c) is a multiple of 4
⇒(a – c) is a multiple of 4
⇒|a – c| is a multiple of 4
⇒ (a, c) ∈R
∴ R is transitive.

Prepared by: M. S. KumarSwamy, TGT(Maths) Page - 11-


Hence, R is an equivalence relation.
The set of elements related to 1 is {1, 5, 9} since
|1 – 1| = 0 is a multiple of 4
|5 – 1| = 4 is a multiple of 4
|9 – 1| = 8 is a multiple of 4
OR
x
Show that the function f :R→ {x  R : −1 < x <1} defined by f( x) = , x  R is one-one and
1 | x |
onto function.
x
Ans: It is given that f :R→ {x  R : −1 < x <1} defined by f( x) = ,xR
1 | x |
x y
Suppose, f(x) = f(y), where x, y  R  
1 | x | 1 | y |
It can be observed that if x is positive and y is negative, then we have
x y
  2 xy  x  y
1  x 1 y
Since, x is positive and y is negative, then x > y  x − y > 0

But, 2xy is negative. Then, 2xy ≠ x − y.


Thus, the case of x being positive and y being negative can be ruled out.
Under a similar argument, x being negative and y being positive can also be ruled out. Therefore,
x and y have to be either positive or negative.
x y
When x and y are both positive, we have f ( x)  f ( y )    x  xy  y  xy  x  y
1 x 1 y
x y
When x and y are both negative, we have f ( x)  f ( y )    x  xy  y  xy  x  y
1 x 1 y
Therefore, f is one-one. Now, let y  R such that −1 < y < 1.
y
If y is negative, then there exists x   R such that
1 y
 y  y

 y   1 y  
1 y y
f ( x)  f     y
1 y  1 y   y  1 y  y
1 
1 y 
 1 y 
y
If y is positive, then there exists x   R such that
1 y
 y  y
 
 y   1 y  1 y y
f ( x)  f     y
1 y  1 y  y  1 y  y
1 
1 y 
 1 y 
Therefore, f is onto. Hence, f is one-one and onto.

34. Find the vector equation of the line passing through (1, 2, – 4) and perpendicular to the two
x  8 y  19 z  10 x  15 y  29 z  5
lines:   and  
3 16 7 3 8 5
Ans:

Prepared by: M. S. KumarSwamy, TGT(Maths) Page - 12-


OR
Find the shortest distance between the lines whose vector equations are:
 
r  (i  j )   (2i  j  k ) and r  (2i  j  k )   (3i  5 j  2k )
Ans:

x2
35. Evaluate:  4 dx
x  x 2  12
Ans:

Prepared by: M. S. KumarSwamy, TGT(Maths) Page - 13-


SECTION – E(Case Study Based Questions)
Questions 36 to 38 carry 4 marks each.

36. Case-Study 1:
A gardener wants to construct a rectangular bed of garden in a circular patch of land. He takes
the maximum perimeter of the rectangular region as possible. (Refer to the images given below
for calculations)

(i) Find the perimeter of rectangle in terms of any one side and radius of circle.
(ii) Find critical points to maximize the perimeter of rectangle?
(iii) Check for maximum or minimum value of perimeter at critical point.
OR
(iii) If a rectangle of the maximum perimeter which can be inscribed in a circle of radius 10 cm
is square, then the perimeter of region.
Ans: (i) Let ‘y’ be the breadth and ‘x’ be the length of rectangle and ‘a’ is radius of given circle.
From figure, 4a2 = x2 + y2 ⇒ y2 = 4a2 – x 2
 y  4a 2  x 2


Perimeter (P) = 2x + 2y = 2 x  4a 2  x 2 

(ii) We know that P = 2x + 2y = 2 x  4a 2  x 2 
dP
Critical points to maximize perimeter, 0
dx
dP  1   4a 2  x 2  x 
  2 1  ( 2 x)   0  2    0  4a 2  x 2  x  0
dx  2 4a  x
2 2
  4a  x 
2 2

 4 a 2  x 2  x  4 a 2  x 2  x 2  2 x 2  4a 2  x 2  2 a 2  x   2 a
But x   2a is not possible as length is never negative
 x  2a  y  2a
Hence, critical point is ( 2a, 2a)
dP  x 
(iii) We have,  2 1  
dx  4a 2  x 2 

Prepared by: M. S. KumarSwamy, TGT(Maths) Page - 14-


 2 x   x2 
 4a 2  x 2  ( x)   4 a 2
 x 2
 
2 4a 2  x 2 4a 2  x 2
2
d P
 2  2    2  
dx  4a 2  x 2   4a 2  x 2 
   
 
2
d P  4a  x  x 
2 2 2
 4a 2

 2  2  2 3/ 2 
 2  2 3/2 
 (4a  x )   (4a  x ) 
2 2
dx

 d 2P   4a 2  2
 2   2  2 3/ 2 
 0
 dx   (4 a 2
 2 a )  2 2 a
xa 2
Hence, Perimeter is maximum at a critical point.
OR
From the above results know that x = y = 2a
a = radius
Here, x = y = 10 2
Perimeter = P = 4 x side = 40 2 cm

37. Case-Study 2:
Three car dealers, say A, B and C, deals in three types of cars, namely Hatchback cars, Sedan
cars, SUV cars. The sales figure of 2019 and 2020 showed that dealer A sold 120 Hatchback, 50
Sedan, 10 SUV cars in 2019 and 300 Hatchback, 150 Sedan, 20 SUV cars in 2020; dealer B sold
100 Hatchback, 30 Sedan, 5 SUV cars in 2019 and 200 Hatchback, 50 Sedan, 6 SUV cars in
2020; dealer C sold 90 Hatchback, 40 Sedan, 2 SUV cars in 2019 and 100 Hatchback, 60 Sedan,
5 SUV cars in 2020.

(i) Write the matrix summarizing sales data of 2019 and 2020.
(ii) Find the matrix summarizing sales data of 2020.
(iii) Find the total number of cars sold in two given years, by each dealer?
OR
(iii) If each dealer receives a profit of = 50000 on sale of a Hatchback, 100000 on sale of a Sedan
and 2200000 on sale of an SUV, then find the amount of profit received in the year 2020 by each
dealer.
Hatchback Sedan SUV
A 120 50 10 
(i)
B 100 30 5 
 
C  90 40 2 

Prepared by: M. S. KumarSwamy, TGT(Maths) Page - 15-


In 2019, dealer A sold 120 Hatchbacks, 50 Sedans and 10 SUV; dealer B sold 100 Hatchbacks,
30 Sedans and 5 SUVs and dealer C sold 90 Hatchbacks, 40 Sedans and 2 SUVs.
Hatchback Sedan SUV
A 120 50 10 
∴ Required matrix, say P, is given by P = 100 30 5 
B
 
C  90 40 2 
In 2020, dealer A sold 300 Hatchbacks, 150 Sedans, 20 SUVs dealer B sold 200
Hatchbacks, 50 sedans, 6 SUVs dealer C sold 100 Hatchbacks, 60 sedans, 5 SUVs.
Hatchback Sedan SUV
A  300 150 20
∴ Required matrix, say Q, is given by Q =  200 50 6 
B
 
C 100 60 5 
Hatchback Sedan SUV
A  300 150 20
(ii)
B  200 50 6 
 
C 100 60 5 
In 2020, dealer A sold 300 Hatchbacks, 150 Sedans, 20 SUVs dealer B sold 200
Hatchbacks, 50 sedans, 6 SUVs dealer C sold 100 Hatchbacks, 60 sedans, 5 SUVs.
Hatchback Sedan SUV
A  300 150 20
∴ Required matrix, say Q, is given by Q =  200 50 6 
B
 
C 100 60 5 
(iii) Total number of cars sold in two given years, by each dealer, is given by
Hatchback Sedan SUV Hatchback Sedan SUV
A 120  300 50  150 10  20  A  420 200 30
B 100  200 30  50 56  B  300 80 11
P+Q= =
   
C  90  100 40  60 2  5  C 190 100 7 
OR
The amount of profit in 2020 received by each dealer is given by the matrix
Hatchback Sedan SUV
A  300 150 20   50000 
B  200 50 6  100000 
  
C 100 60 5   200000 
A 15000000  15000000  4000000  A 34000000
 B  10000000  5000000  1200000   B 16200000 
   
C  5000000  6000000  1000000  C 12000000 

38. Case-Study 3:
In a school, teacher asks a question to three students Kanabh, Raj and Shubi respectively. The
probability of solving the question by Kanabh, Raj and Shubi are 40%, 15% and 50%
respectively. The probability of making error by Kanabh, Raj and Shubi are 1.5%, 2% and 2.5%.

Prepared by: M. S. KumarSwamy, TGT(Maths) Page - 16-


Based on the given information, answer the following questions:
(i) Find the probability that Shubi solved the question and committed an error.
(ii) Find the total probability of committing an error is solving the question.
Ans: According to given case,
Let E1 = Solved by Kanabh, E2 = Solved by Raj and E3 = Solved in Shubi
Also let A = question has some error.
∴ P(E1) = 40/100, P(E2) = 15/100, P(E3) = 50/100
P(A/E1) = 1.5/100, P(A/E2) = 2/100, P(A/E3) = 2.5/100,
(i) The probability that Shubi solved the question and committed an error i.e.,

(ii) The total probability of committing an error is solving the question

Prepared by: M. S. KumarSwamy, TGT(Maths) Page - 17-


KENDRIYA VIDYALAYA GACHIBOWLI, GPRA CAMPUS, HYD–32
SAMPLE PAPER TEST 10 FOR BOARD EXAM (2022-23)
(ANSWERS)
SUBJECT: MATHEMATICS (041) MAX. MARKS : 80
CLASS : XII DURATION: 3 HRS
General Instructions:
1. This Question paper contains - five sections A, B, C, D and E. Each section is compulsory.
However, there are internal choices in some questions.
2. Section A has 18 MCQ’s and 02 Assertion-Reason based questions of 1 mark each.
3. Section B has 5 Very Short Answer (VSA)-type questions of 2 marks each.
4. Section C has 6 Short Answer (SA)-type questions of 3 marks each.
5. Section D has 4 Long Answer (LA)-type questions of 5 marks each.
6. Section E has 3 source based/case based/passage based/integrated units of assessment (4
marks each) with sub parts.

SECTION – A
Questions 1 to 20 carry 1 mark each.

1. The angle between a line with direction ratios 2 : 2 : 1 and a line joining (3, 1, 4) to (7, 2, 12)
2  2 3
(a) cos1   (b) tan 1    (c) cos1   (d) none of these
3  3 2
2
Ans: (a) cos1  
3
2. The angle between two lines having direction ratios 1, 1, 2 and (√3 – 1), (-√3 – 1), 4 is
   
(a) (b) (c) (d)
4 3 6 2

Ans: (b)
3

3. Two numbers are selected at random from integers 1 through 9. If the sum is even, what is the
probability that both numbers are odd?
(a) 5/8 (b) 1/6 (c) 4/9 (d) 2/3
Ans: (a) 5/8
Total outcome = 5C1 x 4C1) (Both the numbers are odd) + 4C1 x 3C1 (Both the numbers are even)
= 32
Number of favourable outcomes = 5C1 x 4C1 =20
Thus, the probability that both numbers are odd will be = 20/32 = 5/8

e x (1  x )
4.  cos 2 ( xe x )dx is equal to
(a) tan(xex) + C (b) cot (xex) + C (c) cot (ex) + C (d) tan[ex (1 + x)] + C
Ans: (a) tan(xex) + C

Prepared by: M. S. KumarSwamy, TGT(Maths) Page - 1-


5.  4  x 2 dx = ?
x x x
(a) 4  x 2  2 sin 1  C (b) x 4  x 2  2 sin 1 C
2 2 2
x x
(c) 4  x 2  2sin 1  C (d) none of these
2 2
x x
Ans: (a) 4  x 2  2 sin 1  C
2 2

6. If P(A) = 1/4, P(B) = 1/3 and P(A ∩ B) = 1/5, then P ( B / A) =?


(a) 11/15 (b) 11/45 (c) 37/45 (d) 37/60
Ans: (c) 37/45

7. The area bounded by the curves y = |x – 1| and y = 1 is given by


(a) 1 (b) 1/2 (c) 2 (d) none of these
Ans: (a) 1

8. The direction ratios of two lines are 3, 2, -6 and 1, 2, 2 respectively. The acute angle between
these lines is
5  8  5  3 
(a) cos1   (b) cos1   (c) cos1   (d) cos1  
 18   21   21   20 
 5
Ans: (c) cos1  
 21 
   
9. Let a and b be two unit vectors and θ is the angle between them. Then a  b is unit vector if θ
is
(a) π/4 (b) π/3 (c) π/2 (d) 2π/3
Ans: (d) 2π/3

3
d 2 y  dy 
10. The degree and order respectively of the differential equation x   x  y
2

 dx 
2
dx
(a) 1, 2 (b) 1, 1 (c) 2, 1 (d) 2, 2
Ans: (a) 1, 2

x2 y2
11. The area of the region bounded by the ellipse   1 is
25 16
(a) 20π² sq. units (b) 25π sq. units (c) 20π sq. units (d) 16π² sq. units
Ans: (c) 20π sq. units
The area of the standard ellipse is given by ; πab. Here, a = 5 and b = 4
Therefore, the area of curve is π(5)(4) = 20π.
1
12. If  (3x 2  2 x  k )dx  0 , then find the value of k.
0

(a) 1 (b) 2 (c) -2 (d) 3


Ans: (c) -2

Prepared by: M. S. KumarSwamy, TGT(Maths) Page - 2-


13. If the set A contains 5 elements and the set B contains 6 elements, then the number of one-one
and onto mapping from A to B is
(a) 720 (b) 120 (c) 0 (d) none of these
Ans: (c) 0
Number of elements in set A = n(A) = 5
and Number of elements in set B = n(B) = 6
As, the number of bijection from A into B can only be possible when provided n(A) ≥ n(B).
But here, we can see, n(A)< n(B)
So, the number of bijection i.e. one-one and onto mappings from A to B = 0

14. If A is an invertible matrix of order 3 and |A| = 5, then |adj A| =


(a) 5 (b) 125 (c) 25 (d) none of these
Ans: (c) 25

15. If A, then A–1 exist if


(a) λ = 2 (b) λ ≠ 2 (c) λ ≠ –2 (d) None of these
Ans: (d) None of these

 cos   sin  
16. If   , and A + A’ = I, if the value of α is
 sin  cos  
(a) π/6 (b) π/3 (c) 3π/2 (d) π
Ans: (b) π/3

17. Domain of cos–1x is


(a) [-1, 0] (b) [0, 1] (c) [-1, 1] (d) None of these
Ans: (c) [-1, 1]

18. What is the equation of a curve passing through (0, 1) and whose differential equation is given
by dy = y tan x dx?
(a) y =sec x (b) y=sin x (c) y = cosec x (d) y=cos x
Ans: (a) y =sec x

ASSERTION-REASON BASED QUESTIONS


In the following questions, a statement of assertion (A) is followed by a statement of Reason (R).
Choose the correct answer out of the following choices.
(a) Both A and R are true and R is the correct explanation of A.
(b) Both A and R are true but R is not the correct explanation of A.
(c) A is true but R is false.
(d) A is false but R is true.

19. Assertion (A): The function f(x) = x² - 4x +6 is strictly increasing in the interval (2, ∞).
Reason (R): The function f(x) = x² - 4x + 6 is strictly decreasing in the interval (-∞, 2).
Ans. (b) Both A and R are true but R is not the correct explanation of A.

1 2
20. Assertion(A): The matrix A =   is Singular.
 4 8
Reason(R): A square matrix A is said to be singular, if |A| = 0.
Ans. (a) Both A and R are true and R is the correct explanation of A.

Prepared by: M. S. KumarSwamy, TGT(Maths) Page - 3-


SECTION – B
Questions 21 to 25 carry 2 marks each.

21. Find the general solution of the following differential equation:


ex tan y dx + (1 – ex) sec2 y dy = 0
Ans:

5 2 3 6 
22. Find X and Y, if X + Y =   and X – Y =  .
0 9  0 1
5 2 3 6 
Ans: ( X  Y )  ( X  Y )   
0 9   0 1
8 8 1 8 8  4 4
 2X    X   X  
0 8 2 0 8 0 4
5 2 3 6 
Now, ( X  Y )  ( X  Y )   
0 9   0 1
2 4  1  2 4 
 2Y    X  
0 10  2  0 10 
1 2 
Y  
0 5 
OR

cos x  sin x 0 
If F ( x )   sin x cos x 0  , show that F(x) F(y) = F(x + y).
 0 0 1 
 cos x  sin x 0   cos y  sin y 0
Ans: LHS  F ( x) F ( y )   sin x cos x 0   sin y cos y 0 
 0 0 1   0 0 1 
 cos x cos y  sin x sin y  sin y cos x  sin x cos y 0
 sin x cos y  cos x sin y  sin x sin y  cos x cos y 0
 0 0 1 
 cos( x  y )  sin( x  y ) 0 
  sin( x  y ) cos( x  y ) 0   F ( x  y )  RHS
 0 0 1 

23. Find the area of a parallelogram ABCD whose side AB and the diagonal AC are given by the
vectors 3i  j  4k and 4i  5k respectively.
Ans:

Prepared by: M. S. KumarSwamy, TGT(Maths) Page - 4-


1 1
24. Find the value of cos 1  2 sin 1 .
2 2
Ans:

25. In a bulb factory, machines A, B and C manufacture 60%, 30% and 10% bulbs respectively. Out
of these bulbs 1%, 2% and 3% of the bulbs produced respectively by A , B and C are found to be
defective. A bulb is picked up at random from the total production and found to be defective.
Find the probability that this bulb was produced by the machine A.
Ans: Let A : bulb manufactured from machine A
B : bulb Manufactured from machine B
C : bulb Manufactured from machine C
D : Defective bulb
We want to find P(B/AD) i. e. probability of selected defective bulb is from machine A.
Therefore, by Baye's theorm, we have,
 B  P( A).P( D / A)
P 
 AD  P( A).P( D / A)  P( B).P( D / B )  P (C ).P ( D / C )
60 1

100 100 60 60 2
   
60

1

30

2

10

3 60  60  30 150 5
100 100 100 100 100 100

SECTION – C
Questions 26 to 31 carry 3 marks each.

26. If i  j  k , 2i  5 j , 3i  2 j  3k and i  6 j  k are the position vectors of points A, B, C and D
respectively, then find the angle between AB and CD . Deduce that AB and CD are collinear.
Ans: Note that if θ is the angle between AB and CD , then θ is also the angle between AB and
CD . Now AB = Position vector of B – Position vector of A
= (2i  5 j )  (i  j  k )  i  4 j  k

Therefore, | AB | 1  16  1  18  3 2
 
Similarly, CD  2i  8 j  2k | CD | 4  64  4  72  6 2

Prepared by: M. S. KumarSwamy, TGT(Maths) Page - 5-


 
AB.CD 1(2)  4(8)  (1)(2)
Thus, cos       1
| AB || CD | (3 2)(6 2)
Since 0 ≤ θ ≤ π, it follows that θ = π. This shows that AB and CD are collinear.
OR
     
Let a , b and c be three vectors such that | a | 3,| b | 4,| c | 5 and each one of them being
  
perpendicular to the sum of the other two, find | a  b  c | .
Ans:
Given that each one of them being perpendicular to the sum of the other two.
        
Therefore, a.(b  c )  0, b.(c  a)  0, c.(a  b)  0
           
Now, | a  b  c |2  (a  b  c )2  (a  b  c).(a  b  c )
            
 a.a  a.(b  c)  b.b  b.(c  a )  c.(a  b)  c.c
  
| a | 2  | b | 2  | c | 2
 9  16  25  50
  
Therefore, | a  b  c | 50  5 2
 /3
sin x  cos x
27. Evaluate: 
 /6 sin 2 x
dx

Ans:

dy
28. Find the particular solution of the differential equation:  y cot x  4 x cos ecx ( x  0)
dx
given that y = 0 when x = π/4
Ans:

Prepared by: M. S. KumarSwamy, TGT(Maths) Page - 6-


OR
Solve the differential equation: ye dx  ( xe  y 2 )dy
x/ y x/ y
( y  0)
Ans:

1
29. Evaluate:  9x  6x  5 dx
2

Ans:

OR
 1  sin x 
Evaluate:  e x  dx
 1  cos x 
Ans:

Prepared by: M. S. KumarSwamy, TGT(Maths) Page - 7-


 k cos x 
   2 x , if x  2
30. Find the values of k so that the function f f ( x)   is continuous at point
 3, if x  
 2
x  /2
 k cos x 
   2 x , if x  2
Ans. Here, f ( x)  
 3, if x  
 2
lim lim
k cos x
LHL   f ( x) 
 
   2x
x x
2 2

 
Putting x = − h as x → when h → 0
2 2
 
lim k cos   h 

lim 2   lim ksinh  k  lim sinh  k  1  k
 f ( x)  h  0

x   h  0 2h 2 h0 h 2 2
2   2  h
2 
  
Since f(x) is continuous at x  , therefore LHL = f  
2 2
  k
Also, f   = 3   3  k  6
2 2

31. Using the method of integration find the area bounded by the curve |x| + |y| = 1 .
Ans:

SECTION – D
Questions 32 to 35 carry 5 marks each.

32. Maximise Z = 8x + 9y subject to the constraints given below :


2x + 3y ≤ 6; 3x – 2y ≤ 6; y ≤ 1; x, y ≥ 0
Ans: For graph of 2x + 3y ≤ 6
We draw the graph of 2x + 3y = 6
2 × 0 + 3 × 0 ≤ 6 ⇒ (0,0) satisfy the constraints.
Hence, feasible region lie towards origin side of line.
For graph of 3x – 2y ≤ 6
We draw the graph of line 3x – 2y = 6.

Prepared by: M. S. KumarSwamy, TGT(Maths) Page - 8-


3 × 0 – 2 × 0 ≤ 6 ⇒ Origin (0, 0) satisfy 3x – 2y = 6.
Hence, feasible region lie towards origin side of line.

For graph of y ≤ 1
We draw the graph of line y = 1, which is parallel to x-axis and meet y-axis at 1.
0 ≤ 1 ⇒ feasible region lie towards origin side of y = 1.
Also, x ≥ 0, y ≥ 0 says feasible region is in Ist quadrant.
Therefore, OABCDO is the required feasible region, having corner point O(0, 0), A(0, 1)
Here, feasible region is bounded. Now the value of objective function Z = 8x + 9y is obtained as.
Corner Points Z = 8x + 9y
O (0, 0) 0
A (0, 1) 9
B (3/2, 1) 21
C (30/13, 6/13) 22.6
D(2, 0) 16

Z is maximum when x = 30/13 and y = 6/13.

33. Show that the relation R in the set A = {1, 2, 3, 4, 5} given by R = {(a, b) : |a – b| is even}, is an
equivalence relation. Show that all the elements of {1, 3, 5} are related to each other and all the
elements of {2, 4} are related to each other. But no element of {1, 3, 5} is related to any element
of 2, 4}.
Ans: Given that A = {1, 2, 3, 4, 5} and R = {(a, b) : |a – b| is even}
It is clear that for any element a ∈A, we have (which is even).
∴R is reflexive.
Let (a, b) ∈ R.
⇒ |a – b| is even ⇒(a – b) is even ⇒– (a – b) is even
⇒(b – a) is even ⇒ |b – a| is even ⇒(b, a) ∈ R
∴R is symmetric.
Now, let (a, b) ∈ R and (b, c) ∈ R.

Prepared by: M. S. KumarSwamy, TGT(Maths) Page - 9-


⇒ |a – b| is even and |b – c| is even ⇒ (a – b) is even and (b – c) is even
⇒ (a – c) = (a – b) + (b – c) is even (Since, sum of two even integers is even)
⇒ |a – c| is even ⇒ (a, c) ∈ R
∴R is transitive.
Hence, R is an equivalence relation.
Now, all elements of the set {1, 2, 3} are related to each other as all the elements of this subset
are odd. Thus, the modulus of the difference between any two elements will be even.
Similarly, all elements of the set {2, 4} are related to each other as all the elements of this subset
are even.
Also, no element of the subset {1, 3, 5} can be related to any element of {2, 4} as all elements of
{1, 3, 5} are odd and all elements of {2, 4} are even. Thus, the modulus of the difference
between the two elements (from each of these two subsets) will not be even.
OR
 x2
Let A = R − {3} and B = R − {1} . Prove that the function f : A→ B defined by f(x) =   is f
 x 3 
one-one and onto ? Justify your answer.
 x2
Ans: Here, A = R − { 3), B = R − {1} and f : A→ B is defined as f(x) =  
 x 3 
Let x, y ∈A such that f(x) = f(y)
x2 y2
   ( x  2)( y  3)  ( y  2)( x  3)
x3 y 3
 xy  3x  2 y  6  xy  3 y  2 x  6
 3 x  2 y  3 y  2 x
 3x  2 x  3 y  2 y  x  y
Therefore, f is one- one. Let y ∈B = R −{1} . Then, y ≠ 1
The function f is onto if there exists x ∈ A such that f(x) = y.
Now, f(x) = y
x2
  y  x  2  xy  3 y
x3
2  3y
 x (1  y )  3 y  2 x  A [ y  1]
1 y
2  3y
Thus, for any y ∈B, there exists  A such that
1 y
 2  3y 
2
 2  3 y   1  y  2  3y  2  2 y  y
f    y
 1  y   2  3 y   3 2  3 y  3  3 y 1
 1 y 
 
Therefore, f is onto. Hence, function f is one-one and onto.

34. Find the vector equation of the line through the point (1, 2, –4) and perpendicular to the two lines
 
r  (8i  19 j  10k )   (3i  16 j  7 k ) and r  (15i  29 j  5k )   (3i  8 j  5k )
Ans:

Prepared by: M. S. KumarSwamy, TGT(Maths) Page - 10-


OR

Find the shortest distance between the lines r  (4i  j )   (i  2 j  3k ) and

r  (i  j  2k )   (2i  4 j  5k )
Ans:

dy
35. Find if yx + xy + xx = ab.
dx
Ans: Given that yx + xy + xx = ab
Putting u = yx, v = xy and w = xx, we get u + v + w = ab
du dv dw
Therefore,   0 ------------------ (1)
dx dx dx
Now, u = yx. Taking logarithm on both sides, we have log u = x log y
Differentiating both sides w.r.t. x, we have
1 du d d 1 dy
 x (log y )  log y ( x )  x .  log y.1
u dx dx dx y dx
du  x dy   x dy 
  u  log y   y x   log y  -------------------- (2)
dx  y dx   y dx 
y
Also v = x
Taking logarithm on both sides, we have log v = y log x

Prepared by: M. S. KumarSwamy, TGT(Maths) Page - 11-


Differentiating both sides w.r.t. x, we have
1 dv d dy 1 dy
 y (log x)  log x  y  log x
v dx dx dx x dx
dv y dy  y dy 
  v   log x   x y   log x  ------------------ (3)
dx x dx  x dx 
Again w = xx
Taking logarithm on both sides, we have log w = x log x.
Differentiating both sides w.r.t. x, we have
1 dw d d 1
 x (log x )  log x ( x )  x  log x.1
w dx dx dx x
dw
  w 1  log x   x x 1  log x  -------------------- (4)
dx
From (1), (2), (3), (4), we have
 x dy  y dy 
yx   log y   x y   log x   x x 1  log x   0
 y dx  x dx 
dy
( x. y x 1  x y .log x)   x x 1  log x   y. x y 1  y x log y
dx
dy   x 1  log x   y.x  y log y 
x y 1 x

 
dx x. y x1  x y .log x

SECTION – E(Case Study Based Questions)


Questions 36 to 38 carry 4 marks each.

36. Case-Study 1:
On the request of villagers, a construction agency designs a tank with the help of an architect.
Tank consists of a rectangular base with rectangular sides, open at the top so that its depth is 2 m
and volume is 8 m3 as shown below. The construction of the tank costs 70 per sq. metre for the
base and Rs. 45 per square metre for sides.

(i) Express making cost C in terms of length of rectangle base.


(ii) If x and y represent the length and breadth of its rectangular base, then find the relation
between the variables.
(iii) Find the value of x so that the cost of construction is minimum.
OR
(iii) Verify by second derivative test that cost is minimum at a critical point.
Ans: (i) Since 'C' is cost of making tank
∴ C = 70xy + 45 х 2(2x + 2y)
⇒ C = 70xy + 90(2x + 2y)
⇒ C = 70xy+180(x + y) [∵ 2xy = 8 ⇒ y = 8/2x ⇒ y = 4/x]
4  4
 C  70 x   180  x  
x  x
 4
 C  280  180  x  
 x

Prepared by: M. S. KumarSwamy, TGT(Maths) Page - 12-


(ii) Volume of tank = length x breadth x height (Depth)
⇒ 8 = xy х 2
⇒ 2xy = 8 ⇒ xy = 4
dC   1 
(ii) For maximum or minimum,  0  180 1  4   2    0
dx   x 
 4 4
 180  1  2   0  1  2  0
 x  x
4
 2  1  x 2  4  x  2
x
⇒ x = 2 (length can never be negative)
OR
8
2
d C
Now, 2  180  3 
dx x 

 d 2C  8
Now,  2   180    180   ve
 dx  x  2 8
Hence, to minimize C, x = 2m

37. Case-Study 2:
Two farmers Ankit and Girish cultivate only three varieties of pulses namely Urad, Masoor and
Mung. The sale (in Rs.) of these varieties of pulses by both the farmers in the month of
September and October are given by the following matrices A and B.

September sales (in Rs.)


 Urad Masoor Mung 
A =  10000 20000 30000  Ankit
 50000 30000 10000  Girish
 
October sales (in Rs.)
 Urad Masoor Mung 
B =  5000 10000

6000  Ankit
 20000 30000 10000  Girish
 
(i) Find the combined sales of Masoor in September and October, for farmer Girish. [1]
(ii) Find the combined sales of Urad in September and October, for farmer Ankit. [1]
(iii) Find a decrease in sales from September to October. [2]
OR
(iii) If both the farmers receive 2% profit on gross sales, then compute the profit for each farmer
and for each variety sold in October. [2]
Ans:

Prepared by: M. S. KumarSwamy, TGT(Maths) Page - 13-


OR

38. Case-Study 3:
Final exams are approaching, so Mr. Kumar decided to check the preparation of the few weak
students in the class. He chooses four students A, B, C and D then a problem in mathematics is
given to those four students A, B, C, D. Their chances of solving the problem, respectively, are
1 1 1 2
, , and .
3 4 5 3

Based on the given information answer the following questions. What is the probability that:
(i) the problem will be solved?
(ii) at most one of them solve the problem?
Ans: Let E be the event = A solves the problem,

Prepared by: M. S. KumarSwamy, TGT(Maths) Page - 14-


F be the event = B solves the problem,
G be the event = C solves the problem,
H be the event = D solves the problem,

Prepared by: M. S. KumarSwamy, TGT(Maths) Page - 15-


KENDRIYA VIDYALAYA GACHIBOWLI, GPRA CAMPUS, HYD–32
SAMPLE PAPER TEST 11 FOR BOARD EXAM (2022-23)
(ANSWERS)
SUBJECT: MATHEMATICS (041) MAX. MARKS : 80
CLASS : XII DURATION: 3 HRS
General Instructions:
1. This Question paper contains - five sections A, B, C, D and E. Each section is compulsory.
However, there are internal choices in some questions.
2. Section A has 18 MCQ’s and 02 Assertion-Reason based questions of 1 mark each.
3. Section B has 5 Very Short Answer (VSA)-type questions of 2 marks each.
4. Section C has 6 Short Answer (SA)-type questions of 3 marks each.
5. Section D has 4 Long Answer (LA)-type questions of 5 marks each.
6. Section E has 3 source based/case based/passage based/integrated units of assessment (4
marks each) with sub parts.

SECTION – A
Questions 1 to 20 carry 1 mark each.
1. The value of the expression ⃗ × ⃗ + ⃗ . ⃗ is
(a) ⃗ . ⃗ ( ) | ⃗ |. ⃗ ( ) | ⃗| ⃗ ( ) ( ⃗ . )⃗
Ans: ( ) | ⃗ | ⃗

 2 x  1;  if  x  2

2. For what value of k the function f ( x)   k , x  2 is continuous at x = 2 ,
 3x  1; x2

(a) Any real value (b) No real value (c) 5 (d) 1/5
Ans: (c) 5

 a b 
3. If A =   and A2 = I, then
 c a
2
(a) a + bc – 1 = 0 (b) 1 – a2 + b c = 0 (c) a2 + bc +1 = 0 (d) a2 – bc +1 = 0
2
Ans: (a) a + bc – 1 = 0

dx
4. The Integrating factor of the differential equation (1  y 2 )  yx  ay is
dy
(a) (b) (c) (d)
Ans: (d)

5. If A is a square matrix of order 3 such that |A| = - 5 , then value of | − | is


(a) 125 (b) – 125 (c) 25 (d) – 25
Ans: (d) – 25

6. If ( ) = ,then (x) is
(a) +C (b) +C (c) +C (d) +C
Ans: (a) +C

Prepared by: M. S. KumarSwamy, TGT(Maths) Page - 1-


4
d  dy  
   0 is
dx  dx  
7. The sum of the order and the degree of the differential equation

(a) 1 (b) 2 (c) 3 (d) 4


Ans: (c) 3

8. The value of (i  j ).k  ( j  k ).i  (k  i).j is


(a) 0 (b) –1 (c) 1 (d) 3
Ans: (d) 3

9. Corner points of the feasible region for an LPP are (0, 3), (1,1) and (3,0). Let Z = px + qy, where
p, q > 0, be the objective function. The condition on p and q so that the minimum of Z occurs at
(3,0) and (1,1) is
q
(a) p = q (b) p  (c) p = 3q (d) p=q
2
q
Ans: (b) p 
2

dx
10.  9x  4 x2
equals

1  9x  8  1 1  8x  9 
(a) sin 1  C (b) sin  C
9  8  2  9 
1  9x  8  1  9x  8 
(c) sin 1  C (d) sin 1  C
3  8  2  8 
1  8x  9 
Ans: (b) sin 1  C
2  9 

11. If A is a 3 x 3 matrix and |A| = - 2 then value of |A(adjA)| is


(a) -2 (b) 2 (c) -8 (d) 8
Ans: (c) -8

12. Corner points of the feasible region for an LPP are (0, 2), (3, 0), (6, 0), (6, 8) and (0, 5).Let F =
4x + 6y be the objective function. The Minimum value of F occurs at
(a) (0, 2) only (b) (3, 0) only
(c) the mid point of the line segment joining the points (0, 2) and (3, 0) only
(d) any point on the line segment joining the points (0, 2) and (3, 0).
Ans: (d) any point on the line segment joining the points (0, 2) and (3, 0).

x 2 6 2
13. If  , then x is equal to
18 x 18 6
(a) 6 (b) ± 6 (c) -6 (d) 0
Ans: (b) ± 6

14. If A is a square matrix of order 3, such that A(adjA) = 10 , then | | is equal to


(a) 1 (b) 10 (c) 100 (d) 101
Ans: (c) 100

15. Given two independent events A and B such that P(A) =0.3, P(B) = 0.6 and P( ’ ) is
(a) 0.42 (b) 0.18 (c) 0.28 (d) 0.12
Ans: (a) 0.42

Prepared by: M. S. KumarSwamy, TGT(Maths) Page - 2-


16. If y = 5e7x + 6e-7x,show that is equal to
(a) 7y (b) 6 (c) 49y (d) 36y
Ans: (c) 49y

17. The projection of ⃗ on ⃗, if ⃗. ⃗ =8 and ⃗ =2 ̂ + 6 ̂ + 3 .


8 2 2 4
(a) (b) (c) (d)
7 3 9 5
8
Ans: (a)
7

18. If the direction cosines of a line are k, k, k then


(a) k > 0 (b) 0 < k< 1 (c) k = 1 (d) k= or k = -
√ √
Ans: (d) k= or k = -
√ √

ASSERTION-REASON BASED QUESTIONS


In the following questions, a statement of assertion (A) is followed by a statement of Reason (R).
Choose the correct answer out of the following choices.
(a) Both A and R are true and R is the correct explanation of A.
(b) Both A and R are true but R is not the correct explanation of A.
(c) A is true but R is false.
(d) A is false but R is true.
 
19. Assertion(A) : The pair of lines given by r  i  j   (2i  k ) and r  2i  k   (i  j  k )
intersect .
Reason(R) : Two lines intersect each other, if they are not parallel and shortest distance = 0.
Ans: (a) Both A and R are true and R is the correct explanation of A.

 2  1  1  
20. Assertion (A) : The value of expression sec 1  1
  tan 1  sin  2  is 4
 3  
  
Reason (R) : Principal value branch of sin 1 x is   ,  and that of sec1 x is [0,  ]  { / 2}
 2 2
Ans: (d) A is false but R is true.

SECTION – B
Questions 21 to 25 carry 2 marks each.

1  x2  1
21. Write the simplest form of tan 1 ,x  0
x
Ans: Put x  tan     tan 1 x
1  x2  1 1  tan 2   1
tan 1  tan 1
x tan 
 sec   1  1  1  cos      1
 tan 1    tan 
1
  tan tan     tan x
1

 tan    sin   2 2 2


OR
n  1,  if  nis
   odd  
Show that f ∶ N → N, given by  f (n)   is a bijection.
n  1,  if  nis
   even  
Ans: Injective test:
Case I: If n is odd:

Prepared by: M. S. KumarSwamy, TGT(Maths) Page - 3-


Let x1 , x2  N such that f ( x1 )  f ( x2 )
f ( x1 )  x1  1, f ( x2 )  x2  1
As f ( x1 )  f ( x2 )  x1  1  x2  1  x1  x2
Case II: If n is even,
Let x1 , x2  N such that f ( x1 )  f ( x2 )
f ( x1 )  x1  1, f ( x2 )  x2  1
As f ( x1 )  f ( x2 )  x1  1  x2  1  x1  x2
∴ f is injective.
Surjection test:
Case I: If n is odd,
As, for every n ∈ N, there exists y = n − 1 in N such that,
f(y) = f(n − 1) = n – 1 + 1 = n
Case II: If n is even,
As, for every n ∈ N, there exists y = n + 1 in N such that,
f(y) = f(n + 1) = n + 1 – 1 = n
∴ f is surjective.
Since f is injective and surjective then, it is a bijection.

22. An edge of a variable cube is increasing at the rate of 5cm per second. How fast is the volume
increasing when the side is 15 cm.
Ans: Let be the edge of the cube and be the volume of the cube at any time . Given, =
5 / , = 15
Since we know the volume of cube = ( ) i.e., = .
⇒ = 3 ⋅

= 3 ⋅ ( 15) × 5 = 3375 /

     
23. If a  5i  j  7k b  i  j   k , then find the value of so that the vectors a  b a b
are orthogonal.
   
Ans: (a  b) and  (a  b) are orthogonal  
     
 ( a  b )  ( a  b )  0  | a |2  | b |2  0
 
 | a || b | 25  1  49  1  1   2
 75   2  2   2  2  75   2  75  2  73
    73
OR
Find the direction ratio and direction cosines of a line parallel to the line whose equations are
6 − 2 = 3 + 1 = 2 − 4
Ans: The equations of the line are 6x – 2 = 3y + 1 = 2z – 4, which when written in standard
 2  1
symmetric form, will be 6  x    3  y    2(z  2)
 6  3
 2  1 2 1
6  x   3 y   x y
 
6
 
3  2( z  2)
  6 3  z2
6 6 6 1 2 3
So, d.r.’s of line are: 1, 2, 3
 1 2 3 
and the required direction cosines are  , , 
 1 2 3 1 2 3 1 2 3 
2 2 2 2 2 2 2 2 3

Prepared by: M. S. KumarSwamy, TGT(Maths) Page - 4-


 1 2 3 
 , , 
 14 14 14 

dy sin 2 (a  y)
24. If x sin(a  y )  sin a cos(a  y )  0 , then pr ove that 
dx sin a
Ans: Given ( + )+ ( + ) = 0
⋅ ( + )
⇒ = − ⇒ = − ⋅ ( + )
( + )
Differentiating with respect to , we get
( + )
= ⋅ ( + )= ⇒ =
( + )
     
25. Find | x | if ( x  a )( x  a )  12 , where a is a unit vector.
   
Ans: ( x  a )  ( x  a )  12
       
 x  x  x  a  a  x  a  a  12
 
| x |2  | a |2  12
  
| x |2 1  12  | a | 1 as  a  is a unit vector 

| x |2  13

 | x | 13

SECTION – C
Questions 26 to 31 carry 3 marks each.
1
26. Evaluate:  9x 2
 6x  5
dx
Ans:

27. Probabilities of solving specific problem independently by A and B are and respectively. If
both try to solve the problem independently. Find the probability that (i) the problem is solved
(ii) exactly one of them solves the problems.
Ans: P(A)= and P(B) = ,P( ) = , P( ) =
(i) P(the problem is solved) =
(ii) P(exactly one of them solves) =

OR
From a lot of 30 bulbs which include 6 defectives, a sample of 4 bulbs is drawn at random with
replacement. Find the probability distribution of the number of defective bulbs.
Ans: It is given that out of 30 bulbs, 6 are defective.
⇒ Number of non-defective bulbs =30 − 6 = 24

Prepared by: M. S. KumarSwamy, TGT(Maths) Page - 5-


4 bulbs are drawn from the lot with replacement.
Let X be the random variable that denotes the number of defective bulbs in the selected bulbs.
4 4 4 4 256
 P(X  0)  P(4  non   defectiveand 0defective ) 4 C0     
5 5 5 5 625
3
1  4  256
P(X  1)  P(3  non   defectiveandldefective )  C1      4

5  5  625
2 2
1 4 90
P(X  2)  P(2  non    defectiveand 2defective )  4 C 2      
5  5  625
3 1
1  4 16
P( X  3)  P( I non    defectiveand 3defective  )  C3        4

 5   5  625
4 0
1 4 1
P(X  4)  P( O non   defectiveand 4defective )  C 4        4

 5   5  625
Therefore, the required probability distribution is as follows
X 0 1 2 3 4
P(X) 256/625 256/625 90/625 16/625 1/625

28. Evaluate : ∫
( ) ( )
Ans: Let = ∫   = ∫  
( )
( − )
=  =  −
1+ 1+
or 2 = ∫   or = ∫  
Put = so that − = .
when = 0, = 1 and = , = −1 , we get


=  =  
2 1+ 1+

= [ ] = [ 1− 0] = −0 =
4 4

OR
Evaluate: ∫ | + 2| dx
Ans: | + 2| = {( + 2) , ≥ −2 − ( + 2) , < −2
∫  | + 2| = ∫   − ( + 2) + ∫  ( + 2)
= − −2 + + 2 =29

29. Solve the differential equation: ( 1 – y2) (1 + logx) dx + 2xydy = 0


Ans: ( 1 – y2) (1 + logx) dx + 2xydy = 0
2 (1 + )
= −
1− 2
( )
Integrated both sides− |1 − | =− ∫
(By using substitution method)
(1 + )
− |1 − |= +
2

OR

Prepared by: M. S. KumarSwamy, TGT(Maths) Page - 6-


Solve the differential equation x dy – ydx = + dx
Ans: x dy - y dx = + dx
= ,x≠ 0
It is a homogeneous diff. Eqn.
Put y = vx and = v+x


=
Integrated on both sides,we get
log + √1 + =log| | +

⇒ + 1+ =| |⇒( + + ) =

30. Minimize and maximize Z = 600x + 400y


Subject to the constraints: x + 2y ≤ 12; 2x + y ≤ 12; 4x + 5y ≤ 20; x≥ 0; y ≥ 0 by graphical 
method
Ans: Draw the graph of 2x + y = 12 ..... (i)
We get intersection of (i) with the coordinate axes at points (0,12) and (6,0)
Draw the graph of x + 2y = 12 ..... (ii)
We get the intersection of (ii) with the coordinate axes at points (06) and (120)
Draw the graph of 4x + 5y = 20 ..... (iii)
We get the intersection of (iii) with the coordinate axes at points (0,4) and (5,0)
Common shaded region is the feasible region with corner points (5,0),(6,0),(4,4),(0,6),(0,4)

Corner points Z = 600x + 400y


(0,4) 1600 minimum
(0.6) 2400
(4,4) 4000 maximum
(6,0) 3600
(5,0) 3000
Hence, Minimum value of z = 1600 at (0,4) and Maximum value of z = 4000 at (4,4)

31. Evaluate: ∫ ( )( )

Ans: Let = ∫ ( )( )

Put = ,∴ =
( )( ) ( )( )
( ) ( )
Now, ( )( )
= + =
( )( )

Prepared by: M. S. KumarSwamy, TGT(Maths) Page - 7-


⇒ = ( + ) + (9 + 4 ) ⇒ + = 1 9 + 4 = 0
4 9
= − , =
5 5
4 9
∴ == − +
( + 4) ( + 9) + 4)
5( 5( + 9)
4 9 1 4 9 1
⇒ = − + = − × + × +
5 + 2 5 + 3 5 2 2 5 3 3
2 3
= − + +
5 2 5 3

SECTION – D
Questions 32 to 35 carry 5 marks each.

32. Find the area of the region in the first quadrant enclosed by the x-axis, the line y = x and the circle
x2 + y2 = 32.
Ans: The given equations are y = x ...(i)
and x2 + y2 = 32 ...(ii)
Solving (i) and (ii), we find that the line and the circle meet at
B(4, 4) in the first quadrant. Draw perpendicular BM to the x-axis.

Therefore, the required area = area of the region OBMO + area of the region BMAB.
4
 x2 
4
42 16
4

Now, the area of the region OBMO  ydx  xdx      0   8 sq. units …. (i)
0 0  2 0 2 2
4 2
4 2 4 2  x 32  x 2 32 1  x  
 ydx   32  x dx    sin  
2
Again, the area of the region BMAB =
4 4  2 2  4 2   4
 4 2 32  (4 2) 2 32  4 2    4 32  (4) 2 32 1  4  
  sin 1    sin 
 2 2  4 2    2 2 

4 2  
    
  1   
 [0  16sin 1 (1)]  8  16sin 1     16  2  8  16  4  [8  8  4 ]
  2 
 [4  8] sq. units
Required area = 8 + 4π – 8 = 4π
Therefore, the area is 4π square units.

Prepared by: M. S. KumarSwamy, TGT(Maths) Page - 8-


33. An insect is crawling along the line = = and another insect is crawling along the line
= = . At what points on the lines should they reach so that the distance between them
is the shortest? Find the shortest possible distance between them.
Ans: = = =k------(1)
= = = …… ( 2)
Lets take a point A on first line as A(k+3,-2k+5,k+7)
And B(7 -1, -6 − 1, − 1) be on the second line
The direction ratio of the line AB 7 − − 4, −6 + 2 − 6, − −8
Now ,as AB is the shor test distance betw een line 1 and line 2 so,
( 7 − − 4) × 1 + ( −6 + 2 − 6) × ( −2) + ( − − 8) × 1 = 0…..( 3)
( 7 − − 4) × 7 + ( −6 + 2 − 6) × ( −6) + ( − − 8) × 1 = 0….( 4)
Solving equation 3 and 4 ,k= 0, = 0
∴ A is ( 3,5,7) and B( -1,-1,-1)
AB= 2 √29 units

OR
The equation of motion of a rocket are: x = 2t, y = -4t, z = 4t, where the time t is given in
seconds, and the coordinates of a moving point in km. What is the path of the rocket? At what
distances will the rocket be from the starting point O(0,0,0) and from the following line in 10
seconds?
⃗ = 20 ̂ − 10 ̂ + 40 + ( 10 ̂ − 20 ̂ + 10
x y 2
Ans: Eliminating t between the equations, we obtain the equation of the path   , which
2 4 4
are the equations of the line passing through the origin having direction ratios.
This line is the path of the rocket.
When t = 10 seconds, the rocket will be at the point (20, -40, 40).
Hence, the required distance from the origin at 10 seconds
202  402  402  km  20  3 km  60 km
The distance of the point (20, -40, 40) from the given line
  
| (a2  a1 )  b | | 30 j  (10i  20 j  10k )
   km
|b| |10i  20 j  10k |
| 300i  300k | 300 2
 km  km  10 3km
|10i  20 j  10k | 10 6

34. Show that the relation R on the set Z of all integers, given by R = {(a, b) : 2 divides (a – b)} is an
equivalence relation.
Ans: Given relation R = {(a, b) : 2 divides (a – b)} on the set Z of all integers
Reflexive: Let a ∈ Z
Since (a – a) = 0, which is divisible by 2 i.e., (a, a) ∈ R
∴ R is reflexive.
Symmetric: Let a, b ∈ Z
such that (a, b) ∈ R ⇒ (a – b) is divisible by 2
⇒ – (a – b) is also divisible by 2
⇒ (b – a) is divisible by 2 ⇒ (b, a) ∈ R
i.e., (a, b) ∈ R ⇒ (b, a) ∈ R
∴ R is symmetric.
Transitive: Let a, b, c ∈ Z
such that (a, b) ∈ R ⇒ (a – b) is divisible by 2
Let a – b = 2k1 where k1 is an integer ...(i)

Prepared by: M. S. KumarSwamy, TGT(Maths) Page - 9-


and (b, c) ∈ R ⇒ (b – c) is divisible by 2 ⇒ b – c = 2 k2 where k2 is an integer ...(ii)
Adding (i) and (ii), we have
(a – b) + (b – c) = 2 (k1 + k2) ⇒ a – c = 2 (k1 + k2) ⇒ (a – c) is divisible by 2.
OR
 x2
Let A = R − {3} and B = R − {1} . Prove that the function f : A→ B defined by f(x) =   is f
 x 3 
one-one and onto ? Justify your answer.
 x2
Ans: Here, A = R − { 3), B = R − {1} and f : A→ B is defined as f(x) =  
 x 3 
Let x, y ∈A such that f(x) = f(y)
x 2 y 2
   ( x  2)( y  3)  ( y  2)( x  3)
x 3 y 3
 xy  3x  2 y  6  xy  3 y  2 x  6
 3x  2 y  3 y  2 x
 3x  2 x  3 y  2 y  x  y

Therefore, f is one- one. Let y ∈B = R −{1} . Then, y ≠ 1


The function f is onto if there exists x ∈ A such that f(x) = y.
Now, f(x) = y
x2
  y  x  2  xy  3 y
x 3
 x(1  y)  3 y  2
2  3y
x  A [ y  1]
1 y
2  3y
Thus, for any y ∈B, there exists  A such that
1 y
 2  3y 
2
 2  3 y   1  y  2  3y  2  2y  y
f    y
 1  y   2  3 y   3 2  3 y  3  3 y 1
 1 
 y 
Therefore, f is onto. Hence, function f is one-one and onto.

 1 1 0   2 2 4
35. Given A = 2 3 4 and B =  4 2 4 , verify that BA = 6I, how can we use the result
 
   
 0 1 2  2 1 5 
to find the values of x, y, z from given equations x – y = 3, 2x + 3y + 4z = 17, y + 2z = 17
 1 1 0   2 2 4
Ans: We have A = 2 3 4 and B =  4 2 4
 
   
 0 1 2  2 1 5 
 1 1 0   2 2 4   2  4  0 2  2  0 4  4  0 
Now, AB   2 3 4  4 2 4    4  12  8 4  6  4 8  12  20 
    
 0 1 2  2 1 5   0  4  4 0  2  2 0  4  10 

Prepared by: M. S. KumarSwamy, TGT(Maths) Page - 10


-
6 0 0   1 0 0
 0 6 0  6  0 1 0
 
   
0 0 6   0 0 1 
1
 AB  61  A 1   B
6
 2 2 4 
 A  4 2 4 
1 1
6 
 2 1 5 
The given system of linear equations can be written in matrix form as AX = B, where
 1 1 0  x  3
A  2 3 4 , X  y , B  17 
   
     
 0 1 2   z   7 
 X  A1  B
 2 2 4   3   2  3  2  17  4  7 
 X  4 2 4 17  X  4  3  2  17  4  7 
1    1
6   6 
 2 1 5   7   2  3  1 17  5  7 
x   2 
  y    1  x  2, y  1, z  4
   
 z   4 

SECTION – E(Case Study Based Questions)


Questions 36 to 38 carry 4 marks each.

36. Case-Study 1: Read the following passage and answer the questions given below.
Rohan, a student of class XII, visited his uncle’s flat with his father. He observe that the window
of the house is in the form of a rectangle surmounted by a semicircular opening having perimeter
10 m as shown in the figure.

(i) If a and y represents the length and breadth of the rectangular region, then find the relation
between x and y. Also, Find the expression of Area (A) of the window. [2]
(ii) Find the value of x for maximizing the Area (A) of whole window. [2]
(iii) Find the maximum area of the window.
OR
(iii) For maximum value of A, find the breadth of the rectangular part of the window. [2]
Ans: (i) Given, perimeter of window = 10
Prepared by: M. S. KumarSwamy, TGT(Maths) Page - 11
-
∴ x + y + y + perimeter of semicircle = 10
x
⇒ x + 2y +  = 10 which is the relation between x and y
2
x  x  1  x2
2
1  x 
Now, Area A  xy      x  5   
2  2  2 4  2 4
x2  x 2  x 2 x2  x 2
 5x     5x  
2 4 8 2 8
x x
2 2
(ii) We have, A  5 x  
2 8
dA x
  5 x
dx 4
dA x
 0 5 x 0
dx 4
x 20
5 x  x (4   )  20  x 
4 4 
2
d A 20
Clearly, 2  0 at x 
dx 4
 20   20  1   20 
2 2
20
(iii) At x  , A  5     
4  4    4   2 8  4  
100 200 50
  
4   (4   ) (4   )2
2

(4   )100  200  50 400  100  200  50


 
(4   )2 (4   ) 2
200  50 50(4   ) 50
  
(4   ) 2
(4   ) 2
4
OR
x x 1  
(iii) We have, y  5    5 x  
2 4 2 4
 2   20   2   
 5 x   5  
 4   4    4 
 2    20  5  10  5 10
 5  5  
 4   4 4 

37. Case-Study 2:
One day Shweta’s Mathematics teacher was explaining the topic Increasing and decreasing
functions in the class. He explained about different terms like stationary points, turning points etc.
He also explained about the conditions for which a function will be increasing or decreasing. He
took examples of different functions to make it more clear to the students. He then took the function
( ) = ( + 1)3( − 3)3 and ask the students to answer the following questions. With Shweta, you
can also test your knowledge by answering the questions

Prepared by: M. S. KumarSwamy, TGT(Maths) Page - 12


-
(i) Find the stationary points on the curve. [2]
(ii) Find the intervals where the function is increasing and decreasing? [2]
Ans: (i) We have, ( ) = ( + 1)3( − 3)3
 f  ( x )  3( x  1) 2 ( x  3)3  3( x  3)2 ( x  1)3  f  ( x)  3( x  1)2 ( x  3) 2 [ x  3  x  1]
 f  ( x)  3( x  1)2 ( x  3)2 (2 x  2)  f  ( x )  6( x  1)2 ( x  3)2 ( x  1)
For stationary points, f  ( x)  0  x  1,3,1
(ii) The points x = −1, x = 1, and x = 3 divide the real line into four disjoint intervals
i.e.,(−∞, −1), (−1, 1), (1, 3) and (3, ∞).
In intervals (−∞, −1) and (−1, 1), f′(x) = 6(x + 1)2(x − 3)2(x − 1) < 0
∴ f is strictly decreasing in intervals (−∞, −1) and (−1, 1).
In intervals (1, 3) and (3, ∞), f′(x) = 6(x + 1)2(x − 3)2(x − 1) > 0
∴ f is strictly increasing in intervals (1, 3) and (3, ∞).

38. Case-Study 3:
Mahindra Tractors is India’s leading farm equipment manufacturer. It is the largest tractor selling
factory in the world. This factory has two machine A and B. Past record shows that machine A
produced 60% and machine B produced 40% of the output(tractors). Further 2% of the tractors
produced by machine A and 1% produced by machine B were defective. All the tractors are put
into one big store hall and one tractor is chosen at random.

(i) Find the total probability of chosen tractor (at random) is defective.
(ii) If in random choosing, chosen tractor is defective ,then find the probability that the chosen
tractor is produced by machine ‘B’
Ans: (i) Let the probability that the product was made by Machine A be E1
Let the probability that the product was made by Machine B be E2
Let the probability that the product was defective be A
P(E1) = 60/100 = 0.6, P(E2) = 40/100 = 0.4
2% of items produced by A was defective
P(A∣E1) = 2/100 = 0.02
1% of items produced by B was defective
P(A∣E2) = 1/100 = 0.01
Prepared by: M. S. KumarSwamy, TGT(Maths) Page - 13
-
Total Probability, P(A) = P( E1 )  P( A∣E1 )  P( E2 )  P( A∣E2 )  0.6  0.02  0.4  0.01
 0.012  0.04  0.016
(ii) Using Bayes’ Theorem, Probability that the product was produced by machine B given it was
P ( E2 )  P ( A∣ E2 )
defective is P ( E2 ∣ A) 
P ( E1 )  P ( A∣ E1 )  P ( E2 )  P ( A∣ E2 )
0.4  0.01
  0.25
0.6  0.02  0.4  0.01
Hence, the probability that the product was produced by machine B given it was defective is 0.25

Prepared by: M. S. KumarSwamy, TGT(Maths) Page - 14


-
KENDRIYA VIDYALAYA GACHIBOWLI, GPRA CAMPUS, HYD–32
SAMPLE PAPER TEST 11 FOR BOARD EXAM (2022-23)
(ANSWERS)
SUBJECT: MATHEMATICS (041) MAX. MARKS : 80
CLASS : XII DURATION: 3 HRS
General Instructions:
1. This Question paper contains - five sections A, B, C, D and E. Each section is compulsory.
However, there are internal choices in some questions.
2. Section A has 18 MCQ’s and 02 Assertion-Reason based questions of 1 mark each.
3. Section B has 5 Very Short Answer (VSA)-type questions of 2 marks each.
4. Section C has 6 Short Answer (SA)-type questions of 3 marks each.
5. Section D has 4 Long Answer (LA)-type questions of 5 marks each.
6. Section E has 3 source based/case based/passage based/integrated units of assessment (4
marks each) with sub parts.

SECTION – A
Questions 1 to 20 carry 1 mark each.
1. The value of the expression ⃗ × ⃗ + ⃗ . ⃗ is
(a) ⃗ . ⃗ ( ) | ⃗ |. ⃗ ( ) | ⃗| ⃗ ( ) ( ⃗ . )⃗
Ans: ( ) | ⃗ | ⃗

 2 x  1;  if  x  2

2. For what value of k the function f ( x)   k , x  2 is continuous at x = 2 ,
 3x  1; x2

(a) Any real value (b) No real value (c) 5 (d) 1/5
Ans: (c) 5

 a b 
3. If A =   and A2 = I, then
 c a
2
(a) a + bc – 1 = 0 (b) 1 – a2 + b c = 0 (c) a2 + bc +1 = 0 (d) a2 – bc +1 = 0
2
Ans: (a) a + bc – 1 = 0

dx
4. The Integrating factor of the differential equation (1  y 2 )  yx  ay is
dy
(a) (b) (c) (d)
Ans: (d)

5. If A is a square matrix of order 3 such that |A| = - 5 , then value of | − | is


(a) 125 (b) – 125 (c) 25 (d) – 25
Ans: (d) – 25

6. If ( ) = ,then (x) is
(a) +C (b) +C (c) +C (d) +C
Ans: (a) +C

Prepared by: M. S. KumarSwamy, TGT(Maths) Page - 1-


4
d  dy  
   0 is
dx  dx  
7. The sum of the order and the degree of the differential equation

(a) 1 (b) 2 (c) 3 (d) 4


Ans: (c) 3

8. The value of (i  j ).k  ( j  k ).i  (k  i).j is


(a) 0 (b) –1 (c) 1 (d) 3
Ans: (d) 3

9. Corner points of the feasible region for an LPP are (0, 3), (1,1) and (3,0). Let Z = px + qy, where
p, q > 0, be the objective function. The condition on p and q so that the minimum of Z occurs at
(3,0) and (1,1) is
q
(a) p = q (b) p  (c) p = 3q (d) p=q
2
q
Ans: (b) p 
2

dx
10.  9x  4 x2
equals

1  9x  8  1 1  8x  9 
(a) sin 1  C (b) sin  C
9  8  2  9 
1  9x  8  1  9x  8 
(c) sin 1  C (d) sin 1  C
3  8  2  8 
1  8x  9 
Ans: (b) sin 1  C
2  9 

11. If A is a 3 x 3 matrix and |A| = - 2 then value of |A(adjA)| is


(a) -2 (b) 2 (c) -8 (d) 8
Ans: (c) -8

12. Corner points of the feasible region for an LPP are (0, 2), (3, 0), (6, 0), (6, 8) and (0, 5).Let F =
4x + 6y be the objective function. The Minimum value of F occurs at
(a) (0, 2) only (b) (3, 0) only
(c) the mid point of the line segment joining the points (0, 2) and (3, 0) only
(d) any point on the line segment joining the points (0, 2) and (3, 0).
Ans: (d) any point on the line segment joining the points (0, 2) and (3, 0).

x 2 6 2
13. If  , then x is equal to
18 x 18 6
(a) 6 (b) ± 6 (c) -6 (d) 0
Ans: (b) ± 6

14. If A is a square matrix of order 3, such that A(adjA) = 10 , then | | is equal to


(a) 1 (b) 10 (c) 100 (d) 101
Ans: (c) 100

15. Given two independent events A and B such that P(A) =0.3, P(B) = 0.6 and P( ’ ) is
(a) 0.42 (b) 0.18 (c) 0.28 (d) 0.12
Ans: (a) 0.42

Prepared by: M. S. KumarSwamy, TGT(Maths) Page - 2-


16. If y = 5e7x + 6e-7x,show that is equal to
(a) 7y (b) 6 (c) 49y (d) 36y
Ans: (c) 49y

17. The projection of ⃗ on ⃗, if ⃗. ⃗ =8 and ⃗ =2 ̂ + 6 ̂ + 3 .


8 2 2 4
(a) (b) (c) (d)
7 3 9 5
8
Ans: (a)
7

18. If the direction cosines of a line are k, k, k then


(a) k > 0 (b) 0 < k< 1 (c) k = 1 (d) k= or k = -
√ √
Ans: (d) k= or k = -
√ √

ASSERTION-REASON BASED QUESTIONS


In the following questions, a statement of assertion (A) is followed by a statement of Reason (R).
Choose the correct answer out of the following choices.
(a) Both A and R are true and R is the correct explanation of A.
(b) Both A and R are true but R is not the correct explanation of A.
(c) A is true but R is false.
(d) A is false but R is true.
 
19. Assertion(A) : The pair of lines given by r  i  j   (2i  k ) and r  2i  k   (i  j  k )
intersect .
Reason(R) : Two lines intersect each other, if they are not parallel and shortest distance = 0.
Ans: (a) Both A and R are true and R is the correct explanation of A.

 2  1  1  
20. Assertion (A) : The value of expression sec 1  1
  tan 1  sin  2  is 4
 3  
  
Reason (R) : Principal value branch of sin 1 x is   ,  and that of sec1 x is [0,  ]  { / 2}
 2 2
Ans: (d) A is false but R is true.

SECTION – B
Questions 21 to 25 carry 2 marks each.

1  x2  1
21. Write the simplest form of tan 1 ,x  0
x
Ans: Put x  tan     tan 1 x
1  x2  1 1  tan 2   1
tan 1  tan 1
x tan 
 sec   1  1  1  cos      1
 tan 1    tan 
1
  tan tan     tan x
1

 tan    sin   2 2 2


OR
n  1,  if  nis
   odd  
Show that f ∶ N → N, given by  f (n)   is a bijection.
n  1,  if  nis
   even  
Ans: Injective test:
Case I: If n is odd:

Prepared by: M. S. KumarSwamy, TGT(Maths) Page - 3-


Let x1 , x2  N such that f ( x1 )  f ( x2 )
f ( x1 )  x1  1, f ( x2 )  x2  1
As f ( x1 )  f ( x2 )  x1  1  x2  1  x1  x2
Case II: If n is even,
Let x1 , x2  N such that f ( x1 )  f ( x2 )
f ( x1 )  x1  1, f ( x2 )  x2  1
As f ( x1 )  f ( x2 )  x1  1  x2  1  x1  x2
∴ f is injective.
Surjection test:
Case I: If n is odd,
As, for every n ∈ N, there exists y = n − 1 in N such that,
f(y) = f(n − 1) = n – 1 + 1 = n
Case II: If n is even,
As, for every n ∈ N, there exists y = n + 1 in N such that,
f(y) = f(n + 1) = n + 1 – 1 = n
∴ f is surjective.
Since f is injective and surjective then, it is a bijection.

22. An edge of a variable cube is increasing at the rate of 5cm per second. How fast is the volume
increasing when the side is 15 cm.
Ans: Let be the edge of the cube and be the volume of the cube at any time . Given, =
5 / , = 15
Since we know the volume of cube = ( ) i.e., = .
⇒ = 3 ⋅

= 3 ⋅ ( 15) × 5 = 3375 /

     
23. If a  5i  j  7k b  i  j   k , then find the value of so that the vectors a  b a b
are orthogonal.
   
Ans: (a  b) and  (a  b) are orthogonal  
     
 ( a  b )  ( a  b )  0  | a |2  | b |2  0
 
 | a || b | 25  1  49  1  1   2
 75   2  2   2  2  75   2  75  2  73
    73
OR
Find the direction ratio and direction cosines of a line parallel to the line whose equations are
6 − 2 = 3 + 1 = 2 − 4
Ans: The equations of the line are 6x – 2 = 3y + 1 = 2z – 4, which when written in standard
 2  1
symmetric form, will be 6  x    3  y    2(z  2)
 6  3
 2  1 2 1
6  x   3 y   x y
 
6
 
3  2( z  2)
  6 3  z2
6 6 6 1 2 3
So, d.r.’s of line are: 1, 2, 3
 1 2 3 
and the required direction cosines are  , , 
 1 2 3 1 2 3 1 2 3 
2 2 2 2 2 2 2 2 3

Prepared by: M. S. KumarSwamy, TGT(Maths) Page - 4-


 1 2 3 
 , , 
 14 14 14 

dy sin 2 (a  y)
24. If x sin(a  y )  sin a cos(a  y )  0 , then pr ove that 
dx sin a
Ans: Given ( + )+ ( + ) = 0
⋅ ( + )
⇒ = − ⇒ = − ⋅ ( + )
( + )
Differentiating with respect to , we get
( + )
= ⋅ ( + )= ⇒ =
( + )
     
25. Find | x | if ( x  a )( x  a )  12 , where a is a unit vector.
   
Ans: ( x  a )  ( x  a )  12
       
 x  x  x  a  a  x  a  a  12
 
| x |2  | a |2  12
  
| x |2 1  12  | a | 1 as  a  is a unit vector 

| x |2  13

 | x | 13

SECTION – C
Questions 26 to 31 carry 3 marks each.
1
26. Evaluate:  9x 2
 6x  5
dx
Ans:

27. Probabilities of solving specific problem independently by A and B are and respectively. If
both try to solve the problem independently. Find the probability that (i) the problem is solved
(ii) exactly one of them solves the problems.
Ans: P(A)= and P(B) = ,P( ) = , P( ) =
(i) P(the problem is solved) =
(ii) P(exactly one of them solves) =

OR
From a lot of 30 bulbs which include 6 defectives, a sample of 4 bulbs is drawn at random with
replacement. Find the probability distribution of the number of defective bulbs.
Ans: It is given that out of 30 bulbs, 6 are defective.
⇒ Number of non-defective bulbs =30 − 6 = 24

Prepared by: M. S. KumarSwamy, TGT(Maths) Page - 5-


4 bulbs are drawn from the lot with replacement.
Let X be the random variable that denotes the number of defective bulbs in the selected bulbs.
4 4 4 4 256
 P(X  0)  P(4  non   defectiveand 0defective ) 4 C0     
5 5 5 5 625
3
1  4  256
P(X  1)  P(3  non   defectiveandldefective )  C1      4

5  5  625
2 2
1 4 90
P(X  2)  P(2  non    defectiveand 2defective )  4 C 2      
5  5  625
3 1
1  4 16
P( X  3)  P( I non    defectiveand 3defective  )  C3        4

 5   5  625
4 0
1 4 1
P(X  4)  P( O non   defectiveand 4defective )  C 4        4

 5   5  625
Therefore, the required probability distribution is as follows
X 0 1 2 3 4
P(X) 256/625 256/625 90/625 16/625 1/625

28. Evaluate : ∫
( ) ( )
Ans: Let = ∫   = ∫  
( )
( − )
=  =  −
1+ 1+
or 2 = ∫   or = ∫  
Put = so that − = .
when = 0, = 1 and = , = −1 , we get


=  =  
2 1+ 1+

= [ ] = [ 1− 0] = −0 =
4 4

OR
Evaluate: ∫ | + 2| dx
Ans: | + 2| = {( + 2) , ≥ −2 − ( + 2) , < −2
∫  | + 2| = ∫   − ( + 2) + ∫  ( + 2)
= − −2 + + 2 =29

29. Solve the differential equation: ( 1 – y2) (1 + logx) dx + 2xydy = 0


Ans: ( 1 – y2) (1 + logx) dx + 2xydy = 0
2 (1 + )
= −
1− 2
( )
Integrated both sides− |1 − | =− ∫
(By using substitution method)
(1 + )
− |1 − |= +
2

OR

Prepared by: M. S. KumarSwamy, TGT(Maths) Page - 6-


Solve the differential equation x dy – ydx = + dx
Ans: x dy - y dx = + dx
= ,x≠ 0
It is a homogeneous diff. Eqn.
Put y = vx and = v+x


=
Integrated on both sides,we get
log + √1 + =log| | +

⇒ + 1+ =| |⇒( + + ) =

30. Minimize and maximize Z = 600x + 400y


Subject to the constraints: x + 2y ≤ 12; 2x + y ≤ 12; 4x + 5y ≤ 20; x≥ 0; y ≥ 0 by graphical 
method
Ans: Draw the graph of 2x + y = 12 ..... (i)
We get intersection of (i) with the coordinate axes at points (0,12) and (6,0)
Draw the graph of x + 2y = 12 ..... (ii)
We get the intersection of (ii) with the coordinate axes at points (06) and (120)
Draw the graph of 4x + 5y = 20 ..... (iii)
We get the intersection of (iii) with the coordinate axes at points (0,4) and (5,0)
Common shaded region is the feasible region with corner points (5,0),(6,0),(4,4),(0,6),(0,4)

Corner points Z = 600x + 400y


(0,4) 1600 minimum
(0.6) 2400
(4,4) 4000 maximum
(6,0) 3600
(5,0) 3000
Hence, Minimum value of z = 1600 at (0,4) and Maximum value of z = 4000 at (4,4)

31. Evaluate: ∫ ( )( )

Ans: Let = ∫ ( )( )

Put = ,∴ =
( )( ) ( )( )
( ) ( )
Now, ( )( )
= + =
( )( )

Prepared by: M. S. KumarSwamy, TGT(Maths) Page - 7-


⇒ = ( + ) + (9 + 4 ) ⇒ + = 1 9 + 4 = 0
4 9
= − , =
5 5
4 9
∴ == − +
( + 4) ( + 9) + 4)
5( 5( + 9)
4 9 1 4 9 1
⇒ = − + = − × + × +
5 + 2 5 + 3 5 2 2 5 3 3
2 3
= − + +
5 2 5 3

SECTION – D
Questions 32 to 35 carry 5 marks each.

32. Find the area of the region in the first quadrant enclosed by the x-axis, the line y = x and the circle
x2 + y2 = 32.
Ans: The given equations are y = x ...(i)
and x2 + y2 = 32 ...(ii)
Solving (i) and (ii), we find that the line and the circle meet at
B(4, 4) in the first quadrant. Draw perpendicular BM to the x-axis.

Therefore, the required area = area of the region OBMO + area of the region BMAB.
4
 x2 
4
42 16
4

Now, the area of the region OBMO  ydx  xdx      0   8 sq. units …. (i)
0 0  2 0 2 2
4 2
4 2 4 2  x 32  x 2 32 1  x  
 ydx   32  x dx    sin  
2
Again, the area of the region BMAB =
4 4  2 2  4 2   4
 4 2 32  (4 2) 2 32  4 2    4 32  (4) 2 32 1  4  
  sin 1    sin 
 2 2  4 2    2 2 

4 2  
    
  1   
 [0  16sin 1 (1)]  8  16sin 1     16  2  8  16  4  [8  8  4 ]
  2 
 [4  8] sq. units
Required area = 8 + 4π – 8 = 4π
Therefore, the area is 4π square units.

Prepared by: M. S. KumarSwamy, TGT(Maths) Page - 8-


33. An insect is crawling along the line = = and another insect is crawling along the line
= = . At what points on the lines should they reach so that the distance between them
is the shortest? Find the shortest possible distance between them.
Ans: = = =k------(1)
= = = …… ( 2)
Lets take a point A on first line as A(k+3,-2k+5,k+7)
And B(7 -1, -6 − 1, − 1) be on the second line
The direction ratio of the line AB 7 − − 4, −6 + 2 − 6, − −8
Now ,as AB is the shor test distance betw een line 1 and line 2 so,
( 7 − − 4) × 1 + ( −6 + 2 − 6) × ( −2) + ( − − 8) × 1 = 0…..( 3)
( 7 − − 4) × 7 + ( −6 + 2 − 6) × ( −6) + ( − − 8) × 1 = 0….( 4)
Solving equation 3 and 4 ,k= 0, = 0
∴ A is ( 3,5,7) and B( -1,-1,-1)
AB= 2 √29 units

OR
The equation of motion of a rocket are: x = 2t, y = -4t, z = 4t, where the time t is given in
seconds, and the coordinates of a moving point in km. What is the path of the rocket? At what
distances will the rocket be from the starting point O(0,0,0) and from the following line in 10
seconds?
⃗ = 20 ̂ − 10 ̂ + 40 + ( 10 ̂ − 20 ̂ + 10
x y 2
Ans: Eliminating t between the equations, we obtain the equation of the path   , which
2 4 4
are the equations of the line passing through the origin having direction ratios.
This line is the path of the rocket.
When t = 10 seconds, the rocket will be at the point (20, -40, 40).
Hence, the required distance from the origin at 10 seconds
202  402  402  km  20  3 km  60 km
The distance of the point (20, -40, 40) from the given line
  
| (a2  a1 )  b | | 30 j  (10i  20 j  10k )
   km
|b| |10i  20 j  10k |
| 300i  300k | 300 2
 km  km  10 3km
|10i  20 j  10k | 10 6

34. Show that the relation R on the set Z of all integers, given by R = {(a, b) : 2 divides (a – b)} is an
equivalence relation.
Ans: Given relation R = {(a, b) : 2 divides (a – b)} on the set Z of all integers
Reflexive: Let a ∈ Z
Since (a – a) = 0, which is divisible by 2 i.e., (a, a) ∈ R
∴ R is reflexive.
Symmetric: Let a, b ∈ Z
such that (a, b) ∈ R ⇒ (a – b) is divisible by 2
⇒ – (a – b) is also divisible by 2
⇒ (b – a) is divisible by 2 ⇒ (b, a) ∈ R
i.e., (a, b) ∈ R ⇒ (b, a) ∈ R
∴ R is symmetric.
Transitive: Let a, b, c ∈ Z
such that (a, b) ∈ R ⇒ (a – b) is divisible by 2
Let a – b = 2k1 where k1 is an integer ...(i)

Prepared by: M. S. KumarSwamy, TGT(Maths) Page - 9-


and (b, c) ∈ R ⇒ (b – c) is divisible by 2 ⇒ b – c = 2 k2 where k2 is an integer ...(ii)
Adding (i) and (ii), we have
(a – b) + (b – c) = 2 (k1 + k2) ⇒ a – c = 2 (k1 + k2) ⇒ (a – c) is divisible by 2.
OR
 x2
Let A = R − {3} and B = R − {1} . Prove that the function f : A→ B defined by f(x) =   is f
 x 3 
one-one and onto ? Justify your answer.
 x2
Ans: Here, A = R − { 3), B = R − {1} and f : A→ B is defined as f(x) =  
 x 3 
Let x, y ∈A such that f(x) = f(y)
x 2 y 2
   ( x  2)( y  3)  ( y  2)( x  3)
x 3 y 3
 xy  3x  2 y  6  xy  3 y  2 x  6
 3x  2 y  3 y  2 x
 3x  2 x  3 y  2 y  x  y

Therefore, f is one- one. Let y ∈B = R −{1} . Then, y ≠ 1


The function f is onto if there exists x ∈ A such that f(x) = y.
Now, f(x) = y
x2
  y  x  2  xy  3 y
x 3
 x(1  y)  3 y  2
2  3y
x  A [ y  1]
1 y
2  3y
Thus, for any y ∈B, there exists  A such that
1 y
 2  3y 
2
 2  3 y   1  y  2  3y  2  2y  y
f    y
 1  y   2  3 y   3 2  3 y  3  3 y 1
 1 
 y 
Therefore, f is onto. Hence, function f is one-one and onto.

 1 1 0   2 2 4
35. Given A = 2 3 4 and B =  4 2 4 , verify that BA = 6I, how can we use the result
 
   
 0 1 2  2 1 5 
to find the values of x, y, z from given equations x – y = 3, 2x + 3y + 4z = 17, y + 2z = 17
 1 1 0   2 2 4
Ans: We have A = 2 3 4 and B =  4 2 4
 
   
 0 1 2  2 1 5 
 1 1 0   2 2 4   2  4  0 2  2  0 4  4  0 
Now, AB   2 3 4  4 2 4    4  12  8 4  6  4 8  12  20 
    
 0 1 2  2 1 5   0  4  4 0  2  2 0  4  10 

Prepared by: M. S. KumarSwamy, TGT(Maths) Page - 10


-
6 0 0   1 0 0
 0 6 0  6  0 1 0
 
   
0 0 6   0 0 1 
1
 AB  61  A 1   B
6
 2 2 4 
 A  4 2 4 
1 1
6 
 2 1 5 
The given system of linear equations can be written in matrix form as AX = B, where
 1 1 0  x  3
A  2 3 4 , X  y , B  17 
   
     
 0 1 2   z   7 
 X  A1  B
 2 2 4   3   2  3  2  17  4  7 
 X  4 2 4 17  X  4  3  2  17  4  7 
1    1
6   6 
 2 1 5   7   2  3  1 17  5  7 
x   2 
  y    1  x  2, y  1, z  4
   
 z   4 

SECTION – E(Case Study Based Questions)


Questions 36 to 38 carry 4 marks each.

36. Case-Study 1: Read the following passage and answer the questions given below.
Rohan, a student of class XII, visited his uncle’s flat with his father. He observe that the window
of the house is in the form of a rectangle surmounted by a semicircular opening having perimeter
10 m as shown in the figure.

(i) If a and y represents the length and breadth of the rectangular region, then find the relation
between x and y. Also, Find the expression of Area (A) of the window. [2]
(ii) Find the value of x for maximizing the Area (A) of whole window. [2]
(iii) Find the maximum area of the window.
OR
(iii) For maximum value of A, find the breadth of the rectangular part of the window. [2]
Ans: (i) Given, perimeter of window = 10
Prepared by: M. S. KumarSwamy, TGT(Maths) Page - 11
-
∴ x + y + y + perimeter of semicircle = 10
x
⇒ x + 2y +  = 10 which is the relation between x and y
2
x  x  1  x2
2
1  x 
Now, Area A  xy      x  5   
2  2  2 4  2 4
x2  x 2  x 2 x2  x 2
 5x     5x  
2 4 8 2 8
x x
2 2
(ii) We have, A  5 x  
2 8
dA x
  5 x
dx 4
dA x
 0 5 x 0
dx 4
x 20
5 x  x (4   )  20  x 
4 4 
2
d A 20
Clearly, 2  0 at x 
dx 4
 20   20  1   20 
2 2
20
(iii) At x  , A  5     
4  4    4   2 8  4  
100 200 50
  
4   (4   ) (4   )2
2

(4   )100  200  50 400  100  200  50


 
(4   )2 (4   ) 2
200  50 50(4   ) 50
  
(4   ) 2
(4   ) 2
4
OR
x x 1  
(iii) We have, y  5    5 x  
2 4 2 4
 2   20   2   
 5 x   5  
 4   4    4 
 2    20  5  10  5 10
 5  5  
 4   4 4 

37. Case-Study 2:
One day Shweta’s Mathematics teacher was explaining the topic Increasing and decreasing
functions in the class. He explained about different terms like stationary points, turning points etc.
He also explained about the conditions for which a function will be increasing or decreasing. He
took examples of different functions to make it more clear to the students. He then took the function
( ) = ( + 1)3( − 3)3 and ask the students to answer the following questions. With Shweta, you
can also test your knowledge by answering the questions

Prepared by: M. S. KumarSwamy, TGT(Maths) Page - 12


-
(i) Find the stationary points on the curve. [2]
(ii) Find the intervals where the function is increasing and decreasing? [2]
Ans: (i) We have, ( ) = ( + 1)3( − 3)3
 f  ( x )  3( x  1) 2 ( x  3)3  3( x  3)2 ( x  1)3  f  ( x)  3( x  1)2 ( x  3) 2 [ x  3  x  1]
 f  ( x)  3( x  1)2 ( x  3)2 (2 x  2)  f  ( x )  6( x  1)2 ( x  3)2 ( x  1)
For stationary points, f  ( x)  0  x  1,3,1
(ii) The points x = −1, x = 1, and x = 3 divide the real line into four disjoint intervals
i.e.,(−∞, −1), (−1, 1), (1, 3) and (3, ∞).
In intervals (−∞, −1) and (−1, 1), f′(x) = 6(x + 1)2(x − 3)2(x − 1) < 0
∴ f is strictly decreasing in intervals (−∞, −1) and (−1, 1).
In intervals (1, 3) and (3, ∞), f′(x) = 6(x + 1)2(x − 3)2(x − 1) > 0
∴ f is strictly increasing in intervals (1, 3) and (3, ∞).

38. Case-Study 3:
Mahindra Tractors is India’s leading farm equipment manufacturer. It is the largest tractor selling
factory in the world. This factory has two machine A and B. Past record shows that machine A
produced 60% and machine B produced 40% of the output(tractors). Further 2% of the tractors
produced by machine A and 1% produced by machine B were defective. All the tractors are put
into one big store hall and one tractor is chosen at random.

(i) Find the total probability of chosen tractor (at random) is defective.
(ii) If in random choosing, chosen tractor is defective ,then find the probability that the chosen
tractor is produced by machine ‘B’
Ans: (i) Let the probability that the product was made by Machine A be E1
Let the probability that the product was made by Machine B be E2
Let the probability that the product was defective be A
P(E1) = 60/100 = 0.6, P(E2) = 40/100 = 0.4
2% of items produced by A was defective
P(A∣E1) = 2/100 = 0.02
1% of items produced by B was defective
P(A∣E2) = 1/100 = 0.01
Prepared by: M. S. KumarSwamy, TGT(Maths) Page - 13
-
Total Probability, P(A) = P( E1 )  P( A∣E1 )  P( E2 )  P( A∣E2 )  0.6  0.02  0.4  0.01
 0.012  0.04  0.016
(ii) Using Bayes’ Theorem, Probability that the product was produced by machine B given it was
P ( E2 )  P ( A∣ E2 )
defective is P ( E2 ∣ A) 
P ( E1 )  P ( A∣ E1 )  P ( E2 )  P ( A∣ E2 )
0.4  0.01
  0.25
0.6  0.02  0.4  0.01
Hence, the probability that the product was produced by machine B given it was defective is 0.25

Prepared by: M. S. KumarSwamy, TGT(Maths) Page - 14


-
KENDRIYA VIDYALAYA GACHIBOWLI, GPRA CAMPUS, HYD–32
SAMPLE PAPER TEST 12 FOR BOARD EXAM (2022-23)
(ANSWERS)
SUBJECT: MATHEMATICS (041) MAX. MARKS : 80
CLASS : XII DURATION: 3 HRS
General Instructions:
1. This Question paper contains - five sections A, B, C, D and E. Each section is compulsory.
However, there are internal choices in some questions.
2. Section A has 18 MCQ’s and 02 Assertion-Reason based questions of 1 mark each.
3. Section B has 5 Very Short Answer (VSA)-type questions of 2 marks each.
4. Section C has 6 Short Answer (SA)-type questions of 3 marks each.
5. Section D has 4 Long Answer (LA)-type questions of 5 marks each.
6. Section E has 3 source based/case based/passage based/integrated units of assessment (4
marks each) with sub parts.

SECTION – A
Questions 1 to 20 carry 1 mark each.

1. The integrated factor of the differential equation: (1+x2) + = is


(a) (b) 2 (c) 3 (d)
Ans: (d)

2. For any square matrix A, AAT is a


(a) unit matrix (b) symmetric matrix (c) skew-symmetric matrix (d) diagonal matrix
Ans: (b) symmetric matrix

 2   3
3. If A   0 2 5  then A-1 exist if
 
 1 1 3 
(a) λ = 2 (b) λ = 0 (c) λ ≠ 2 (d) λ ≠ 0
Ans: (c) λ ≠ 2

4. If A is square matrix such that A2 = I , then (A - I)3 + (A + I)2 – 7A is equal to


(a) -A (b) I – A (c) I + A (d) 3A
Ans: (a) -A

5 x
5. If A=   and A = A’, then
 y 0
(a) x = 0, y = 5 (b) x + y = 5 (c) x = y (d) none of these
Ans: (c) x = y

6. The differential coefficient of sec(tan-1 x ) w.r.t. x is


(a) √ (b) (c) x √1 + (d) √
Ans: (a) √

7. If A is a square matrix of order ,then | ( )| =


(a) | | (b) | | (c) | | (d) | |
Ans: (b) | |

Prepared by: M. S. KumarSwamy, TGT(Maths) Page - 1-


8. P is a point on the line joining the points (0,5, -2) and (3, -1,2) . If the x-coordinate of P is 6,
then its z-coordinate is
(a) 10 (b) 6 (c) -6 (d) -10
Ans: (b) 6


9. The value of ∫
(a) (b) (c) (d)
Ans: (d)

10. The sum of the order and degree of the following differential equation = 0, is
(a) 5 (b) 4 (c) 3 (d) 2
Ans: (c) 3

11. The value of such that the vectors ⃗ = 2 ̂ + ̂ + and ⃗= ̂ + 2 ̂ + 3 are orthogonal is
(a) 0 (b) 1 (c) (d) -
Ans: (d) -

12. The corner points of the shaded bounded feasible region of an LPP are (0,0),(30,0),(20,30) and
(0,50) as shown in the figure .

The maximum value of the objective function Z = 4x+y is


(a) 120 (b) 130 (c) 140 (d) 150
Ans: (a) 120

13. The projection of the vector 2i  3 j  2k on the vector i  2 j  k is


√ √
(a) ( ) ( ) ( )

Ans: (a)

14. If ⃗ + ⃗ = 60, ⃗ − ⃗ = 40 and | ⃗ | = 22 then ⃗ =


(a) 36 (b) 22/60 (c) 46 (d) None of these
Ans: (c) 46

15. If the function f(x) defined by f(x) = , ≠3 , = 3 is continuous at x = 3, then the


value of k is
(a) 6 (b) 3 (c) -6 (d) 3
Ans: (a) 6

Prepared by: M. S. KumarSwamy, TGT(Maths) Page - 2-


16. Let A and B be two events . If P(A)=0.2,P(B)=0.4 ,P(AUB)=0.6 then P is equal to
(a) 1 (b) 0 (c) 0.2 (d) 0.4
Ans: (b) 0

17. The corner points of the feasible region determined by the following system of linear inequalities:
2x + y ≤ 10, x + 3y ≤ 15, x, y ≥ 0 are (0,0), (5,0), (3,4), (0,5). Let Z= px + qy, where p,q > 0.
Condition on p and q so that the maximum of Z occurs at both (3,4) and (0,5) is
(a) p = q (b) p = 2q (c) p = 3q (d) q = 3p
Ans: (d) q = 3p

18. The value of ∫ dx is


(a) 2 (b) 3/4 (c) 0 (d) – 2
Ans: (c) 0

ASSERTION-REASON BASED QUESTIONS


In the following questions, a statement of assertion (A) is followed by a statement of Reason (R).
Choose the correct answer out of the following choices.
(a) Both A and R are true and R is the correct explanation of A.
(b) Both A and R are true but R is not the correct explanation of A.
(c) A is true but R is false.
(d) A is false but R is true.

19. Assertion(A): The value of + − =


Reason(R) : (− ) = −
Ans: (b) Both A and R are true but R is not the correct explanation of A.

20. Assertion(A): = = and = = are coplaner.


Reason (R) : Let line passes through the point ( , , ) and parallel to the vector whose
direction ratios are , , ,: Let line passes through the point ( , , ) and parallel to
the vector whose direction ratios are , , ,. Then both lines are coplaner if and
only if | − − − | =0
Ans: (a) Both A and R are true and R is the correct explanation of A.

SECTION – B
Questions 21 to 25 carry 2 marks each.

21. Find the value of sin-1[cos( )]


Ans: = 6 +
3 3
= = − = − = − sin
5 2 5 10 10
= − = − ,
10 10
∵ ∈ − ,
10 2 2
OR
Let A = R- { 3 } and B = R- { 1 } . Consider the function f : A → B defined by f(x) = . Is f is
one-one and onto? Justify your Answer.
Ans: A = R- { 3 } and B = R- { 1 }
function f : A → B defined by f(x) =

Prepared by: M. S. KumarSwamy, TGT(Maths) Page - 3-


x1  2 x2  2
Now, f ( x1 )  f ( x2 )  
x1  3 x2  3
 ( x2  3)( x1  2)  ( x2  2)( x1  3)
 x1 x2  3 x1  2 x2  6  x1 x2  3x2  2 x1  6
 3x1  2 x2  3x2  2 x1
  x1   x2  x1  x2
Hence, f(x) is one-one function.
x2 x2
Let f ( x)   y  y( x  3)  x  2  yx  3 y  x  2
x3 x 3
3y  2
 x( y  1)  3 y  2  x 
( y  1)
3y  2 3 y  2  2( y  1)
2
x2 y 1 y 1
 f ( x)   
x3 3y  2 3 y  2  3( y  1)
3
y 1 y 1
3y  2  2 y  2 3y  2 y
  y
3 y  2  3 y  3 2  3
f(x) is onto.
So f(x) is bijective and invertible

22. If √1 − + 1− = ( − ), show that = .


Ans: Let x  sin A; y  sin B  (1)
f ( x)  1  sin 2 A  1  sin 2 B  a(sin A  sin B)  cos A  cos B  a(sin A  sin B)
We know that [1  sin 2 A  cos2 A]
AB
cos  
AB  A B  2  a
 2 cos    a  2sin   
 2   2  AB
sin  
 2 
AB  cos  
 cot    a   cot  
 2   sin  
AB
  cot 1 a  sin 1 x  sin 1 y  2 cot 1 a
2
1 1 dy
  0
1  x2 1  y 2 dx
dy 1  y2
 
dx 1  x2

23. The volume of a cube is increasing at the rate of 9 cubic cm per sec . How fast is the surface area
increasing when the length of an edge is 10 cm.
Ans: Let V and S be the volume and surface area of a cube of side x cm respectively.
dV
Given,  9 cm3 / sec
dt
 ds 
We require  
 dt  x 10 cm

Prepared by: M. S. KumarSwamy, TGT(Maths) Page - 4-


dv dx dx
Now, V  x 3   3x 2   9  3x 2 
dt dt dt
dx 9 3
  
dt 3x 2 x 2
Again, S  6 x 2  By formula for surface area of a cube
ds dx 3 36  ds  36
  12 x   12 x  2      3.6 cm 2 / sec
dt dt x x   x 10 cm
dt 10

24. Find the vector of magnitude 6, which is perpendicular to both the vectors 2i  j  2k
4i  j  3k
Ans: Any vector perpendicular to ⃗ = 2 ̂ − ̂ + 2 ⃗ = 4 ̂ − ̂ + 3 is
⃗× ⃗= ̂ ̂ 2 −124 −13 = - ̂+ 2 ̂+ 2 = ⃗
A vector of magnitude 6 in the direction of ⃗

=| ⃗ | . 6 =-2 ̂ + 4 ̂ + 4
OR
Find the equation of a line in vector and cartesian form which passes through the point (1,2,3) and
is parallel to the vector 3i  2 j  3k .
Ans:

25. If ⃗+ ⃗+ ⃗ =0⃗ and | ⃗ | =3, ⃗ =5 and | ⃗| =7 then what is the angle between ⃗ and ⃗ .
Ans: ⃗+ ⃗+ ⃗ =0⃗
( ⃗ + ⃗) = ( − ⃗) ,
⇒ ⃗+ ⃗. ⃗+ ⃗ = ⃗. ⃗, | ⃗ | + ⃗ + 2 ⃗ . ⃗ = | ⃗|
⇒ 2 ⃗ . ⃗=15,
⇒ 2 | ⃗ | ⃗ cos = 15, = 60°

SECTION – C
Questions 26 to 31 carry 3 marks each.
26. Find: ∫ √
Ans: 5  4 x  2 x 2  (2 x 2  4 x  5)
 5  5  2  5   7 
 2  x 2  2 x    2  x 2  2 x  1  1    2  ( x  1)2    2  ( x  1)2  
 2  2  2   2 
7  7  2 
2
 2   ( x  1)   2    ( x  1) 
2

2   2 
 

Prepared by: M. S. KumarSwamy, TGT(Maths) Page - 5-


dx dx 1 dx
   
5  4 x  2 x2  72  2  72 
2  ( x  1) 2    ( x  1) 2 
 2   2 
dx x
Using   sin 1  c , we get
a2  x2 a
1 x 1 1 2
 sin 1 c  sin 1 ( x  1)  c
2 7 2 7
2

OR
x  x 1
3
Evaluate:  x2 1
dx
Ans:

27. Two cards are drawn simultaneously from a well shuffled pack of 52 playing cards. Find the mean
of the number of aces drawn.
Ans: Let X denote the number of King in a draw of two card
48
C 48  47 188
 P( X  0)  P( no ace)  52 2  
C2 52  51 221
4
C1 48 C1 4  48 32
 P( X  1)  P ( one ace and one non ace )   
52
C2 52  51 221
C2 4
4 3 1
 P( X  2)  P( two ace)   
C2 52  51 221
52

n
188 32 2 34
 Mean of X  xi p ( xi )  0   1  
2 1 221 221 221 221
OR
Three friends go for coffee. They decide who will pay the bill, by each tossing a coin and then
letting the “odd person” pay. There is no odd person if all three tosses produce the same result. If
there is no odd person in the first round, they make a second round of tosses and they continue to
do so until there is an odd person. What is the probability that exactly three rounds of tosses are
made?
Ans: P(not obtaining an odd person in a single round)
1 1 1 1
= P(All three of them throw tails or All three of them throw heads) =    2 
2 2 2 4
P(obtaining an odd person in a single round)
3
= 1 – P(not obtaining an odd person in a single round) =
4
∴ Required probability

Prepared by: M. S. KumarSwamy, TGT(Maths) Page - 6-


= P(‘In first round there is no odd person’ and ‘In second round there is no odd person’ and ‘In
1 1 3 3
third round there is an odd person’) =   
4 4 4 64
 /4
28. Evaluate:  log(1  tan x)dx
0

Ans:

29. Solve the following Linear Programming Problem graphically:


Maximize: Z = 100 + 120
Subject to : 5 + 8 ≤ 200, 5 + 4 ≤ 120, , ≥ 0
Ans: Plotting the constraints in graph

Corner points(0,0),(24,0),(8,20),and (0,25)


At(0,0) Z=0
At(0,25) Z=3000
At(24,0) Z=2400
At(8,20) Z=3200 (Maximum)
Maximum value of Z is Rs.3200 at point (8,20)

d2 y
30. If x = a (cos t + t sin t) and y = a (sin t – t cos t), find
dx2
Ans. Given that x = a (cos t + t sin t) and y = a (sin t – t cos t)
Differentiating both sides w.r.t. t, we get

Prepared by: M. S. KumarSwamy, TGT(Maths) Page - 7-


dx d  d d 
 a  cos t   t sin t  sin t (t )    a[ sin t  (t sin t  sin t )]  at cos t
dt  dt  dt dt  
dy d  d d 
and  a  sin t   t cos t  cos t (t )    a[cos t  (t sin t  cos t )]  at sin t
dt  dt  dt dt  
dy
dy dt at sin t
Now,    tan t
dx dx at cos t
dt
Again, Differentiating both sides w.r.t. x, we get
d2 y d dt sec2 t 1
2
 tan t  sec 2
t    sec3 t
dx dx dx at cos t at

31. Find the particular solution of the differential equation (1 + e2x)dy + (1 + y2) ex dx = 0 given that
y = 1 when x = 0.
Ans: (1  e 2 x )dy  (1  y 2 )e x dx  0
dy e x dx 1 e x dx
    tan y  
(1  y 2 ) (1  e 2 x ) (1  e 2 x )
Let t  e x  dt  e x dx
dt
 tan 1 y    tan 1 y   tan 1 t  c
(1  t 2 )
 tan 1 y   tan 1 (e x )  c
When x = 0 and y = 1, tan 1 1   tan 1 (e 0 )  c
  
   c  c 
4 4 2
 
Hence, tan 1 y   tan 1 (e x )   tan 1 y + tan 1 (e x ) 
2 2
OR
dy 
Solve the following differential equation:  2 y tan x  sin x , given that, when x  , y = 0
dx 3
dy
Ans: The given differential equation is of the form  Py  Q
dx
Comparing the two equations, P  2 y tan x & Q  sin x
Integrating Factor (IF) = e  e e 
Pdx 2tan xdx 2 tan xdx
 e2[logsec x ]  elog(sec x )  sec2 x
2

∴ Solution to the differential equation


 y( IF )  (Q  IF )dx  C  ysec2 x   sin xsec 2 xdx  C
sin x sin x 1
 y sec 2 x   2
dx  C    dx  C
cos x cos x cos x
 y sec2 x   tan x sec xdx  C  y sec2 x  sec x  C
1 C
y  2
 cos x  C (cos 2 x)
sec x sec x

When x  , y  0
3
   1 1
 y  0  cos  C cos 2     C  0  C  2
3 3 2 4
 y  cos x  2cos x
2

Prepared by: M. S. KumarSwamy, TGT(Maths) Page - 8-


SECTION – D
Questions 32 to 35 carry 5 marks each.
32. Find the equation of a line passing through the pointg P(2, −1, 3) and perpendicular to the lines
    
r  i  j  k   (2i  2 j  k ) and r  2i  j  3k   (i  2 j  2 k )
x  2 y 1 z  3
Ans: Let the equation of line passing through the point (2, -1, 3) be   …(1)
a b c

Given lines are r  i  j  k   (2i  2 j  k ) ……. (2)

and r  2i  j  3k   (i  2 j  2 k ) …… (3)
Since (1), (2) and (3) are perpendicular to each other
 2 a  2b  c  0 and a  2b  2c  0
a b c a b c
      l
4  2 1  4 4  2 6 3 6
 a  6l ,b  3l , c  6l
x  2 y 1 z  3
Putting it in (1) we get required equation of line as  
6l 3l 6l
x2 z 3
  y 1 
2 2
OR
Find the coordinates of the foot of perpendicular drawn from the point A(–1, 8, 4) to the line
joining the points B(0, –1, 3) and C(2, –3, –1). Hence find the image of the point A in the line BC.
Ans: Let P be the foot of the perpendicular drawn from point A on the line joining points B and
C. Equation of the line joining the points B(0,–1,3) and C(2,–3,–1) is
x  x1 y  y1 z  z1
 
x2  x1 y2  y1 z2  z1
x  0 y 1 z 3 x  0 y 1 z  3
     
2  0 3  1 1  3 2 2 4
x  0 y 1 z  3
Let   
2 2 4
General coordinates of P is (2λ,−2λ−1,−4λ+3)
Direction ratios of AP(2λ+1,−2λ−9,−4λ−1)
∵ Both the lines AP and BC are perpendicular to each other.
∴ 2(2λ+1)−2(−2λ−9)−4(−4λ−1)=0
⇒ 24λ+24=0
⇒ λ=−1
∴ P(−2, 1, 7)
So, the coordinates of the foot of perpendicular drawn from the point A to BC is P(−2, 1, 7).

Let Q(h, p, s) be the image of A in the line BC,

Prepared by: M. S. KumarSwamy, TGT(Maths) Page - 9-


So P must be the mid-point of AQ.
 h 1 p  8 s  4 
P , ,   P(2,1, 7)
 2 2 2 
On comparing the coordinates, we get h = -3, p=-6, s= 10,
Hence the image is Q(-3, -6, 10).

33. Make a rough sketch of the region {( , ) : 0 ≤ ≤ + 1,0 ≤ ≤ + 1,0 ≤ ≤ 2 } and find
the area of the region using integration.
Ans:

Required Area=∫ ( + 1) + ∫ ( + 1)
=

34. Let = {1,2,3, …,9} and be the relation in × defined by (a, b) R (c, d) if a+ d = b+ c, for
( , ) , ( , ) in × . Prove that is an equivalence relation and also obtain the equivalence class
[ ( 2,5) ] .
Ans: Ans: A = {1,2,3...9}
R in A × A
(a,b) R (c,d) if (a,b)(c,d) ∈ A∈A
a+b=b+c
Consider (a,b) R (a,b) (a,b) ∈ A × A
a+b=b+a
Hence, R is reflexive.
Consider (a, b) R (c, d) given by (a, b) (c, d) ∈ A × A
a+d=b+c⇒c+b=d+a
⇒(c, d) R (a, b)
Hence R is symmetric.
Let (a, b) R (c, d) and (c, d) R (e, f)
(a, b),(c, d),(e, f) ∈ A × A
a + b = b + c and c + f = d + e
a+b=b+c
⇒ a − c = b − d .....(1)
c + f = d + e ......(2)
Adding (1) and (2)
a−c+c+f=b−d+d+e
⇒a+f=b+e
⇒ (a, b) R (e, f)
R is transitive.
R is an equivalence relation.
We select from set A={1,2,3,....9}
a and b such that 2 + b = 5 + a
so b = a + 3
Consider (1,4)
(2, 5) R (1, 4) ⇒ 2 + 4 = 5 + 1

Prepared by: M. S. KumarSwamy, TGT(Maths) Page - 10-


[(2,5)=(1,4)(2,5),(3,6),(4,7),(5,8),(6,9)] is the equivalent class under relation R.

OR
x
Show that the function f : R→ {x  R : −1 < x <1} defined by f( x) = , x  R is one-one
1 | x |
and onto function.
x
Ans: It is given that f : R→ {x  R : −1 < x <1} defined by f( x) = , xR
1 | x |
x y
Suppose, f(x) = f(y), where x, y  R  
1 | x | 1 | y |
It can be observed that if x is positive and y is negative, then we have
x y
  2 xy  x  y
1  x 1 y
Since, x is positive and y is negative, then x > y  x − y > 0

But, 2xy is negative. Then, 2xy ≠ x − y.


Thus, the case of x being positive and y being negative can be ruled out.
Under a similar argument, x being negative and y being positive can also be ruled out. Therefore,
x and y have to be either positive or negative.
x y
When x and y are both positive, we have f ( x)  f ( y)    x  xy  y  xy  x  y
1 x 1 y
x y
When x and y are both negative, we have f ( x)  f ( y)    x  xy  y  xy  x  y
1 x 1  y
Therefore, f is one-one. Now, let y  R such that −1 < y < 1.
y
If y is negative, then there exists x   R such that
1 y
 y  y

 y   1 y  
1 y y
f ( x)  f     y
 1  y  1 y 1 
  y  1 y  y

1 y  1 y 
y
If y is positive, then there exists x   R such that
1 y
 y  y

 y   1 y  
1 y y
f ( x)  f     y
 1 y  1 y  y  1 y  y
1 
1 y 
 1 y 
Therefore, f is onto. Hence, f is one-one and onto.

 1 1 0   2 2 4
35. Given A =  2 3 4  and B =  4 2 4 , verify that BA = 6I, how can we use the result
 
 0 1 2   2 1 5 
to find the values of x, y, z from given equations x – y = 3, 2x + 3y + 4z = 17, y + 2z = 7
 1 1 0   2 2 4
Ans: We have A =  2 3 4  and B =  4 2 4
 
 0 1 2   2 1 5 

Prepared by: M. S. KumarSwamy, TGT(Maths) Page - 11-


 1 1 0   2 2 4   2  4  0 2  2  0 4  4  0 
Now, AB   2 3 4   4 2 4    4  12  8 4  6  4 8  12  20 
    
 0 1 2   2 1 5   0  4  4 0  2  2 0  4  10 
6 0 0 1 0 0 
  0 6 0   6 0 1 0 
 
 0 0 6  0 0 1 
1
 AB  61  A 1  B
6
 2 2 4 
 A   4 2 4 
1 1
6
 2 1 5 
The given system of linear equations can be written in matrix form as AX = B, where
 1 1 0  x  3
A   2 3 4  , X   y  , B  17 
 0 1 2   z   7 
 X  A 1 B
 2 2 4   3   2  3  2  17  4  7 
 X   4 2 4  17   X   4  3  2  17  4  7 
1 1
6 6
 2 1 5   7   2  3  1 17  5  7 
x   2 
  y    1  x  2, y  1, z  4
 z   4 

SECTION – E(Case Study Based Questions)


Questions 36 to 38 carry 4 marks each.
36. Case-Study 1: Read the following passage and answer the questions given below.
Some young entrepreneurs started an industry “Young achievers” for casting metal into various
shapes. They put up an advertisement online stating the same and expecting order to cast method
for toys, sculptures, decorative pieces and more.
A group of friends wanted to make innovative toys and hence contacted the “Young achievers”
to order them to cast metal into solid half cylinders with a rectangular base and semi circular
ends.

(i) If r, h and V are radius, length and volume respectively casted half cylinder, then find the total
surface area function S of the casted half cylinder in terms of V and r.
(ii) For the given volume V, Find the condition for the total surface area S to be minimum.
(iii) Use second derivative test to prove that Surface area is minimum for given volume.
OR
(iii) Find the ratio h : 2r for S to be minimum.

Prepared by: M. S. KumarSwamy, TGT(Maths) Page - 12-


1 2V
Ans: (i) The volume of half - cylinder is, V   r 2 h  h  2 ….. (i)
2 r
Total surface area = area of rectangular base + area of 2 semicircular ends + area of curved surface
1 1
 S  h  2r  2   r 2   2 rh
2 2
S  2rh   r   rh
2

From equation (i) let's substitute the value of h,


 2V   2V  4V 2V
By substituting we get, S  2r   2    r 2   r  2    r2 
r  r  r r
(ii) In order to find the extrema, we need to find the first differentiation and equate it to zero.
dS dS 4V 2V
Therefore, 0   2 r  2  0
dr dr  r 2
r
2V 4V V 2V
 2 r  2  2  r 3   2
r r  
d 2S
(iii) To find whether it’s maxima or minima we need to differentiate again. If  0 then it’s a
dr 2
d 2S
minima and if  0 it’s a maxima.
dr 2
d 2S 1  4V 
 2  2  3   2V 
dr r   
d 2S
As all the numbers are positive, 0
dr 2
Therefore, its minima.
OR
1 2 
3
r
(iii) Now, r 3  V   2   V 
   1 2 
  2
  
Substituting in equation (i) we get,
r3 1
   r 2h
1 2  2
  2
  
2r  1 2  2r 2 2r   2
   2   1  
h    h  h 
h 
 
2r   2

37. Case-Study 2:
In a street two lamp posts are 600 feet apart. The light intensity at a distance d from the first
1000
(stronger) lamp post is , the light intensity at distance d from the second (weaker) lamp post
d2
125
is 2 (in both cases the light intensity is inversely proportional to the square of the distance to
d
the light source). The combined light intensity is the sum of the two light intensities coming from
both lamp posts.

Prepared by: M. S. KumarSwamy, TGT(Maths) Page - 13-


Based on the above information, answer the following questions.
(i) If you are in between the lamp posts, at distance x feet from the stronger light, then find the
formula for the combined light intensity coming from both lamp posts as function of x
(ii) If I(x) denote the combined light intensity, then find the value of x for which I(x) is minimum.
OR
(ii) Find the darkest spot between the two lights
Ans: (i) Since, the distance is x feet from the stronger light, therefore the distance from the weaker
light will be 600 – x.
1000 125
So, the combined light intensity from both lamp posts is given by 
x 2
(600  x) 2
1000 125
(ii) We have, I ( x)  2 
x (600  x) 2
2000 250 6000 750
 I '( x)    I ''( x)  4 
x 3
(600  x)3
x (600  x) 4
2000 250
For maxima/minima, I′(x) = 0 ⇒  3   8(600  x)3  x3
x (600  x) 3

Taking cube root on both sides, we get, 2(600 – x) = x ⇒ 1200 = 3x ⇒ x = 400


Thus, I(x) is minimum when you are at 400 feet from the strong intensity lamp post.
OR
(ii) Since, I(x) is minimum when x = 400 feet, therefore the darkest spot between the two light is
at a distance of 400 feet from stronger lamp post, i.e., at a distance of 600 – 400 = 200 feet from
the weaker lamp post.

38. Case-Study 3:
The reliability of a COVID 19 test is specified as follows:
Of people having COVID 19, 90% of the test detect the disease but 10% go undetected.
Of people free of COVID 19, 99% of the test are judged COVID 19 negative but 1% are diagnosed
as showing COVID 19 positive.
From a large population of which only 0.1% have COVID 19, one person is selected at random,
given the COVID19 test, and the pathologist reports him/her as COVID 19 positive.

Prepared by: M. S. KumarSwamy, TGT(Maths) Page - 14-


(i) What is the probability of the ‘person to be tested as COVID 19 positive’ given that ‘he is
actually having COVID 19’ and What is the probability of the ‘person to be tested as COVID 19
positive’ given that ‘he is actually not having COVID 19’?
(ii) What is the probability that the ‘person is actually not having COVID 19’?
(iii) What is the probability that the ‘person is actually having COVID 19’ given that ‘he is tested
as COVID 19 positive’?
OR
(iii) What is the probability that the ‘person selected will be diagnosed as COVID 19 positive’?
Ans: (i) Let E: the person selected is actually having COVID 19,
E : the person selected is not having COVID 19,
A: person’s COVID19 test is diagnosed as positive.
Clearly, probability of the ‘person to be tested as COVID 19 positive’ given that ‘he is actually
having COVID 19’,
P (A | E) =90% =0.9.
And probability of the ‘person to be tested as COVID 19 positive’ given that ‘he is not actually
having COVID 19’,
P(A | E )= 1% = 0.01
(ii) Refer to (i). P(E) = 0.1% = 0.001
So, P( E ) = 1 – P(E) = 1 – 0.001 = 0.999
(iii) Refer to (i).
We have P(E) = 0.1% = 0.001, P( E ) = 1 – P(E) = 1 – 0.001 = 0.999
P(A|E) = 90% = 0.9 , P(A| E ) = 1% = 0.01
P ( A | E ) P( E )
By Bayes’ Theorem, P( E | A) 
P( A | E ) P( E )  P( A | E ) P( E )

OR
(iii) Refer to (i).
We have P(E) = 0.1% = 0.001, P( E )=1 – P(E) = 1 – 0.001 = 0.999
P(A|E) = 90% = 0.9 , P(A| E ) = 1% = 0.01
We have P ( A)  P ( A | E ) P ( E )  P ( A | E ) P ( E )
 P( A)  (0.9  0.001)  (0.01 0.999)  0.01089

Prepared by: M. S. KumarSwamy, TGT(Maths) Page - 15-

You might also like